Sie sind auf Seite 1von 197

Contents

Page No.

Q Electric Charge, Force and Electric Field


+Q – +
– +
+ + + + + –
1. Electric charge 1
+ – +
+
+ – +
+ – + 2. Comparison of charge and mass 3
+ + –

3. Methods of charging 4
4. Electroscope 5
5. Coulombs law 8
6. Electric field 13

Electric potential and Electric lines

6. Electric potential and potential difference 15


7. Potential due to concentric spheres 27
8. Electric lines of force and equipotential surface 30
9. Relation between electric field and potential 36
10. Work done in displacing a charge 39

11. Conservation of electric field 39


E
Equilibrium, Motion and oscillation of charge and E.P.E.

15. Equilibrium of charge 42


l
O QE 16. Oscillation of charged body 48

mg
17. Neutral point 49
O 18. Zero potential due to a system of two charges 50
19. Electrostatic potential energy (E.P.E.) 52

20. Motion of charged particle in electric field 56

Charged bubble, dipole, flux and Gauss’s law


19. Force on charged conductor 59
20. Equilibrium of charged soap bubble 59
R
21. Electric dipole 60

n̂ 22. Electric flux 67


23. Gauss’s law 67
C o n td … …. .
24. Application of Gauss’s law 71

Capacitor and Capacitance


+Q –Q

27. Capacitance 72
28. Capacitor an it’s types 75

+ – 29. Dielectric 80
30. Variation of different variables (Q, C, V, E and U) of parallel plate capacitor 82
31. Force between the plates of parallel plate capacitor 85
32. Energy density between the plates of parallel plate capacitor 85

C
Grouping of capacitors and RC circuits
C
C C
A C 33. Grouping of capacitor’s 88
C
C 34. Redistribution of charge between two capacitors 88
C C
C B
C C 35. Circuit with resistors and capacitors 89
36. Networking solving 91

Assignments & Answers

Basic Level Advance Level

Page No.
Electric Charge, Force and Electric Field 109-110 111-113

Electric potential and Electric lines 113-116 116-119

Equilibrium, Motion and oscillation of charge and E.P.E. 119-122 122-124

Charged bubble, dipole, flux and Gauss’s law 124-126 126-128


Capacitor and Capacitance 128-132 133-134

Grouping of capacitors and RC circuits 134-138 138-144

Practice Network 145-147 148-151

Answer sheet (Basic and Advance level) 152


– +

Electrostatics
+ 1

– +


+

Electric Charge.

(1) Definition: Charge is the property associated with matter due to which it produces and experiences
electrical and magnetic effects.

(2) Origin of electric charge: It is known that every atom is electrically neutral, containing as many
electrons as the number of protons in the nucleus.

Charged particles can be created by disturbing neutrality of an atom. Loss of electrons gives positive
charge (as then np > ne) and gain of electrons gives negative charge (as then ne > np) to a particle. When an
object is negatively charged it gains electrons and therefore its mass increases negligibly. Similarly, on
charging a body with positive electricity its mass decreases. Change in mass of object is equal to n  me.
Where, n is the number of electrons transferred and m e is the mass of electron  9 .1  10 31 Kg .
Mass M

Electron = Proton
Mass M

+ –
Electron < Proton
Neutral Electron > Proton

Positively charged Negatively charged

M < M M > M

(3) Type: There exists two types of charges in nature (i) Positive charge (ii) Negative charge

Charges with the same electrical sign repel each other, and charges with opposite electrical sign attract each
other.

+ + + – – –
2 Electrostatics

dQ
(4) Unit and dimensional formula: Rate of flow of electric charge is called electric current i.e., i 

dt
dQ  idt , hence S.I. unit of charge is – Ampere  sec = coulomb (C), smaller S.I. units are mC, C, nC
(1mC  10 3 C, 1  C  10 6 C, 1nC  10 9 C ) . C.G.S. unit of charge is – Stat coulomb or e.s.u. Electromagnetic
1
unit of charge is – ab coulomb 1C  3  10 9 stat coulomb  ab coulomb . Dimensional formula [Q]  AT 
10
Note :  Benjamin Franklin was the first to assign positive and negative sign of charge.
 The existence of two type of charges was discovered by Dufog.

 Franklin ( i.e. , e.s.u . of charge) is the smallest unit of charge while faraday is

largest (1 Faraday = 96500 C ).

 The e.s.u. of charge is also called stat coulomb or Franklin (Fr) and is related to e.m.u. of
emu of charge
charge through the relation  3  10 10
esu of charge

(5) Point charge : A finite size body may behave like a point charge if it produces an inverse square electric

field. For example an isolated charged sphere behave like a point charge at very large distance as well as very

small distance close to it’s surface.

(6) Properties of charge

(i) Charge is transferable: If a charged body is put in contact with an uncharged body, uncharged body

becomes charged due to transfer of electrons from one body to the other.

(ii) Charge is always associated with mass, i.e., charge can not exist without mass though mass can exist

without charge.

(iii) Charge is conserved: Charge can neither be created nor be destroyed. e.g. In radioactive decay the

uranium nucleus (charge   92 e ) is converted into a thorium nucleus (charge  90 e ) and emits an  -particle

(charge  2 e )

238
92 U 90 Th 234  2 He 4 . Thus the total charge is  92 e both before and after the decay.

(iv) Invariance of charge: The numerical value of an elementary charge is independent of velocity. It is

proved by the fact that an atom is neutral. The difference in masses on an electron and a proton suggests that

electrons move much faster in an atom than protons. If the charges were dependent on velocity, the neutrality of

atoms would be violated.


Electrostatics 3

(v) Charge produces electric field and magnetic field : A charged particle at rest produces only electric

field in the space surrounding it. However, if the charged particle is in unaccelerated motion it produces both

electric and magnetic fields. And if the motion of charged particle is accelerated it not only produces electric

and magnetic fields but also radiates energy in the space surrounding the charge in the form of

electromagnetic waves.

 
 v = constant v  constant
+ v 0 + +
    
E E and B but no Radiation E , B and Radiates energy

(vi) Charge resides on the surface of conductor: Charge resides on the outer surface of a conductor

because like charges repel and try to get as far away as possible from one another and stay at the farthest

distance from each other which is outer surface of the conductor. This is why a solid and hollow conducting

sphere of same outer radius will hold maximum equal charge and a soap bubble expands on charging.

(vii) Charge leaks from sharp points : In case of conducting body no doubt charge resides on its outer

surface, if surface is uniform the charge distributes uniformly on the surface and for irregular surface the

distribution of charge, i.e., charge density is not uniform. It is maximum where the radius of curvature is

minimum and vice versa. i.e., σ  1 /R  . This is why charge leaks from sharp points.

+
+ +
+ + + + +
+ + +
+ +
+
+ +
+ +
+
+ + +
+
+ + +
+ + + +
+

(viii) Quantization of charge: When a physical quantity can have only discrete values rather than any
value, the quantity is said to be quantised. The smallest charge that can exist in nature is the charge of an
electron. If the charge of an electron (  1 . 6  10 19 C ) is taken as elementary unit i.e. quanta of charge the
charge on any body will be some integral multiple of e i.e.,

Q   ne with n  1, 2, 3 ....

2
Charge on a body can never be  e ,  17 .2 e or  10 5 e etc.
3
4 Electrostatics

Note :  Recently it has been discovered that elementary particles such as proton or neutron are composed
of quarks having charge  1 / 3  e and  2 / 3  e. However, as quarks do not exist in free state, the

quanta of charge is still e.

 Quantization of charge implies that there is a maximum permissible magnitude of charge.

Comparison of Charge and Mass.

We are familiar with role of mass in gravitation, and we have just studied some features of electric charge.
We can compare the two as shown below

Charge Mass

(1) Electric charge can be positive, negative or zero. (1) Mass of a body is a positive quantity.

(2) Charge carried by a body does not depend upon (2) Mass of a body increases with its velocity as
velocity of the body. m0
m  where c is velocity of light in
1  v2 / c2
vacuum, m is the mass of the body moving with
velocity v and m 0 is rest mass of the body.

(3) Charge is quantized. (3) The quantization of mass is yet to be established.

(4) Electric charge is always conserved. (4) Mass is not conserved as it can be changed into

energy and vice-versa.

(5) Force between charges can be attractive or (5) The gravitational force between two masses is

repulsive, according as charges are unlike or like always attractive.

charges.

Methods of Charging.

A body can be charged by following methods :

(1) By friction : In friction when two bodies are rubbed together, electrons are transferred from one body

to the other. As a result of this one body becomes positively charged while the other negatively charged, e.g.,

when a glass rod is rubbed with silk, the rod becomes positively charged while the silk negatively. However,

ebonite on rubbing with wool becomes negatively charged making the wool positively charged. Clouds also

become charged by friction. In charging by friction in accordance with conservation of charge, both positive
Electrostatics 5

and negative charges in equal amounts appear simultaneously due to transfer of electrons from one body to

the other.

(2) By electrostatic induction: If a charged body is brought near an uncharged body, the charged body will

attract opposite charge and repel similar charge present in the uncharged body. As a result of this one side of

neutral body (closer to charged body) becomes oppositely charged while the other is similarly charged. This

process is called electrostatic induction.

Q
+Q – + +Q – +Q – – – ––
+ + + + +
– + + + + + +
– + + + + +
– – –
+ – + – + – – –
+ – + + – + – – –
+
– +
+
– +
– – –
+ +
+
– + + +
+
– + +
+
– – –
–  –  –  – –

Note :  Inducting body neither gains nor loses charge.


 Induced charge can be lesser or equal to inducing charge (but never greater) and its maximum
 1
value is given by Q'  Q 1   where Q is the inducing charge and K is the dielectric
 K

constant of the material of the uncharged body. Dielectric constant of different media are shown

below

Medium K

Vacuum / air 1

Water 80

Mica 6

Glass 5–10

Metal 

 Dielectric constant of an insulator can not be 


6 Electrostatics

 For metals in electrostatics K   and so Q'   Q; i.e. in metals induced charge is equal and

opposite to inducing charge.

(3) Charging by conduction : Take two conductors, one charged and other uncharged. Bring the

conductors in contact with each other. The charge (whether  ve or  ve ) under its own repulsion will spread

over both the conductors. Thus the conductors will be charged with the same sign. This is called as charging

by conduction (through contact).

+ + + + + + + + + + + + + + + + + + + +
+ + + + + + + +
+ + + + + + + +
+ + + +
+ + + +
+ + + + + + + +
+
+ +  + + + +
 + + + +

Uncharged Charged Bodies in contact Both are positively charged

Note :  A truck carrying explosives has a metal chain touching the ground, to conduct away the charge
produced by friction.

Electroscope.

It is a simple apparatus with which the presence of electric charge on a body is detected (see figure). When

metal knob is touched with a charged body, some charge is transferred to the gold leaves, which then diverges

due to repulsion. The separation gives a rough idea of the amount of charge on the body. If a charged body

brought near a charged electroscope the leaves will further diverge. If the charge on body is similar to that on

electroscope and will usually converge if opposite. If the induction effect is strong enough leaves after converging

may again diverge.

(1) Uncharged electroscope

– ––
+ – + – –
+ – – + + – –
– – + + – –
+ + – – + +
+ +



+ + –
+ + –
+ + –
+ + –
+ + + + – –
+ + + + – –
+ + + + – –

(A) D C D
(B) (C)

Charging by conduction Charging by conduction


Electrostatics 7

(2) Charged electroscope

– –– – ––
– – – –
+ – +
+ – – – – + + – –
– – – – + + – –
+ + – – +
+
+ +







+ + – – + +
+ + – – + +
+ + – – + +

(A) D D (C) C
(B)

Concepts

 After earthing a positively charged conductor electrons flow from earth to conductor and if a negatively charged conductor is
earthed then electrons flows from conductor to earth.

+ –
+ –
+ + e– + – –
+ e–
+ + – –
+ –
+ –

 When a charged spherical conductor placed inside a hollow insulated conductor and connected if through a fine conducting
wire the charge will be completely transferred from the inner conductor to the outer conductor.

 Lightening-rods arrestors are made up of conductors with one of their ends earthed while the other sharp, and protects a
building from lightening either by neutralising or conducting the charge of the cloud to the ground.
+Q
 With rise in temperature dielectric constant of liquid decreases.

 Induction takes place only in bodies (either conducting or non-conducting) and not in particles.

 If X-rays are incident on a charged electroscope, due to ionisation of air by X-rays the electroscope will get discharged and
hence its leaves will collapse. However, if the electroscope is evacuated. X-rays will cause photoelectric effect with gold and
so the leaves will further diverge if it is positively charged (or uncharged) and will converge if it is negatively charged.

 If only one charge is available than by repeating the induction process, it can be used to obtain a charge many times
greater than it’s equilibrium. (High voltage generator)

+ + +
+ +
+
+
+
Examples based on properties of charge
+ +

Example: 1 A soap bubble is given negative charge. Its radius will [DCE 2000; RPMT 1997; CPMT 1997; MNR 1988]

(a) Increase (b) Decrease (c) Remain unchanged (d) Fluctuate


8 Electrostatics

Solution: (a) Due to repulsive force.

Example: 2 Which of the following charge is not possible

(a) 1 .6  10 18 C (b) 1 .6  10 19 C (c) 1 .6  10 20 C (d) None of these


1
Solution: (c) 1 .6  10 20 C, because this is of electronic charge and hence not an integral multiple.
10
Example: 3 Five balls numbered 1 to 5 balls suspended using separate threads. Pair (1,2), (2,4) and (4,1) show
electrostatic attraction, while pair (2,3) and (4,5) show repulsion. Therefore ball 1 must be [NCERT 1980]

(a) Positively charged (b) Negatively charged (c) Neutral (d) Made of metal

Solution: (c)Since 1 does not enter the list of repulsion, it is just possible that it may not be having any charge. Moreover,
since ball no. 1 is being attracted by 2 and 4 both. So 2 and 4 must be similarly charged, but it is also
given that 2 and 4 also attract each other. So 2 and 4 are certainly oppositely charged.

Since 1 is attracting 2, either 1 or 2 must be neutral but since 2 is already in the list of balls repelling each
other, it necessarily has some charge, similarly 4 must have some charge. It means that though 1 is
attracting 2 and 4 it does not have any charge.
Example: 4 If the radius of a solid and hollow copper spheres are same which one can hold greater charge

(a) Solid sphere (b) Hollow sphere


(c) Both will hold equal charge (d) None of these

Solution: (c) Charge resides on the surface of conductor, since both the sphere having similar surface area so they will
hold equal charge.

Example: 5 Number of electrons in one coulomb of charge will be

(a) 5 . 46  10 29 (b) 6 . 25  10 18 (c) 1 . 6  10 19 (d) 9  10 11


Q 1
Solution: (b) By using Q  ne  n   n  6 .25  10 18
e 1 .6  10 19
Example: 6 The current produced in wire when 107 electron/sec are flowing in it
(a) 1.6  10–26 amp (b) 1.6  1012 amp (c) 1.6  1026 amp (d) 1.6  10–12 amp
Q ne
Solution: (d) i   10 7  1 . 6  10 19  1 . 6  10 12 amp
t t
Example: 7 A table-tennis ball which has been covered with a conducting paint is suspended by a silk thread so that it
hangs between two metal plates. One plate is earthed. When the other plate is connected to a high
voltage generator, the ball

(a) Is attracted to the high voltage plate and stays there

(b) Hangs without moving

(c) Swings backward and forward hitting each plate in turn

(d) None of these


Electrostatics 9

Solution: (c)The table tennis ball when slightly displaced say towards the positive plate gets attracted towards the positive
plate due to induced negative charge on its near surface.

The ball touches the positive plate and itself gets positively charged by

+
+
the process of conduction from the plate connected to high voltage + + +

+ –
+ + +
generator. On getting positively charged it is repelled by the positive +
+
+ –
+ –
+
plate and therefore the ball touches the other plate (earthed), which has
negative charge due to induction. On touching this plate, the positive charge of the ball gets neutralized
and in turn the ball shares negative charge of the earthed plate and is again repelled from this plate also,
and this process is repeated again and again.

Here it should be understood that since the positive plate is connected to high voltage generator, its

potential and hence its charge will always remain same, as soon as this plate gives some of its charge to

ball, excess charge flows from generator to the plate, and an equal negative charge is always induced on

the other plate.

Tricky example: 1

In 1 gm of a solid, there are 5  1021 atoms. If one electron is removed from everyone of 0.01%
atoms of the solid, the charge gained by the solid is (given that electronic charge is 1.6  10–19 C)

(a) + 0.08 C (b) + 0.8 C (c) – 0.08 C (d) – 0.8 C

Solution: (a) To calculate charge, we will apply formula Q = ne for this, we must have number of electrons.
Here, number of electrons n  . 01 % of 5  1021

5  10 21  . 01
i.e. n   5  10 21  10 4 = 5  1017
100
So Q = 5  1017  1.6  10–19 = 8  10–2 = 0.08 C

Since electrons have been removed, charge will be positive i.e. Q = + 0.08 C

Coulomb’s Law.

If two stationary and point charges Q 1 and Q 2 are kept at a distance r, then it is found that force of

attraction Q1 Q2

Q1 Q 2 kQ 1 Q 2
or repulsion between them is F  2
i.e., F  ; (k = Proportionality constant)
r r2

(1) Dependence of k : Constant k depends upon system of units and medium between the two charges.
10 Electrostatics

(i) Effect of units

Q1 Q 2
(a) In C.G.S. for air k  1, F  Dyne
r2

1 N m 2 1 Q1 Q 2
(b) In S.I. for air k   9  10 9 2
, F . 2 Newton (1 Newton = 105 Dyne)
4  0 C 4  0 r

C2  Farad 
Note :   0  Absolute permittivity of air or free space = 8 . 85  10 12 2
  . It’s Dimension
N m  m 

is [ML3 T 4 A 2 ]

  0 Relates with absolute magnetic permeability (  0 ) and velocity of light (c) according to the
1
following relation c 
0 0

(ii) Effect of medium

(a) When a dielectric medium is completely filled in between charges rearrangement of the charges inside

the dielectric medium takes place and the force between the same two charges decreases by a factor of K

known as dielectric constant or specific inductive capacity (SIC) of the medium, K is also called relative

permittivity r of the medium (relative means with respect to free space).

Fair 1 Q Q
Hence in the presence of medium Fm   . 12 2
K 4  0 K r K
Q1 Q2

Here  0 K   0  r   (permittivity of medium) r

(b) If a dielectric medium (dielectric constant K, thickness t) is partially filled between the charges then

effective air separation between the charges becomes (r  t  t K ) Q1


K
Q2

1 Q1 Q 2 r
Hence force F 
4  0 (r  t  t K ) 2

q1 q 2 q1 q 2
(2) Vector form of coulomb’s law : Vector form of Coulomb’s law is F 12  K . 3
r 12  K . rˆ12 , where
r r2
r̂12 is the unit vector from first charge to second charge along the line joining the two charges.

(3) A comparative study of fundamental forces of nature


Electrostatics 11

S.No. Force Nature and formula Range Relative

strength

(i) Force of gravitation Attractive F = Gm1m2/r2, Long range (between planets 1


between two masses obey’s Newton’s third law of and between electron and
motion, it’s a conservative proton)
force

(ii) Electromagnetic force Attractive as well as repulsive, Long (upto few kelometers) 10 37
(for stationary and obey’s Newton’s third law of
moving charges) motion, it’s a conservative
force

(iii) Nuclear force Exact expression is not known Short (of the order of nuclear 1039
(between nucleons) till date. However in some size 10–15 m) (strongest)
cases empirical formula
U0 e r / r0 can be utilized for
nuclear potential energy U 0
and r0 are constant.

(iv) Weak force (for Formula not known Short (upto 10–15m) 1024
processes like  decay)

Note :  Coulombs law is not valid for moving charges because moving charges produces magnetic field
also.

 Coulombs law is valid at a distance greater than 10 15 m .

 A charge Q1 exert some force on a second charge Q 2 . If third charge Q 3 is brought near, the
force of Q1 exerted on Q 2 remains unchanged.

 Ratio of gravitational force and electrostatic force between (i) Two electrons is 10–43/1. (ii) Two
protons is 10–36/1 (iii) One proton and one electron 10–39/1.

 Decreasing order to fundamental forces FNuclear  FElectromag netic  FWeak  FGravitatio nal

(4) Principle of superposition: According to the principle of super


Q
position, total force acting on a given charge due to number of charges is
r1
the vector sum of the individual forces acting on that charge due to all the
Q1 r2
r3 Qn
charges.
Q2
Qn – 1
Q3
12 Electrostatics

Consider number of charge Q1 , Q 2 , Q 3 …are applying force on a charge Q

Net force on Q will be


    
Fnet  F1  F2  ..........  Fn 1  Fn

Concepts

 Two point charges separated by a distance r in vacuum and a force F acting between them. After filling a dielectric medium
having dielectric constant K completely between the charges, force between them decreases. To maintain the force as

before separation between them changes to r K . This distance known as effective air separation.

Examples based on Coulomb’s law

Example: 8 Two point charges 3 C and 8 C repel each other with a force of 40 N. If a charge of 5 C is added to
each of them, then the force between them will become +Q –Q

(a) 10 N (b) 10 N (c) 20 N (d) 20 N

3  8  10 12 2  3  10 12 F' 1


Solution: (a) Initially F k 2
and Finally F '  k so   F '  10 N
r r2 F 4
Example: 9 Two small balls having equal positive charge Q (coulomb) on each are suspended by two insulated string of

equal length L meter, from a hook fixed to a stand. The whole set up is taken in satellite into space where

there is no gravity (state of weight less ness). Then the angle between the string and tension in the string

is

1 Q2
(a) 180 o , .
4  0 (2 L)2 L
L
2 +Q
1 Q
(b) 90  , . +Q
4  0 L2
1 Q2
(c) 180  , . 2
4  0 2 L
1 QL
(d) 180 o , . 2
4  0 4 L

Solution: (a) In case to weight less ness following situation arises


+Q 180o +Q

L L
Electrostatics 13

1 Q2
So angle   180  and force F  .
4  0 2 L 2

Example: 10 Two point charges 1 C & 5 C are separated by a certain distance. What will be ratio of forces acting on
these two

(a) 1 : 5 (b) 5 : 1 (c) 1 : 1 (d) 0

Solution: (c) Both the charges will experience same force so ratio is 1:1

Example: 11 Two charges of 40 C and 20 C are placed at a certain distance apart. They are touched and kept at
the same distance. The ratio of the initial to the final force between them is

(a) 8 : 1 (b) 4 : 1 (c) 1 : 8 (d) 1 : 1


F1 q q 40  20 8
Solution: (a) Since only magnitude of charges are changes that’s why F  q1q 2   1 2  
F2 q'1 q' 2 10  10 1
Example: 12 A total charge Q is broken in two parts Q1 and Q 2 and they are placed at a distance R from each other.
The maximum force of repulsion between them will occur, when
Q Q Q 2Q Q 3Q Q Q
(a) Q2  , Q1  Q  (b) Q 2  , Q1  Q  (c) Q 2  , Q 1  (d) Q 1  , Q 2 
R R 4 3 4 4 2 2
Q1 Q 2 Q 1 Q  Q 1 
Solution: (d) Force between charges Q1 and Q 2 Fk k
R2 R2

For F to be maximum,
dF
0 i.e.,
d 
 Q1Q  Q12
k
  0 or Q  2Q 1  0, Q1 
Q
dQ1 dQ1  R2  2
Q
Hence Q1  Q 2 
2
Example: 13 The force between two charges 0.06m apart is 5 N. If each charge is moved towards the other by 0.01m,
then the force between them will become

(a) 7.20 N (b) 11.25 N (c) 22.50 N (d) 45.00 N

Solution: (b) Initial separation between the charges = 0.06m

Final separation between the charges = 0.04m


2 2
1 F r  5  0 . 04  4
Since F  2  1   2       F2  11 .25 N
r F2  r1  F2  0 . 06  9

Example: 14 Two charges equal in magnitude and opposite in polarity are placed at a certain distance apart and force
acting between them is F. If 75% charge of one is transferred to another, then the force between the
charges becomes
F 9F 15
(a) (b) (c) F (d) F
16 16 16
Solution: (a) +Q/4 – Q/4
+Q –Q
A B A B
r r
14 Electrostatics

2
Q
k . 
Q2 4 F
Initially F  k Finally F '   2  
r2 r 16
Example: 15 Three equal charges each +Q, placed at the corners of on equilateral triangle of side a what will be the
 1 
force on any charge  k  

 4  0 
kQ 2 2kQ 2 2 kQ 2 3 kQ 2
(a) (b) (c) (d)
a2 a2 a2 a2
Solution: (d)Suppose net force is to be calculated on the charge which is kept at A. Two charges kept at B and C are
FC FB
applying force on that particular charge, with direction as shown in the figure. 60o
A +Q
Q2
Since Fb  Fc  F  k
a2 60o

So, Fnet  FB2  FC2  2 FB FC cos 60


+Q +Q
3 kQ 2
Fnet  3 F  2 B C
a

Example: 16 Equal charges Q are placed at the four corners A, B, C, D of a square of length a. The magnitude of the
force on the charge at B will be

3Q 2 4Q2 1  1 2  Q2  1  Q2
(a) (b) (c)   (d) 2 


 4  a 2
4  0 a 2 4  0 a 2  2  4  a 2 2
  0   0

Solution: (c)After following the guidelines mentioned above FC


FD
Fnet  FAC  FD  FA2  FC2  FD +Q +Q FAC
A FA
B
kQ 2 kQ 2
Since FA  FC  and FD 
a2 (a 2 )2

2 kQ 2 kQ 2 kQ 2  1 Q2 1  2 2  D C
Fnet     2      +Q
a 2
2a 2
a2  2  4  0 a 2  2 
 
Example: 17 Two equal charges are separated by a distance d. A third charge placed on a perpendicular bisector at x
distance, will experience maximum coulomb force when
d d d d
(a) x  (b) x  (c) x  (d) x 
2 2 2 2 2 3
Solution: (c)Suppose third charge is similar to Q and it is q F
F
 
So net force on it Fnet = 2F cos
q
1 Qq x
Where F  . and cos    
4  0  2 d 2  d2 2 2 x x
2
d /4
2
x   x2  x d /4
 4  4
 

Q Q
B d d C
2 2
Electrostatics 15

1 Qq x 2 Qqx
 Fnet  2  .  1/2
 3/2
4  0  2 d 2   2 d2   d 2 
x   x   4  0  x 2 
 4  
   4  
 4 

 
 
dFnet d  2 Qqx 
for Fnet to be maximum  0 i.e.  3/2
0
dx dx   2 d2  
 4  0  x  
 
  4  
 3 / 2 5 / 2 
2  2 
 x 2  d  2 2 d 0 d
or  3x  x   i.e. x 
 4  
 4 
  2 2
 

Example: 18 ABC is a right angle triangle in which AB = 3 cm, BC = 4 cm and ABC  . The three charges +15, +12
2
and – 20 e.s.u. are placed respectively on A, B and C. The force acting on B is

(a) 125 dynes (b) 35 dynes (c) 25 dynes (d) Zero

Solution: (c)Net force on B Fnet  FA2  FC2

15  12
FA   20 dyne
3 2 A +15 esu

12  20
FC   15 dyne
4 2 3 cm

Fnet  25 dyne B FC C
+12 esu – 20 esu
4 cm
2 2
Fnet  FA  FC
FA

Example: 19 Five point charges each of value +Q are placed on five vertices of a regular hexagon of side L. What is
the magnitude of the force on a point charge of value – q placed at the centre of the hexagon

Q2 Q2
(a) k (b) k
L2 4 L2
(c) Zero (d) Information is insufficient
Solution: (a)Four charges cancels the effect of each other, so the net force on the charge placed at centre due to
remaining fifth charge is +Q +Q
2
Q
Fk –Q
L2 +Q

+Q +Q
L
16 Electrostatics

Example: 20 Two small, identical spheres having +Q and – Q charge are kept at a certain distance. F force acts
between the two. If in the middle of two spheres, another similar sphere having +Q charge is kept, then it
experience a force in magnitude and direction as

(a) Zero having no direction (b) 8F towards +Q charge

(c) 8F towards – Q charge (d) 4F towards +Q charge

Q2
Solution: (c)Initially, force between A and C Fk
r2
+Q +Q –Q
When a similar sphere B having charge +Q is kept at the mid point
A B C
of line joining A and C, then Net force on B is FA FC
r/2 r/2
Q2 kQ 2 kQ 2 r
Fnet  FA  FC  k  8  8F . (Direction is
r 2 2
r 2 
2
r2
shown in figure)

Tricky example: 2

Two equal spheres are identically charged with q units of electricity separately. When they are
placed at a distance 3R from centre-to-centre where R is the radius of either sphere the force of
repulsion between them is

1 q2 1 q2 1 q2
(a) . 2 (b) . (c) . (d) None of these
4  0 R 4 0 9 R 2 4 0 4 R 2

1 q2
Solution: (a) Generally students give the answer but it is not true. Since the charges are not
4  0 (3 R ) 2
uniformly distributed, they cannot be treated as point charges and so we cannot apply coulombs
law which is a law for point charges. The actual distribution is shown in the figure above.
+ +
+ +
+ + +
+ +
+ + + +
+ +
+ + + +
+ + +
+

Electrical Field.

A positive charge or a negative charge is said to create its field around itself. If a charge Q1 exerts a force

on charge Q 2 placed near it, it may be stated that since Q 2 is in the field of Q1 , it experiences some force, or it
Electrostatics 17

may also be said that since charge Q1 is inside the field of Q 2 , it experience some force. Thus space around a

charge in which another charged particle experiences a force is said to have electrical field in it.

(1) Electric field intensity (E) : The electric field intensity at any point is defined as the force experienced

 F
by a unit positive charge placed at that point. E  +Q (q0) 
q0 F
P

Where q 0  0 so that presence of this charge may not affect the source charge Q and its electric field is

 F
not changed, therefore expression for electric field intensity can be better written as E  Lim
q 0 0 q
0

Newton volt Joule


(2) Unit and Dimensional formula : It’s S.I. unit –   and
coulomb meter coulomb  meter

C.G.S. unit – Dyne/stat coulomb.

Dimension : [ E ] =[ MLT 3 A 1 ]

(3) Direction of electric field : Electric field (intensity) E is a vector quantity. Electric field due to a positive

charge is always away from the charge and that due to a negative charge is always towards the charge


+Q E –Q 
E


(4) Relation between electric force and electric field : In an electric field E a charge (Q) experiences a

force F  QE . If charge is positive then force is directed in the direction of field while if charge is negative force

acts on it in the opposite direction of field

E E
+Q F –Q

(5) Super position of electric field (electric field at a point due to various charges) : The resultant electric
field at any point is equal to the vector sum of electric fields at that point due to various charges.
   
E  E1  E 2  E 3  ...

The magnitude of the resultant of two electric fields is given by E2 E




E E12  E 22  2 E1 E 2 cos  and the direction is given by
E1
18 Electrostatics

E 2 sin 
tan  
E1  E 2 cos 

(6) Electric field due to continuous distribution of charge : A system of closely spaced electric charges

forms a continuous charge distribution

Continuous charge distribution

Linear charge distribution Surface charge distribution Volume charge distribution

In this distribution charge distributed In this distribution charge distributed In this distribution charge distributed

on a line. on the surface. in the whole volume of the body.

For example : charge on a wire, For example : Charge on a For example : Non conducting

charge on a ring etc. Relevant conducting sphere, charge on a charged sphere. Relevant

parameter is  which is called sheet etc. Relevant parameter is parameter is  which is called

linear charge density i.e.,  which is called surface charge volume charge density i.e.,
Q +
charge + density i.e., Q + charge
 +  Q+
length + + volume +
+ R + +
+
charge + R + +
+ +  
Q
+ + R +
Q + + area + + +
+ + +
  +
2R + + 4
Circular charged Q R 3 + +
  Spherical shall 3
ring Non conducting
4R 2
sphere

To find the field of a continuous charge distribution, we divide the charge into infinitesimal charge

elements. Each infinitesimal charge element is then considered, as a point charge and electric field dE is

determined due to this charge at given point. The Net field at the given point is the summation of fields of all

the elements. i.e., E  dE


Electric Potential.

(1) Definition : Potential at a point in a field is defined as the amount of work done in bringing a unit

positive test charge, from infinity to that point along any arbitrary path (infinity is point of zero potential). Electric
W
potential is a scalar quantity, it is denoted by V; V 
q0

Joule 1
(2) Unit and dimensional formula : S. I. unit –  volt C.G.S. unit – Stat volt (e.s.u.); 1 volt 
Coulomb 300
Stat volt Dimension – [V ]  [ML2 T 3 A 1 ]
Electrostatics 19

(3) Types of electric potential : According to the nature of charge potential is of two types

(i) Positive potential : Due to positive charge. (ii) Negative potential: Due to negative charge.

(4) Potential of a system of point charges : Consider P is a point at which net electric potential is to be

determined due to several charges. So net potential at P

Q1 Q Q  Q 4  P
V k k 2 k 3 k  ...
r1 r2 r3 r4
r1

X
kQ i r2
In general V  i1 ri
– Q1 r3
r4

– Q2 – Q4
+ Q3

Note :  At the centre of two equal and opposite charge V = 0 but E0

 At the centre of the line joining two equal and similar charge V  0, E  0

(5) Electric potential due to a continuous charge distribution : The potential due to a continuous charge

distribution is the sum of potentials of all the infinitesimal charge elements in which the distribution may be
dQ

divided i.e., V  dV ,   4 πε 0r

(6) Graphical representation of potential: When we move from a positive charge towards an equal

negative charge along the line joining the two then initially potential decreases in magnitude and at centre

become zero, but this potential is throughout positive because when we are nearer to positive charge, overall

potential must be positive. When we move from centre towards the negative charge then though potential

remain always negative but increases in magnitude fig. (A). As one move from one charge to other when both

charges are like, the potential first decreases, at centre become minimum and then increases Fig. (B).

+q –q +q +q

Y Y
V v

O X
x X
O
x
(A) (B)
20 Electrostatics

(7) Potential difference : In an electric field potential difference between two points A and B is defined as

equal to the amount of work done (by external agent) in moving a unit positive charge from point A to point B.

W
i.e., VB  V A  in general W  Q. V ; V  Potential difference through which charge Q moves.
q0

Electric Field and Potential Due to Various Charge Distribution.

(1) Point charge : Electric field and potential at point P due to a point charge Q is

Q  Q  1  Q
Ek 2
or E  k 2 rˆ  k   , V k Q P
r r  4  0  r
r

Note :  Electric field intensity and electric potential due to a point charge q, at a distance t1 + t2 where t1 is
thickness of medium of dielectric constant K1 and t2 is thickness of medium of dielectric constant K2

are:

1 Q 1 Q
E ; V 
4 πε 0 (t 1 K 1  t 2 K 2 ) 2 4 πε 0 (t 1 K 1  t 2 K 2 )

(2) Line charge

(i) Straight conductor: Electric field and potential due to a charged straight conducting wire of length l and

charge density  Ey
+

k k
+

(a) Electric field : E x  (sin   sin  ) and E y  (cos   cos  ) 


+
l P
r r + r

Ex
2k 
+

If  = ; Ex  sin  and Ey = 0
r
 2k  
If l   i.e.  =  = ; Ex  and Ey = 0 so Enet 
2 r 20r

 k 2 k
If  = 0,   ; | E x| | Ey|  so Enet  Ex2  Ey2 
2 r r

  r2  l2  1  
(b) Potential : V  log e   for infinitely long conductor V  log e r  c
2 0  r  l  1 
2 2 2 0
Electrostatics 21

(ii) Charged circular ring: Suppose we have a charged circular ring of radius R and charge Q. On it’s axis
electric field and potential is to be determined, at a point ‘x’ away from the centre of the ring.
+
+ dQ
+
+ E
+
R +
+ +  P
dE cos 
+ x 
R O
+  x 
R x
+ + dE 2
dE sin  2
+ +
+

KdQ
(a) Electric field: Consider an element carrying charge dQ . It’s electric field dE 
 R2  x2  directed as
shown. It’s component along the axis is dE cos  and perpendicular to the axis is dE sin  . By
kdQ x

symmetry dE sin  0 , hence E  dE cos   2 2
. 2 
(R  x ) (R  x 2 )1 2 
kQx
E directed away from the centre if Q is positive
R 2
 x2  3 2

1 Q
(b) Potential : V  .
4  0 x  R2
2

kQ
Note :  At centre x = 0 so Ecentre= 0 and Vcentre 
R
kQ kQ
 At a point on the axis such that x >> R E  2
and V 
x x
R Q
 At a point on the axis if x   , E max 
2 6 3 0 a 2
(3) Surface charge :

(i) Infinite sheet of charge : Electric field and potential at a point P as shown


E (E  r o ) +
2 0 + +
+ +
+ +
+ + P
+ + r
r +
+ + +
and V  C +
2 0

(ii) Electric field due to two parallel plane sheet of

charge : Consider two large, uniformly charged parallel. A B


+ +
+ + + +
+ + + +
Plates A and B, having surface charge densities are  A EA + + EB EA + + + + EA
+ + 
EB  + +
+ +  + +
+ +
EB
P + + + Q + + + R
+ +
22 Electrostatics

and  B respectively. Suppose net electric field at points P, Q and R is to be calculated.

1
At P, E P  (E A  E B )  ( A   B )
2 0

1 1
At Q, E Q  (E A  E B )  ( A   B ) ; At R, E R  (E A  E B )   ( A   B )
2 0 2 0

Note :  If  A   and  B   then E p  0, E Q  , E R  0 . Thus in case of two infinite plane
0

sheets of charges having equal and opposite surface charge densities,


+ –
+ –
the field is non-zero only in the space between the two sheets and is
+ E=/0 –
independent of the distance between them i.e., field is uniform in this + –

+ –
region. It should be noted that this result will hold good for finite plane + –

sheet also, if they are held at a distance much smaller then the

dimensions of sheets i.e., parallel plate capacitor.

(iii) Conducting sheet of charge:

 +
E + +
0 +
+ +
+ + P
r
r +
V  C +
0

(iv) Charged conducting sphere: If charge on a conducting sphere of radius R is Q as shown in figure

then electric field and potential in different situation are –

+Q +Q
+ + + + +
+
+ + + +
+ R + + R +

+ + + +
+ + + +
+ + + +
Solid Hollow

(a) Out side the sphere: P is a point outside the sphere at a distance r from the centre at which electric

field and potential is to be determined.


Electrostatics 23

Electric field at P

1 Q R 2 1 Q R 2 Q    A
E out  . 2  and Vout  .  
4  0 r 0r2 4  0 r  0 r     4 R 2

(b) At the surface of sphere : At surface r  R

1 Q  1 Q R
So, Es  . 2  and Vs  . 
4  0 R 0 4  0 R  0

(c) Inside the sphere : Inside the conducting charge sphere electric field is zero and potential remains

constant every where and equals to the potential at the surface.

E in  0 and Vin = constant  Vs

Note :  Graphical variation of electric field and potential of a charged spherical conductor with distance

E - r graph V - r graph
R R
O O

E VS
1 1
E out  Vout 
r
2 r

Ein=0 r
O r O r =R

(4) Volume charge (charged non-conducting sphere) :

Charge given to a non conducting spheres spreads uniformly throughout it’s volume.

(i) Outside the sphere at P


+
+
++ +Q
1 Q 1 Q Q + + +
E out  . 2 and Vout  . by using   + +
+
4  0 r 4  0 r 4 + + + R + + P
R 3 + + + + +
3 + + +
+
+
+
+ +
R 3 R 3 + + +
E out  and Vout  r
3 0 r 2 3 0 r

(ii) At the surface of sphere : At surface r  R

1 Q R 1 Q R 2
Es  . 2  and Vs  . 
4  0 R 3 0 4  0 R 3 0
24 Electrostatics

(iii) Inside the sphere : At a distance r from the centre

1 Qr r 1 Q [3 R 2  r 2 ]  (3 R 2  r 2 )
E in  . 3  E in  r and Vin  
4  0 R 3 0 4  0 2R 3 6 0

3 1 Q 3
Note :  At centre r  0 So, Vcentre   .  Vs i.e., V centre  V surface  V out
2 4  0 R 2

 Graphical variation of electric field and potential with distance

E-r graph + V-r graph +


+ + + + + +
+ + + +
+
O R +
O R
+ + + + + + + +
+ + + +
+ + +
+

E VC
1
E out  1
Ein  r 2 VS Vout 
r
r

r r
O r=R r=R

(5) Electric field and potential in some other cases

charge
(i) Uniformly charged semicircular ring :  
length
+Q
At centre : +
+ + +
+
+ +
2 K Q
E  + +

R 2  0 R 2
2
+ +

R
KQ Q
V  
R 4 0 R

(iii) Charged cylinder of infinite length

(a) Conducting (b) Non-conducting R


+ + +
+
+ + +
+
+ +
P + + P
r + + +r
+
+ + +
+

 
For both type of cylindrical charge distribution E out  , and E suface  but for conducting
2 0 r 2 0 R
r
E in  0 and for non-conducting E in  . (we can also write formulae in form of  i.e.,
2 0 R 2
Electrostatics 25

R 2
E out  etc.)
2 0 r

(ii) Hemispherical charged body :


+ + +
 +
At centre O, E +
+
+ + +
+ +
4 0 + + + + + +
+ +
O + + +

R
V
2 0

(iv) Uniformly charged disc

At a distance x from centre O on it’s axis



  x  R
E 1  
2 0  x 2  R 2  O
x

  2
V  x  R 2  x
2 0  

Note :  Total charge on disc Q = R2



 If x  0, E ~– i.e. for points situated near the disc, it behaves as an infinite sheet of
2 0
charge.

Concepts

 No point charge produces electric field at it’s own position.

 + on it’s surface hence electric field inside it is zero .


Since charge given to a conductor resides
+ + + + +
+ + + +
+ +
+ + + +
+ + E=0 +
E=0 +
+ +
+
+ + +
+ + + + + +

 The electric field on the surface of a conductor is directly proportional to the surface charge density at that point i.e, E  

 Two charged spheres having radii r1 and r2 charge densities  1 and  2 respectively, then the ratio of electric field on
26 Electrostatics

E1  r2  Q
their surfaces will be  1  2  
E2  2 r 2
1  4 r 2

 In air if intensity of electric field exceeds the value 3  10 6 N/C air ionizes.

 A small ball is suspended in a uniform electric field with the help of an insulated thread. If a high energy x–ray bean falls on
the ball, x-rays knock out electrons from the ball so the ball is positively charged and therefore the ball is deflected in the
direction of electric field.

 Electric field is always directed from higher potential to lower potential.

 A positive charge if left free in electric field always moves from higher potential to lower potential while a negative charge
moves from lower potential to higher potential.

 The practical zero of electric potential is taken as the potential of earth and theoretical zero is taken at infinity.

 An electric potential exists at a point in a region where the electric field is zero and it’s vice versa.

 A point charge +Q lying inside a closed conducting shell does not exert force another point charge q placed outside the
shell as shown in figure
+ +
+ +
+Q
+ – + +q
– –
+ – +
+Q –
+ – – +
+ +
+ +

Actually the point charge +Q is unable to exert force on the charge +q because it can not produce electric field at the
position of +q. All the field lines emerging from the point charge +Q terminate inside as these lines cannot penetrate the
conducting medium (properties of lines of force).

The charge q however experiences a force not because of charge +Q but due to charge induced on the outer surface of the shell.

–e E0 +e
V=0 Examples based on electric field and electric potential

 1 
Example: 21 A half ring of radius R has a charge of  per unit length. The electric field at the centre is  k  

 4  0 

k 2k k
(a) Zero (b) (c) (d)
R R R
Solution: (c) dl  Rd 
Electrostatics 27

dl
d
 C
Charge on dl  Rd  .  

Rd  .
Field at C due to dl  k  dE dl dE
R2
We need to consider only the component dE cos  , as the component dE sin  will cancel out because of
the field at C due to the symmetrical element dl,
 2 k  2   Q 
The total field at C is  2 0
dE cos   2
R 
0
cos  d   2k
R
 2 
 2 0 R 

Example: 22 What is the magnitude of a point charge due to which the electric field 30 cm away has the magnitude 2
newton/coulomb [1 / 4 0  9  10 9 Nm 2 ]
(a) 2  10 11 coulomb (b) 3  10 11 coulomb (c) 5  10 11 coulomb (d) 9  10 11 coulomb
1 Q Q
Solution: (a) By using E  . 2 ; 2  9  10 9   Q  2  10 11 C
4  0 r 30  10  2 2

Example: 23 Two point charges Q and – 3Q are placed at some distance apart. If the electric field at the location of Q
is E, then at the locality of – 3Q, it is
(a)  E (b) E/3 (c) 3 E (d) – E/3
Solution: (b) Let the charge Q and – 3Q be placed respectively at A and B at a distance x

Now we will determine the magnitude and direction to the field produced by charge – 3Q at A, this is E as
A B
mentioned in the Example.
Q –3Q
3Q
E (along AB directed towards negative charge) x
x2
Q E
Now field at location of – 3Q i.e. field at B due to charge Q will be E'   (along AB directed away
x2 3
from positive charge)

Example: 24 Two charged spheres of radius R 1 and R 2 respectively are charged and joined by a wire. The ratio of
electric field of the spheres is

R1 R2 R 12 R 22
(a) (b) (c) (d)
R2 R1 R 22 R 12
Q1 k . Q2 Q1 R
Solution: (b)After connection their potential becomes equal i.e., k .  ;   1
R1 R2 Q2 R2
2
E Q R  R
Ratio of electric field 1  1   2   2 .
E2 Q 2  R1  R1
28 Electrostatics

Example: 25 The number of electrons to be put on a spherical conductor of radius 0.1m to produce an electric field of

0.036 N/C just above its surface is

(a) 2 .7  10 5 (b) 2 .6  10 5 (c) 2 .5  10 5 (d) 2 .4  10 5

Q ne
Solution: (c) By using E  k , where R = radius of sphere so 0.036 = 9  10 9   n  2 .5  10 5
R 2
0 .12
Example: 26 Eight equal charges each +Q are kept at the corners of a cube. Net electric field at the centre will be

 1 
 k  
 4  0 

kQ 8 kQ 2kQ
(a) 2
(b) 2
(c) (d) Zero
r r r2

Solution: (d) Due to the symmetry of charge. Net Electric field at centre is zero.
+q q a q

Note :
a a a E=0 a
E=0

+q +q q
a q a
Equilateral triangle Square

Example: 27 q, 2q, 3q and 4q charges are placed at the four corners A, B, C and D of a square. The field at the centre
O of the square has the direction along.
q 2q
A B

D C
4q 3q

(a) AB (b) CB (c) AC (d) BD

Solution: (b) By making the direction of electric field due to all charges at centre. Net electric field has the direction along

CB
Electrostatics 29

Example: 28 Equal charges Q are placed at the vertices A and B of an equilateral triangle ABC of side a. The

magnitude of electric field at the point A is

Q 2Q 3Q Q
(a) (b) (c) (d)
4  0 a 2 4  0 a 2 4  0 a 2 2 0 a 2
E
FC FB
Solution: (c) As shown in figure Net electric field at A
A

E E B2  EC2  2 E B EC cos 60 60o


a a
1 Q
E B  EC  .
4  0 a 2
+Q +Q
a C
B
3Q
So, E 
4  0 a 2

Example: 29 Four charges are placed on corners of a square as shown in figure having side of 5 cm. If Q is one micro

coulomb, then electric field intensity at centre will be

(a) 1 . 02  10 7 N / C upwards Q – 2Q

(b) 2 . 04  10 7 N / C downwards

(c) 2 . 04  10 7 N / C upwards –Q + 2Q

(d) 1 . 02  10 7 N / C downwards

Solution: (a) | E C|  | E A| so resultant of EC & E A is ECA  EC  E A directed toward Q


Q – 2Q
A ECA Enet EBD B
Also | E B |  | E D| so resultant of EB and ED i.e.
EC ED
EBD  EB  ED directed toward – 2Q charge hence Net electric field at centre is
EB
O
EA

E ECA 2  EBD 2 .… (i)


C
D
–Q +2Q
10 6
By proper calculations | E A |  9  10 9  2
 0 . 72  10 7 N /C
 5 

  10  2 
 2 

2  10 6 2  10 6
| E B |  9  10 9  2
 1 . 44  10 7 N /C ; | E C |  9  10 9  2
 1 . 44  10 7 N /C
 5   5 

  10  2  
  10  2 
 2   2 
30 Electrostatics

10 6
| E D |  9  10 9  2
 0 . 72  10 7 N /C ; So, |E CA|  |E C|  |E A|  0 .72  10 7 N/C
 5 

  10  2 
 2 

and | E BD | | E B |  | E D |  0 . 72  10 7 N /C. Hence from equation – (i) E  1 . 02  10 7 N /C upwards

Example: 30 Infinite charges are lying at x = 1, 2, 4, 8…meter on X-axis and the value of each charge is Q. The value

of intensity of electric field and potential at point x = 0 due to these charges will be respectively

(a) 12  10 9 Q N/C, 1.8  104 V (b) Zero, 1.2  104V

(c) 6  10 9 Q N/C, 9  103 V (d) 4  10 9 Q N/C , 6  103 V

Solution: (a) By the superposition, Net electric field at origin

1 1 1 1 
E  kQ  2  2  2  2  ...  
1 2 4 8 
x=0 x=1 x=2 x=4 x=8
 1 1 1 
E  kQ 1     ...  
 4 16 64 

1 1 1
1    ...  is an infinite geometrical progression it’s sum can be obtained by using the formula
4 16 64
a
S  ; Where a = First term, r = Common ratio.
1r
1 1 1 1 1 4
Here a  1 and r  so, 1     .....    .
4 4 16 64 1 1/ 4 3

4
Hence E  9  10 9  Q   12  10 9 Q N /C
3

1  1  10 6 1  10 6 1  10 6 1  10 6 
Electric potential at origin V       .......  
4  0  1 2 4 8 

 
 1 1 1   1 
 9  10 9  10 6 1     .......... ..   9  10 3  4
  1 . 8  10 volt
 2 4 8  1
1  
 2 

Note :  In the arrangement shown in figure +Q and – Q are alternatively and equally spaced from each
1 Q log e 2
other, the net potential at the origin O is V  .
4 0 x

+Q –Q +Q –Q
O x 2x 3x 4x
Electrostatics 31

Example: 31 Potential at a point x-distance from the centre inside the conducting sphere of radius R and charged with
charge Q is
Q Q Q
(a) (b) (c) (d) xQ
R x x2
Solution: (a) Potential inside the conductor is constant.

Example: 32 The electric potential at the surface of an atomic nucleus (Z = 50) of radius 9  10 5 V is

(a) 80 V (b) 8  10 6 V (c) 9 V (d) 9  10 5 V

ne 50  1 . 6  10 19
Solution: (b) V  9  10 9   9  10 9   8  10 6 V
r 9  10 15

Example: 33 Eight charges having the valves as shown are arranged symmetrically on a circle of radius 0.4m in air.
Potential at centre O will be
+5 C
– 7 C – 5 C

+15 C O +7 C

+7 C – 5 C

+11 C

(a) 63  10 4 volt (b) 63  10 10 volt (c) 63  10 6 volt (d) Zero

Solution: (a) Due to the principle of superposition potential at O

1 28  10 6 28  10 6
V    9  10 9   63  10 4 volt
4  0 0 .4 0 .4

Example: 34 As shown in the figure, charges +q and –q are placed at the vertices B and C of an isosceles triangle. The
potential at the vertex A is
A

b b
B C
+q –q

1 2q 1 q 1 (q )
(a) . (b) . (c) . (d) Zero
4  0 a  b2
2 4  0 a  b2
2 4  0 a2  b 2
Solution: (d) Potential at A = Potential due to (+q) charge + Potential due to (– q) charge
32 Electrostatics

1 q 1 (q )
 .  0
4  0 a2  b 2 4  0 a2  b 2
Example: 35 A conducting sphere of radius R is given a charge Q. consider three points B at the surface, A at centre
and C at a distance R/2 from the centre. The electric potential at these points are such that
(a) VA = VB = VC (b) VA = VB  VC (c) VA  VB  VC (d) VA  VB = VC
Solution: (a)Potential inside a conductor is always constant and equal to the potential at the surface.
10
Example: 36 Equal charges of  10 9 coulomb are lying on the corners of a square of side 8 cm. The electric
3
potential at the point of intersection of the diagonals will be
(a) 900 V (b) 900 2 V (c) 150 2 V (d) 1500 2 V
Solution: (d) Potential at the centre O
Q Q
1 Q 10
V 4 . given Q   10 9 C  a  8 cm  8  10 2 m a 2
4  0 a / 2 3
10 O
 10 9
9
V  5  9  10  3  1500 2 volt
8  10  2 Q Q
a
2

Tricky example: 3

A point charge Q is placed outside a hollow spherical conductor of radius R, at a distance (r > R) from
 1 
its centre C. The field at C due to the induced charges on the conductor is  K  
 4  0 
Q
(a) Zero (b) K
r  R 2
Q Q
(c) K 2 directed towards Q (d) K 2 directed away from Q
r r
Solution: (c) A according to the figure shown below. The total field at C must be zero. The field at C due to the
Q KQ
point charge is E  K 2 towards left. The field at C due to the induced charges must be 2
r r
towards right i.e. directed towards Q.

+ –
+ –
+ –
C +Q
+
+ –
R
+ –
+ –
r
Electrostatics 33

Tricky example: 4

A point charge q is placed at a distance of r from the centre of an uncharged conducting sphere of
radius R (< r). The potential at any point on the sphere is

1 q 1 qR 1 qr 2
(a) Zero (b) . (c) . 2 (d) .
4  0 r 4  0 r 4  0 R
Solution: (c) Since, potential V is same for all points of the sphere. Therefore, we can calculate its value at the
centre of the sphere.
1 q
 V  .  V ' ; where V = potential at centre due to induced charge = 0 (because net
4  0 r
1 q
induced charge will be zero)  V  . .
4  0 r

Potential Due to Concentric Spheres.

To find potential at a point due to concentric sphere following guideline are to be considered

Guideline 1: Identity the point (P) at which potential is to be determined.

Guideline 2: Start from inner most sphere, you should know where point (P) lies w.r.t. concerning

sphere/shell (i.e. outside, at surface or inside)

Guideline 3: Then find the potential at the point (P) due to inner most sphere and then due to next and so on.

Guideline 4: Using the principle of superposition find net potential at required shell/sphere.

Standard cases

Case (i) : If two concentric conducting shells of radii r1 and r2(r2 > r1) carrying uniformly distributed charges Q1 and
Q2 respectively. What will be the potential of each shell
Q2
To find the solution following guidelines are to be taken. Q1

Here after following the above guideline potential at the surface of inner shell is r2

1 Q 1 Q r1
V1  . 1  . 2
4  0 r1 4  0 r2

and potential at the surface of outer shell


1 Q 1 Q
V2  . 1  . 2
4 0 r2 4 0 r2
34 Electrostatics

Case (ii) : The figure shows three conducting concentric shell of radii a, b and c (a < b < c) having charges Qa, Qb
and Qc respectively what will be the potential of each shell
Qc
After following the guidelines discussed above Qb
Qa
1  Qa Qb Qc 
Potential at A; V A    c
4  0  a b c  b a A B

1  Qa Qb Qc 
Potential at B; VB   
4  0  b b c 

1  Qa Qb Qc 
Potential at C; VC   
4  0  c c c 

Case (iii) : The figure shows two concentric spheres having radii r1 and r2 respectively (r2 > r1). If charge on inner sphere is +Q
and outer sphere is earthed then determine.

(a) The charge on the outer sphere +Q


Q
r2
(b) Potential of the inner sphere
1 Q 1 Q' r1
(i) Potential at the surface of outer sphere V2  .  . 0
4 0 r2 4 0 r2
 Q'  Q
1 Q 1 (Q) Q 1 1
(ii) Potential of the inner sphere V1  .     
4  0 r1 4  0 r2 4  0  r1 r2 
Case (iv) : In the case III if outer sphere is given a charge +Q and inner sphere is earthed then

(a) What will be the charge on the inner sphere

(b) What will be the potential of the outer sphere

(i) In this case potential at the surface of inner sphere is zero, so if Q' is the charge induced on inner sphere

1  Q' Q  r1 r2 +Q
then V1      0 i.e., Q'   Q
4  0  r1 r2  r2
r1
(Charge on inner sphere is less than that of the outer sphere.)

(ii) Potential at the surface of outer sphere


1 Q' 1 Q
V2  .  .
4 0 r2 4 0 r2

1  r1  Q  r1 
V2   Q  Q   1  
4  0 r2  r2  4 0 r2  r2 
Electrostatics 35

Examples based on concentric spheres

Example: 37 A hollow metal sphere of radius 5 cm is charged such that the potential on its surface is 10 volts. The
potential at the centre of the sphere is

(a) Zero (b) 10 V

(c) Same as at a point 5 cm away from the surface (d) Same as at a point 25 cm away from the
surface

Solution: (b)Inside the conductors potential remains same and it is equal to the potential of surface, so here potential at
the centre of sphere will be 10 V

Example: 38 A sphere of 4 cm radius is suspended within a hollow sphere of 6 cm radius. The inner sphere is charged
to a potential 3 e.s.u. When the outer sphere is earthed. The charge on the inner sphere is

1
(a) 54 e.s.u. (b) e.s.u. (c) 30 e.s.u. (d) 36 e.s.u.
4

Solution: (d)Let charge on inner sphere be +Q. charge induced on the inner surface of outer sphere will be –Q.

So potential at the surface of inner sphere (in CGS) +Q

Q Q
3 
4 6 4cm

 Q  36 e.s.u. 6cm

Example: 39 A charge Q is distributed over two concentric hollow spheres of radii r and (R  r) such that the surface
densities are equal. The potential at the common centre is

Q(R 2  r 2 ) Q Q(R  r)
(a) (b) (c) Zero (d)
4  0 (R  r) R r 4  0 (R 2  r 2 )

Solution: (d)If q1 and q 2 are the charges on spheres of radius r and R respectively, in accordance with conservation of
charge
q2
Q  q1  q 2 ….(i)
q1
and according to the given problem  1   2
r
q1 q2 q1 r2 R
i.e.,    2 …. (ii)
4 r 2 4 R 2 q2 R

Qr 2 QR 2
So equation (i) and (ii) gives q1  2 2
and q 2 
(R  r ) (R 2  r 2 )
36 Electrostatics

1  q1 q 2  1  Qr QR  1 Q ( R  r)
Potential at common centre V        .
4  0  r R  4  0  (R 2  r 2 ) (R 2  r 2 )  4  0 (R 2  r 2 )

Example: 40 A solid conducting sphere having a charge Q is surrounded by an uncharged concentric conducting
hollow spherical shell. Let the potential difference between the surface of the solid sphere and that of the
outer surface of the hollow shell be V. If the shell is now given a charge of – 3Q, the new potential
difference between the two surfaces is

(a) V (b) 2V (c) 4V (d) –2V

Solution: (a)If a and b are radii of spheres and spherical shell respectively, potential at their surfaces will be
1 Q 1 Q
Vsphere  . and Vshell  . Q
4  0 a 4  0 b +
+ + +
+ Sphere +
and so according to the given problem. + +
+
a +
Q 1 1  b
+ + +
V  Vsphere  Vshell  a  b  …. (i)
4  0  
Now when the shell is given a charge –3Q the potential at its surface and also inside will change by
1  3Q 
V0   b 
4  0  
1  Q 3Q  1  Q 3Q 
So that now Vsphere   and Vshell   hence
4  0  a b  4  0  b b 
Q 1 1 
Vsphere  Vshell  a  b   V
4  0  
Example: 41 Three concentric metallic spheres A, B and C have radii a, b and c (a  b  c) and surface charge
densities on them are  ,   and  respectively. The valves of VA and VB will be

   a2 
(a) (a  b  c),   b  c
0 0  b  C
 
B
a2 c
A b
(b) (a  b  c),
c   
a
   a2 
(c) 0 (a  b  c), 0   b  c
   c 
 

  a2 b 2  
(d)    c  and (a  b  c)
0  c c  0
 

Solution: (a)Suppose charges on A, B and C are q a , qb and q c

qa qb
Respectively, so  A    2
 q a    4 a 2 ,  B     q b    4 b 2
4 a 4 b 2

qc
and  C     q c    4 c 2
4 c 2
Electrostatics 37

Potential at the surface of A

1  q a qb q c  1    4 a 2 ( )  4 b 2   4 c 2 
VA  (VA )surface  (VB )in  (VC )in          
4  0  a b c  4  0  a b c 


VA  a  b  c]
0

Potential at the surface of B

1  q a qb q c  1   4a2   4b 2   4c2    a 2 


VB  (VA )out  (VB )surface  (VC )in             b  c
40  b b c  40  b b c   0  b 

Electric Lines of Force.

(1) Definition: The electric field in a region is represented by continuous lines (also called lines of force).

Field line is an imaginary line along which a positive test charge will move if left free.

Electric lines of force due to an isolated positive charge, isolated negative charge and due to a pair of

charge are shown below

+ – + – +  +
N

(Radially outward) (Radially inward)

(2) Properties of electric lines of force

(i) Electric field lines come out of positive charge and go into the negative charge.

(ii) Tangent to the field line at any point gives the direction of the field at that point.

EA
EB
B

A
38 Electrostatics

(iii) Field lines never cross each other.

(iv) Field lines are always normal to conducting surface.

+ –
+ –
+ –
+ –
+ –
(A) + –
(B)

(v) Field lines do not exist inside a conductor.

(vi) The electric field lines never form closed loops. (While magnetic lines of forces form closed loop)

+ –

S N

(vii) The number of lines originating or terminating on a charge is proportional to the magnitude of charge.

In the following figure electric lines of force are originating from A and terminating at B hence QA is positive

while QB is negative, also number of electric lines at force linked with QA are more than those linked with QB

hence | Q A | | Q B |
A B QB
QA

(viii) Number of lines of force per unit area normal to the area at a point represents magnitude of intensity

(concept of electric flux i.e.,   EA )

(ix) If the lines of forces are equidistant and parallel straight lines the field is uniform and if either lines of

force are not equidistant or straight line or both the field will be non uniform, also the density of field lines is

proportional to the strength of the electric field. For example see the following figures.

X Y
X Y

(A) (B)
EX = EY EX > EY
Electrostatics 39

(3) Electrostatic shielding: Electrostatic shielding/screening is the phenomenon of protecting a certain

region of space from external electric field. Sensitive instruments and 


E

E
+ + ––
+
appliances are affected seriously with strong external electrostatic fields. + –

E=0
q =0
Their working suffers and they may start misbehaving under the effect of Hollow space
 –

+
E Conductor +
unwanted fields.  – –
+ + 
E E

The electrostatic shielding can be achieved by protecting and

enclosing the sensitive instruments inside a hollow conductor because inside hollow conductors, electric fields

is zero.

(i) It is for this reason that it is safer to sit in a car or a bus during lightening rather than to stand under a

tree or on the open ground.

(ii) A high voltage generator is usually enclosed in such a cage which is earthen. This would prevent the

electrostatic field of the generator from spreading out of the cage.

(iii) An earthed conductor also acts as a screen

against the electric field. When conductor is not C C

earthed field of the charged body C due to electrostatic

induction continues beyond AB. If AB is earthed,


– – – – – – – – – –
A – A
induced positive charge neutralizes and the field in the B B
+ + + + +

region beyond AB disappears.

Equipotential Surface or Lines.

If every point of a surface is at same potential, then it is said to be an equipotential surface

or

for a given charge distribution, locus of all points having same potential is called “equipotential surface”

regarding equipotential surface following points should keep in mind :

(1) The density of the equipotential lines gives an idea about the magnitude of electric field. Higher the

density larger the field strength.


40 Electrostatics

(2) The direction of electric field is perpendicular to the equipotential surfaces or lines.

(3) The equipotential surfaces produced by a point charge or a spherically charge distribution are a family

of concentric spheres.

V1 V2 V3 V4 V5
V = V2

V = V1

Equipotential
surface
V1 > V2 > V3 > V4 > V5

(4) For a uniform electric field, the equipotential surfaces are a family of plane perpendicular to the field

lines.

(5) A metallic surface of any shape is an equipotential surface e.g. When a charge is given to a metallic

surface, it distributes itself in a manner such that its every point comes at same potential even if the object is of

irregular shape and has sharp points on it.

+
+
+ + + + +
+ + +
+ +
+ + + +
O
+  + V = const.
+ V = const. + + +
+ + + + +
+ +
+ + +
Metallic charged Charged metallic body of irregular shape
sphere

If it is not so, that is say if the sharp points are at higher potential then due to potential difference between

these points connected through metallic portion, charge will flow from points of higher potential to points of

lower potential until the potential of all points become same.

(6) Equipotential surfaces can never cross each other

(7) Equipotential surface for pair of charges


Electrostatics 41

– + + +

Pair of two equal and opposite Pair of two equal and similar charges
charges

Concepts

 Unit field i.e. 1N/C is defined arbitrarily as corresponding to unit density of lines of force.

1
 Number of lines originating from a unit charge is
0

 It is a common misconception that the path traced by a positive test charge is a field line but actually the path traced by a
unit positive test charge represents a field full line only when it moves along a straight line.

 Both the equipotential surfaces and the lines of force can be used to depict electric field in a certain region of space. The
advantage of using equipotential surfaces over the lines of force is that they give a visual picture of both the magnitude and
direction of the electric field.

Examples based on electric lines of force

Example: 42 Three positive charges of equal value q are placed at the vertices of an equilateral triangle. The resulting
lines of force should be sketched as in

(a) (b) (c) (d) 

 
42 Electrostatics

Solution (c) Option (a) shows lines of force starting from one positive charge and terminating at another. Option (b) has
one line of force making closed loop. Option (d) shows all lines making closed loops. All these are not
correct. Hence option (c) is correct

 +q

+q +q
 

Example: 43 A metallic sphere is placed in a uniform electric field. The lines of force follow the path (s) shown in the
figure as
1 1
2 2
3 3
4 4

(a) 1 (b) 2 (c) 3 (d) 4

Solution: (d) The field is zero inside a conductor and hence lines of force cannot exist inside it. Also, due to induced
charges on its surface the field is distorted close to its surface and a line of force must deviate near the
surface outside the sphere.

Example: 44 The figure shows some of the electric field lines corresponding to an electric field. The figure suggests

A B C

(a) E A  E B  E C (b) E A  E B  E C (c) E A  E C  E B (d) E A  E C  E B

Solution: (c)

Example: 45 The lines of force of the electric field due to two charges q and Q are sketched in the figure. State if

(a) Q is positive and Q  q

(b) Q is negative and Q  q


Q q

(c) q is positive and Q  q


Electrostatics 43

(d) q is negative and Q  q

Solution: (c) q is +ve because lines of force emerge from it and Q  q because more lines emerge from q and less

lines terminate at Q.

Example: 46 The figure shows the lines of constant potential in a region in which an electric field is present. The
magnitude of electric field is maximum at

20 V

A
B

50 V
C
40 V 30 V

(a) A (b) B (c) C (d) Equal at A, B and C

Solution: (b) Since lines of force are denser at B hence electric field is maximum at B

Example: 47 Some equipotential surface are shown in the figure. The magnitude and direction of the electric field is

y 20 V 30 V 40 V

   
x
10 20 30
cm

 = 30o

(a) 100 V/m making angle 120o with the x-axis (b) 100 V/m making angle 60o with the x-axis

(c) 200 V/m making angle 120o with the x-axis (d) None of the above

Solution: (c)By using dV  E dr cos  suppose we consider line 1 and line 2 then

(30 – 20) = E cos 60o (20 – 10) × 10–2

y 
20 V 30 V
E 40 V

120o
30o
dr x
10 20 30 40

1 2
44 Electrostatics

So E  200 volt / m making in angle 120o with x-axis

Tricky example: 5

Which of the following maps cannot represent an electric field

(a) (b) (c) (d)

a b

d c
Solution: (a) If we consider a rectangular closed path, two parallel sides of it considering with lines of force

as shown, then we find that work done along the closed path abcd is
abE1 – cdE2  0. Hence the field cannot represent a conservative field.
But electric field is a conservative field. Hence a field represented by
these lines cannot be an electric field.

Tricky example: 6

A charge Q is fixed at a distance d in front of an infinite metal plate. The lines of force are
represented by
 +Q

(a) (b) (c) (d)

+Q +Q +Q +Q

Solution: (a) Metal plate acts as an equipotential surface, therefore the field lines should act normal to the
surface of the metal plate.
Electrostatics 45

Relation Between Electric Field and Potential.

In an electric field rate of change of potential with distance is known as

potential gradient. It is a vector quantity and it’s direction is opposite to that

of electric field. Potential gradient relates with electric field according to the
dV dr
following relation E   ; This relation gives another unit of electric field is +Q
A 
dr B E

volt
. In the above relation negative sign indicates that in the direction of
meter
electric field potential decreases.

In space around a charge distribution we can also write E  E x ˆi  E y ˆj  E z kˆ

dV dV dV
where E x   , Ey   and E z  
dx dy dz

dV
With the help of formula E   , potential difference between any two points in an electric field can be
dr
r2 r2
determined by knowing the boundary conditions dV   r1
E . dr   
r1
E . dr cos  .

For example: Suppose A, B and C are three points in an uniform electric field as shown in figure.

(i) Potential difference between point A and B is


B d
VB  V A    A
E  dr
A  B

Since displacement is in the direction of electric field, hence  = 0o C

B B
So, VB  V A   
A
E  dr cos 0   
A
E  dr   Ed

V V
In general we can say that in an uniform electric field E   or | E | 
d d
+ –
+ –

+ E –
+ – V
Another example E
+ – d
+ –
V1 = V V2 = 0
d
46 Electrostatics

(ii) Potential difference between points A and C is :


C
VC  V A    A
E dr cos    E( AC ) cos    E( AB ) = – Ed

Above relation proves that potential difference between A and B is equal to the potential difference

between A and C i.e. points B and C are at same potential.

Concept

V
 Negative of the slope of the V-r graph denotes intensity of electric field i.e. tan    E
r

+ –
E = V/d Example based on E = – dV/dr

Example: 48 The electric field, at a distance of 20 cm from the centre of a dielectric sphere of radius 10 cm is 100 V/m.

The ‘E’ at 3 cm distance from the centre of sphere is

(a) 100 V/m (b) 125 V/m (c) 120 V/m (d) Zero

k .q kqr
Solution: (c) For dielectric sphere i.e. for non-conducting sphere E out  2
and E in 
r R3

KQ 100  (0 . 2) 2  (3  10 2 ) 2
E out  100  KQ = 100  (0.2)2 so E in  = 120 V/m
(20  10  2 ) 2 (10  10  2 ) 3

Example: 49 In x-y co-ordinate system if potential at a point P(x, y) is given by V  axy ; where a is a constant, if r is

the distance of point P from origin then electric field at P is proportional to

(a) r (b) r–1 (c) r—2 (d) r2

dV dV dV
Solution: (a) By using E   Ex    ay , Ey    ax
dr dx dy

Electric field at point P E  E x2  E y2  a x 2  y 2  ar i.e., E  r

Example: 50 The electric potential V at any point x, y, z (all in metres) in space is given by V = 4x2 volt. The electric

field at the point (1m, 0, 2m) in volt/metre is

(a) 8 along negative X-axis (b) 8 along positive X-axis

(c) 16 along negative X-axis (d) 16 along positive Z-axis


Electrostatics 47

dV d
Solution: (a) By using E    E (4 x 2 )  8 x . Hence at point (1m, 0, 2m). E = – 8 volt/m i.e. 8 along – ve x-
dx dx
axis.

Example: 51 The electric potential V is given as a function of distance x (metre) by V = (5x2 + 10x – 9) volt. Value of

electric field at x = 1m is

(a) – 20 V/m (b) 6 V/m (c) 11 V/m (d) – 23 V/m

dV d
Solution: (a) By using E   ; E (5 x 2  10 x  9)  (10 x  10 ) ,
dx dx

at x = 1m E  20 V /m

Example: 52 A uniform electric field having a magnitude E0 and direction along the positive X-axis exists. If the electric

potential V, is zero at X = 0, then, its value at X = +x will be

(a) V(x)= +xE0 (b) V(x)= – xE0 (c) V(x)= x2E0 (d) V(x)= – x2E0

V (V  V1 ) {V (x )  0}
Solution: (b) By using E    2 ; E0    V(x) = – xE0
r (r2  r1 ) x 0

Example: 53 If the potential function is given by V = 4x + 3y, then the magnitude of electric field intensity at the point

(2, 1) will be

(a) 11 (b) 5 (c) 7 (d) 1

dV d
Solution: (b) By using i.e., E  E x2  E y2 ; E x    (4 x  3 y )  4
dx dx

dV d
and Ey     (4 x  3 y )  3
dy dy

 E  ( 4 ) 2  ( 3 ) 2  5 N /C

Tricky example: 7

The variation of potential with distance R from a fixed point is as shown below. The electric field
at R  5 m is

(a) 2.5 volt/m


48 Electrostatics
5

Potential in volts
4
(b) – 2.5 volt/m
3

2 2
(c) volt /m
5 1

0
2 1 2 3 4 5 6
(d)  volt /m Distance R in metres
5

Solution: (a) Intensity at 5 m is same as at any point between B and C because the slope of BC is same
throughout (i.e. electric field between B and C is uniform). Therefore electric field at R = 5m is
dV
equal to the slope of line BC hence by E  ;
dr
(0  5 ) V
E  2 .5
64 m
(5  0) V
Note :  At R = 1m , E  
(2  0)
 2 . 5
m

and at R  3 m potential is constant so E = 0.

Work Done in Displacing a Charge.

(1) Definition : If a charge Q displaced from one point to another point in electric field then work done in

this process is W  Q  V where V = Potential difference between the two position of charge Q.

( V  E . r  Er cos  where  is the angle between direction of electric field and direction of motion of

charge).

(2) Work done in terms of rectangular component of E and r : If charge Q is given a displacement
r  (r1ˆi  r2 ˆj  r3 kˆ ) in an electric field E  (E1ˆi  E 2 ˆj  E 3 kˆ ). The work done is W  Q( E . r )  Q(E1 r1  E 2 r2  E 3 r3 ) .

Conservation of Electric Field.

As electric field is conservation, work done and hence potential difference between two point is path

independent and depends only on the position of points between. Which the charge is moved.

I
A B WI = WII = WIII
II

III
Electrostatics 49

Concept

 No work is done in moving a charge on an equipotential surface.

I
II
Examples based on work done
WI = WII = WIII A B
III

Example: 54 A charge (– q) and another charge (+Q) are kept at two points A and B respectively. Keeping the charge

(+Q) fixed at B, the charge (– q) at A is moved to another point C such that ABC forms an equilateral

triangle of side l. The network done in moving the charge (– q) is

1 Qq 1 Qq 1
(a) (b) (c) Qql (d) Zero
4  0 l 4  0 l 2 4 0

kQ A –q
Solution: (d) Since V A  VC 
l
l l
so W  q (VC  V A )  0

B C
+Q l

Example: 55 The work done in bringing a 20 coulomb charge from point A to point B for distance 0.2 m is 2 Joule. The

potential difference between the two points will be (in volt)

(a) 0.2 (b) 8 (c) 0.1 (d) 0.4

Solution: (c) W  Q.V  2 = 20  V  V = 0.1 volt

Example: 56 A charge +q is revolving around a stationary +Q in a circle of radius r. If the force between charges is F

then the work done of this motion will be

F
(a) F × r (b) F  2r (c) (d) 0
2r
+Q
Solution: (d) Since +q charge is moving on an equipotential surface so work done is zero. +q
50 Electrostatics

Example: 57 Four equal charge Q are placed at the four corners of a body of side ‘a’ each. Work done in removing a

charge – Q from its centre to infinity is

2 Q2 2 Q2 Q2
(a) 0 (b) (c) (d)
4  0 a  0 a 2 0 a

Solution: (c) We know that work done in moving a charge is W = QV Q Q


a
A B
Here W  Q (V0  V )  V  0  W = Q × V0

a O a
1 Q 4 2Q 2Q –Q
Also V0  4  .  
4  0 a / 2 4  0 a  0 a
D C
a
2Q 2 Q Q
So, W 
 0 a

Example: 58 Two point charge 100 C and 5 C are placed at point A and B respectively with AB = 40 cm. The work
done by external force in displacing the charge 5 C from B to C, where BC = 30 cm, angle
 1
ABC  and  9  10 9 Nm 2 /C 2
2 4 0
81 9 9
(a) 9 J (b) J (c) J (d)  J
20 25 4
Solution: (d) Potential at B due to +100 C charge is A
6 + 100 C
100  10 9
VB  9  10 9  2
  10 6 volt
40  10 4
Potential at C due to +100 C charge is
40 cm

50 cm
6
100  10 9
VC  9  10 9    10 6 volt
50  10  2 5 /2 + 50 C
Hence work done in moving charge +5C from B to C B 30 cm C
6
W  5  10 (VC  VB )
9 9  9
W  5  10 6   10 6   10  6    J
5 4  4
Example: 59 There is an electric field E in x-direction. If the work done in moving a charge 0.2 C through a
distance of 2 metres along a line making an angle 60o with the x -axis is 4J , what is the value of E

(a) 4 N/C (b) 8 N/C (c) 3 N/C (d) 20 N/C

Solution: (d) By using W  q  V and V  Er cos  A


0.2C
So, W  qE r cos  2m

W  4 j  0 . 2  E  2  cos 60 60o
O x
Electrostatics 51

 E = 20 N/C

Example: 60 An electric charge of 20 C is situated at the origin of X-Y co-ordinate system. The potential difference
between the points. (5a, 0) and (– 3a, 4a) will be

a
(a) a (b) 2a (c) Zero (d)
2
kQ kQ B (–3a, 4a)
Solution: (c) VA  and VB 
5a 5a
5a
 V A  VB  0
A (5a, 0)
Q 5a

Example: 61 Two identical thin rings each of radius R, are coaxially placed a distance R apart. If Q1 and Q2 are
respectively the charges uniformly spread on the two rings, the work done in moving a charge q from the
centre of one ring to that of the other is
Q 
q  1  ( 2  1)
q(Q1  Q 2 )( 2  1) q(Q1  Q 2 ) 2 Q2 
(a) Zero (b) (c) (d) 
4  0 R 2 4 0 R 4  0 R 2

Q1 Q2
Solution: (b) Potential at the centre of first ring VA  
4  0 R 4  R 2  R 2 Q1 Q2
0

Q2 Q1 R R
Potential at the centre of second ring VB  
4  0 R 4  R 2  R 2 1 2
0
A B

( 2  1)(Q1  Q 2 )
Potential difference between the two centres V A  VB 
4 0 R 2 R

q( 2  1)(Q1  Q 2 )
 Work done W 
4  0 R 2

Tricky example: 8

A point charge q moves from point A to point D along the path ABCD in a uniform electric field. If

the co-ordinates of the points A, B, C and D are (a, b, 0), (2a, 0, 0), (a, – b, 0) and (0, 0, 0) then the

work done by the electric field in this process will be

(a) – qEa
52 Electrostatics

(b) Zero Y

A (a,b,0) E
(c) 2E (a + b)q b a2 + b2
a O 
X
qEa D
 
B
(d) b
2b a2 + b2 a2 + b2

Solution: (a) As electric field is a conservative field

Hence the work done does not depend on path


Y

E
A

X
D B
 W ABCD  W AOD  W AO  WOD

= Fb cos 90o + Fa cos 180o = 0 + qEa (– 1)= – qEa C

Equilibrium of Charge.

(1) Definition : A charge is said to be in equilibrium, if net force acting on it is zero. A system of charges is
said to be in equilibrium if each charge is separately in equilibrium.

(2) Type of equilibrium : Equilibrium can be divided in following type:

(i) Stable equilibrium : After displacing a charged particle from it's equilibrium position, if it returns back
d 2U
then it is said to be in stable equilibrium. If U is the potential energy then in case of stable equilibrium is
dx 2
positive i.e., U is minimum.

(ii) Unstable equilibrium : After displacing a charged particle from it's equilibrium position, if it never
d 2U
returns back then it is said to be in unstable equilibrium and in unstable equilibrium is negative i.e., U is
dx 2
maximum.

(iii) Neutral equilibrium : After displacing a charged particle from it's equilibrium position if it neither comes

back, nor moves away but remains in the position in which it was kept it is said to be in neutral equilibrium and
d 2U
in neutral equilibrium is zero i.e., U is constant
dx 2

(3) Guidelines to check the equilibrium


Electrostatics 53

(i) Identify the charge for which equilibrium is to be analysed.

(ii) Check, how many forces acting on that particular charge.

(iii) There should be atleast two forces acts oppositely on that charge.

(iv) If magnitude of these forces are equal then charge is said to be in equilibrium then identify the nature of

equilibrium.

(v) If all the charges of system are in equilibrium then system is said to be in equilibrium

(4) Different cases of equilibrium of charge

Case – 1 : Suppose three similar charge Q 1 , q Case – 2 : Two similar charge Q1 and Q 2 are

and Q 2 are placed along a straight line as shown placed along a straight line at a distance x from

below each other and a third dissimilar charge q is

placed in between them as shown below

A B
F2 O F1 A B
Q1 Q2 F1 O F2
q Q1 Q2
x1 x2 q
x1 x2
x
x

Charge q will be in equilibrium if | F1 |  | F2 |


Charge q will be in equilibrium if | F1 |  | F2 | 2
Q1  x1 
i.e.,   .
Q 2  x 2 
2
Q1  x1 
i.e.,   ; This is the condition of
Q 2  x 2 
Note :  Same short trick can be used here to
equilibrium of charge q. After following the
find the position of charge q as we discussed in
guidelines we can say that charge q is in stable
Case–1 i.e.,
equilibrium and this system is not in equilibrium
x x
x x1  and x 2 
Note : x1  1  Q 2 /Q1 1  Q 1 /Q 2
1  Q 2 /Q1
 It is very important to know that magnitude of
x
and x2  charge q can be determined if one of the extreme
1  Q 1 /Q 2
charge (either Q1 or Q 2 ) is in equilibrium i.e. if
e.g. if two charges +4C and +16 C are
54 Electrostatics

2
separated by a distance of 30 cm from each other x 
Q2 is in equilibrium then | q |  Q 1  2  and if
then for equilibrium a third charge should be  x 
2
x 
placed between them at a distance Q1 is in equilibrium then | q |  Q 2  1  (It should
 x 
30
x1   10 cm or x 2  20 cm be remember that sign of q is opposite to that of
1  16 / 4
Q 1 (or Q 2 ) )

Case – 3 : Two dissimilar charge Q1 and Q 2 are placed along a straight line at a distance x from
each other, a third charge q should be placed out side the line joining Q1 and Q 2 for it to
experience zero net force.
Q1 – Q2 q

x d (Let |Q2| < |Q1|)

Short Trick :

For it's equilibrium. Charge q lies on the side of chare which is smallest in magnitude and
x
d 
Q 1 /Q 2  1

(5) Equilibrium of suspended charge in an electric field

(i) Freely suspended charged particle : To suspend a charged a particle freely in air under the influence of

electric field it’s downward weight should be balanced by upward electric force for example if a positive charge

is suspended freely in an electric field as shown then



E
– – – – – – – – –
or F = QE or F = QE
F = QE V d
+Q
mg
+Q + + + + + + + + +
+Q
mg mgd
Q  ne  
E V
 mg
mg E
Electrostatics 55

mg
In equilibrium QE  mg  E 
Q

Note :  In the above case if direction of electric field is suddenly reversed in any figure then acceleration
of charge particle at that instant will be a = 2g.

(ii) Charged particle suspended by a massless insulated string (like simple pendulum) : Consider a

charged particle (like Bob) of mass m, having charge Q is suspended in an electric field as shown under the

influence of electric field. It turned through an angle (say ) and comes in equilibrium.

So, in the position of equilibrium (O position)



E
T sin   QE ….(i)
 l T cos 
T 
T cos   mg ….(ii) T sin  QE
O

By squaring and adding equation (i) and (ii) T  QE 2  mg 2 O


mg

QE QE
Dividing equation (i) by (ii) tan    θ  tan 1
mg mg

(iii) Equilibrium of suspended point charge system : Suppose two small balls having charge +Q on each

are suspended by two strings of equal length l. Then for equilibrium position as shown in figure.

 
T sin   Fe ….(i) l l
T cos 
T

T cos   mg ….(ii)
+Q Fe
T sin  +Q
x
T 2  Fe   mg 
2 2
mg

Fe 1 Q2 x
and tan   ; here Fe  and  l sin 
mg 4  0 x 2 2

(iv) Equilibrium of suspended point charge system in a liquid : In the previous discussion if point charge

system is taken into a liquid of density  such that  remain same then

In equilibrium Fe'  T ' sin  and (mg  Vg)  T ' cos 


56 Electrostatics

Fe' Q2
 tan   
(mg  Vg) 4  0 K (mg  Vg)x 2  
l l
Fe Q 2 T cos 
When this system was in air tan    T 
2
mg 4  0 mgx
+Q F e
T sin  +Q
x
(mg –
Vg)
1 1 m 1
 So equating these two gives us  K 
m k (m  V ) m  V  V 
1   
 m 

1
If  is the density of material of ball then K 
 ρ
 1  
 σ
QE
Q
mg Examples based on equilibrium of charge

Example: 62 A charge q is placed at the centre of the line joining two equal charges Q. The system of the three

charges will be in equilibrium. If q is equal to

Q Q Q Q
(a)  (b)  (c)  (d) 
2 4 4 2
2
x /2
Solution: (b)By using Tricky formula q  Q 
 x 

Q Q
 q since q should be negative so q   .
4 4
Electrostatics 57

Example: 63 Two point charges +4q and +q are placed at a distance L apart. A third charge Q is so placed that all the

three charges are in equilibrium. Then location and magnitude of third charge will be

L 4q
(a) At a distance from +4q charge,
3 9
L 4q
(b) At a distance from +4q charge, 
3 9
2L 4q
(c) At a distance from +4q charge, 
3 9
2L 4q
(d) At a distance from +q charge, 
3 9

Solution: (c) Let third charge be placed at a distance x 1 from +4q charge as shown

L 2L L Q
Now x 1    x2  +4q +q
q 3 3
1 x1 x2
4q
L
2
L/3 4q 4q
For equilibrium of q, Q  4 q     Q .
 L  9 9

Example: 64 A drop of 10 6 kg water carries 10 6 C charge. What electric field should be applied to balance it’s weight

(assume g = 10 m/sec2)

(a) 10 V / m , Upward (b) 10 V / m , Downward (c) 0.1 V/m Downward (d) 0 . 1V / m , Upward

Solution: (a) In equilibrium QE = mg

mg 10 6  10
E = = 10 V/m; Since charge is positive so electric field will be upward.
Q 10 6

Example: 65 A charged water drop of radii 0.1 m is under equilibrium in some electric field. The charge on the drop is

equivalent to electronic charge. The intensity of electric field is

(a) 1 . 61 N / C (b) 25 . 2 N / C (c) 262 N / C (d) 1610 N / C

4 3 
 r  . g 6 3 3
Solution: (c) In equilibrium QE = mg ; mg 3   4  (3 .14 ) (0 .1  10 )  10  10 = 262 N/C
E 
Q Q 3 1 .6  10 19

Example: 66 The bob of a pendulum of mass 8 g carries an electric charge of 39 . 2  10 10 coulomb in an electric field

of 20  10 3 volt / meter and it is at rest. The angle made by the pendulum with the vertical will be

(a) 27o (b) 45o (c) 87o (d) 127o


58 Electrostatics

Solution: (b) T sin   qE , T cos   mg




E
qE
 tan  
mg
T cos 
39 . 2  10 10  20  10 3 T 
tan   6
1
8  10  9 .8 T sin  qE

   45 o mg

Example: 67 Two small spherical balls each carrying a charge Q = 10 C (10 micro-coulomb) are suspended by two

insulating threads of equal lengths 1 m each, from a point fixed in the ceiling. It is found that in equilibrium

threads are separated by an angle 60o between them, as shown in the figure. What is the tension in the
1
threads. (Given :  9  10 9 Nm / C 2 )
4 0 
(a) 18 N
60o
(b) 1.8 N

(c) 0.18 N Q Q

(d) None of these

Solution: (b) From the geometry of figure

30o 30
o

r = 1m
1m 1m
o
In the condition of equilibrium T sin 30  Fe T cos 30o
T 30o

1 (10  10 6 ) 2 +10 C Fe
T  9  10 9 . T sin 30o +10 C
r
2 12 mg

 T= 1.8 N

Example: 68 Two similar balloons filled with helium gas are tied to L m long strings. A body of mass m is tied to

another ends of the strings. The balloons float on air at distance r. If the amount of charge on the balloons

is same then the magnitude of charge on each balloon will be


1/2
 mgr 2 
(a)  tan  
 2k 
r
1/2 Q
 2k  Q
(b)  2
tan  
 mgr 
1/2 L L
 mgr   
(c)  cot  
 2k 
m
Electrostatics 59

1/2
 2k 
(d)  tan  
 mgr 
Solution: (a) In equilibrium
R R

2 R  mg …. (i) Fe  T sin  …. (ii) R  T cos  …. (iii) r


Fe
From equation (i) and (iii) T sin 
T
2 T cos   mg …. (iv) T cos  L  
L

Dividing equation (ii) by equation (iv)


m
2
Q mg
k 2 1/2
1 F 1  mgr 2 
tan   e  tan   r     tan  
2 mg 2 mg  2k 
 

Time Period of Oscillation of a Charged Body.

(1) Simple pendulum based : If a simple pendulum having length l and mass of bob m oscillates about it's
l
mean position than it's time period of oscillation T  2 π
g
l

Case – 1 : If some charge say +Q is given to bob and Case – 2 : If electric field is applied in the downward

an electric field E is applied in the direction as shown in direction then.

figure then equilibrium position of charged bob (point Effective acceleration 


T
l
charge) changes from O to O. 
E
 g'  g  QE / m
E
d

So new time period
mg + QE
l
O
l
QE T2  2 π
g  QE/m 

mg T2 < T
O mg 
60 Electrostatics

Case – 3 : In case 2 if electric field is applied in upward


 QE
direction then, effective acceleration.
l 
E
On displacing the bob from it’s equilibrium position 0. It g'  g  QE / m

will oscillate under the effective acceleration g, where


So new time period
mg
mg'  mg 2  QE 2 l
T3  2 π
g  QE/m 
 g'  g 2  QE / m 2
T3 > T
l
Hence the new time period is T1  2
g'

Case – 4 : In the case 3,


l
T1  2 π 1
g 2
 QE/m 2 
2 if T3 
T
2
i.e., 2
l
g  QE / m 
Since g' >g, hence T1 < T
1 l
 2  QE = 3 mg
2 g
i.e. time period of pendulum will decrease.
i.e., effective vertical force (gravity + electric) on the bob
= mg – 3 mg = – 2 mg, hence the equilibrium position
O of the bob will be above the point of suspension and
bob will oscillate under on effective acceleration 2g
directed upward.

l
Hence new time period T4  2 π , T4 < T
2g

(2) Charged circular ring : A thin stationary ring of radius R has a positive charge +Q unit. If a negative
charge – q (mass m) is placed at a small distance x from the centre. Then motion of the particle will be simple
harmonic motion.
1 Qx
Electric field at the location of – q charge E  .
4  0 3
x 2
 R2  2
+ + + +Q
1 Qx + +
Since x<< R, So x 2 neglected hence E  . +
4  0 R3 + R + –q
+ +
1 Qx + O x
Force experienced by charge – q is F  q . + +
4  0 R3 +
+
+
+ +
 F   x hence motion is simple harmonic +
Electrostatics 61

4 πε 0 mR 3
Having time period T  2 π
Qq
(3) Spring mass system : A block of mass m containing a negative charge – Q is placed on a frictionless
horizontal table and is connected to a wall through an unstretched spring of spring
constant k as shown. If electric field E applied as shown in figure the block
experiences an electric force, hence spring compress and block comes in new E
k m, – Q
position. This is called the equilibrium position of block under the influence of
electric field. If block compressed further or stretched, it execute oscillation having
m QE
time period T  2 π . Maximum compression in the spring due to electric field =
k k
Neutral Point.

A neutral point is a point where resultant electrical field is zero. It is obtained where two electrical field are
equal and opposite. Thus neutral points can be obtained only at those points where the resultant field is
subtractive. Thus it can be obtained.
(1) At an internal point along the line joining two like charges (2) At an external point along the line joining two like charges
(Due to a system of two like point charge) : Suppose two like (Due to a system of two unlike point charge) : Suppose two
charges. Q1 and Q2 are separated by a distance x from each unlike charge Q1 and Q2 separated by a distance x from each
other along a line as shown in following figure. other. Q1
N N
Q1 Q2 Q2
x1 x2 l
x x

If N is the neutral point at a distance x 1 from Q1 and at a


distance x 2  x  x1  from Q 2 then – Here neutral point lies outside the line joining two unlike charges
and also it lies nearer to charge which is smaller in magnitude.
At N | E.F. due to Q 1 |  | E.F. due to Q 2 |
2
If Q 1  Q 2 then neutral point will be obtained on the side of
1 Q 1 Q Q x 
i.e., . 1 = . 2  1   1  Q1 , suppose it is at a distance l from Q1
4  0 x12 4  0 x 22 Q2  x 2 
2
x x kQ1 kQ 2 Q1  l 
Short rick : x1  and x 2  Hence at neutral point ;    
1  Q 2 /Q1 1  Q 1 /Q 2 l2 x  l 2 Q2  x  l 
x
Note :  In the above formula if Q1  Q2 , neutral point lies at
Short rick : l 
Q 2 /Q 1 1 
the centre so remember that resultant field at the
midpoint of two equal and like charges is zero.
Note :  In the above discussion if | Q1 | | Q 2 | neutral

point will be at infinity.


62 Electrostatics

Zero Potential Due to a System of Two Point Charge.

If both charges are like then resultant potential is not zero at any finite point because potentials due to like
charges will have same sign and can therefore never add up to zero. Such a point can be therefore obtained
only at infinity.

If the charges are unequal and unlike then all such points where resultant potential is zero lies on a
closed curve, but we are interested only in those points where potential is zero along the line joining the two
charges.

Two such points exist, one lies inside and one lies outside the charges on the line joining the charges.
Both the above points lie nearer the smaller charge, as potential created by the charge larger in magnitude will
become equal to the potential created by smaller charge at the desired point at larger distance from it.

I. For internal point : II. For External point :

P Q1 Q2
Q1 P
Q2
x1 x2 x1
x x

(It is assumed that Q1  Q2 ). Q1 Q2 x


  x1 
x1 x  x1  Q 2 /Q 1  1
Q1 Q2 x
  x1 
x1 x  x1  Q 2 /Q 1  1 

N
Examples based on oscillation of charge and neutral point

Example: 69 Two similar charges of + Q as shown in figure are placed at points A and DB. – q charge is placed at

point C midway between A and B. – q charge will oscillate if

(a) It is moved towards A


A C– q
(b) It is moved towards B +Q +Q B

(c) It is moved along CD

(d) Distance between A and B is reduced

Solution: (c) When – q charge displaced along CD, a restoring force act on it which causes oscillation.
Electrostatics 63

Example: 70 Two point charges (+Q) and (– 2Q) are fixed on the X-axis at positions a and 2a from origin respectively.

At what position on the axis, the resultant electric field is zero

3a
(a) Only x  2 a (b) Only x   2 a (c) Both x   2 a (d) x  only
2
k .Q k (2 Q)
Solution: (b) Let the electric field is zero at a point P distance d from the charge +Q so at P. 2
 0
d (a  d ) 2
1 2 a
   d
d2 (a  d ) 2 ( 2  1) P +Q – 2Q

x a
2a
d

Since d > a i.e. point P must lies on negative x-axis as shown at a distance x from origin hence
a
x  d a   a  2 a . Actually P lies on negative x-axis so x   2 a .
2 1

Example: 71 Two charges 9e and 3e are placed at a distance r. The distance of the point where the electric field
intensity will be zero is
r r
(a) from 9e charge (b) from 9e charge
 3 1  1 1 3
r r
(c) from 3e charge (d) from 3e charge
1  3  1 1 3

Solution: (b) Suppose neutral point is obtained at a distance x 1 from charge 9e and x 2 from charge 3e

N
9e 3e
x r r
By using x 1  =  x1 x2
Q2 3e  1 
1 1  1   r
Q1 9e  3

Example: 72 Two point charges – Q and 2Q are separated by a distance R, neutral point will be obtained at
R
(a) A distance of from – Q charge and lies between the charges.
( 2  1)
R
(b) A distance of from – Q charge on the left side of it
( 2  1)
R
(c) A distance of from 2Q charge on the right side of it
( 2  1)

(d) A point on the line which passes perpendicularly through the centre of the line joining – Q and 2Q
charge.
64 Electrostatics

Solution: (b) As already we discussed neutral point will be obtained on the side of charge which is smaller in
magnitude i.e. it will obtained on the left side of – Q charge and at a distance.
R R
l l
2Q ( 2  1)
1
Q

Example: 73 A charge of + 4C is kept at a distance of 50 cm from a charge of – 6C. Find the two points where the
potential is zero

(a) Internal point lies at a distance of 20 cm from 4C charge and external point lies at a distance of 100
cm from 4C charge.

(b) Internal point lies at a distance of 30 cm from 4C charge and external point lies at a distance of 100
cm from 4C charge

(c) Potential is zero only at 20 cm from 4C charge between the two charges

(d) Potential is zero only at 20 cm from – 6C charge between the two charges
x 50
Solution: (a) For internal point X, x1    20 cm and for external point Y,
 Q2  6
  1  1
4
 Q1 
x 50 Y X
x1    100 cm 4C – 6C
 Q2  6
  1  1 20cm
4
 Q1  100cm 50cm

Tricky example: 9

Two equal negative charges – q are fixed at points (0, a) and (0, – a) on the y-axis. A positive
charge Q is released from rest at the point (2a, 0) on the x-axis. The charge Q will
A
–q

(a) Execute simple harmonic motion about the origin a


(b) Move to the origin and remains at rest O
Q
x C
(c) Move to infinity a 2a

(d) Execute oscillatory but not simple harmonic motion.


–q
B
Solution: (d) By symmetry of problem the components of force on Q due to charges at A and B along y-axis will

cancel each other while along x-axis will add up and will be along CO. Under the action of this force

charge Q will move towards O. If at any time charge Q is at a distance x from O.


Electrostatics 65

1 qQ x
F  2 F cos   2 
4  0 (a 2  x 2 ) (a 2  x 2 )1 2

1 2 qQx
i.e., F   .

4  0 a  x 2 3
2
 2

As the restoring force F is not linear, motion will be oscillatory

(with amplitude 2a) but not simple harmonic.

Electric Potential Energy.


(1) Potential energy of a charge : Work done in bringing the given charge from infinity to a point in the

electric field is known as potential energy of the charge. Potential can also be written as potential energy per
W U
unit charge. i.e. V  .
Q Q

(2) Potential energy of a system of two charges : Since work done in bringing charge Q2 from  to point B
1 Q1
is W  Q2 VB , where VB is potential of point B due to charge Q1 i.e. V B 
4  0 r

1 QQ Q1 Q2
So, W  U2  . 1 2
4  0 r
A B
r

1 QQ
This is the potential energy of charge Q2, similarly potential energy of charge Q1 will be U1  . 1 2
4  0 r
Q1Q2
Hence potential energy of Q1 = Potential energy of Q2 = potential energy of system U  k (in C.G.S.
r
Q1Q2
U )
r
Note :  Electric potential energy is a scalar quantity so in the above formula take sign of Q1 and Q2.
(3) Potential energy of a system of n charges : In a system of n charges electric potential energy is

calculated for each pair and then all energies so obtained are added algebraically. i.e.
66 Electrostatics

1  Q1Q2 Q2 Q3 
U    .........  and in case of continuous distribution of charge. As dU  dQ .V 
4  0  r12 r23 
Q1
U  V dQ
e.g. Electric potential energy for a system of three charges r12 r31

1  Q1Q2 Q2 Q3 Q3 Q1 
Potential energy      Q2 r23 Q3
4  0  r12 r23 r31 

1  Q1Q2 Q3 Q1 
While potential energy of any of the charge say Q1 is   
4  0  r12 r31 

n (n  1)
Note :  For the expression of total potential energy of a system of n charges consider number of
2
pair of charges.

(4) Electron volt (eV) : It is the smallest practical unit of energy used in atomic and nuclear physics. As

electron volt is defined as “the energy acquired by a particle having one quantum of charge 1e when
1J
accelerated by 1volt” i.e. 1eV  1 . 6  10 19 C   1 .6  10 19 J = 1.6  10–12 erg
C

Energy acquired by a charged particle in eV when it is accelerated by V volt is E = (charge in quanta) × (p.d. in

volt)

Commonly asked examples :

S.No. Charge Accelerated by Gain in K.E.

p.d.

(i) Proton 5  104 V K = e  5  104 V = 5  104 eV = 8  10–15 J

(ii) Electron 100 V K = e  100 V = 100 eV = 1.6  10–17 J

(iii) Proton 1V K = e  1 V = 1 eV = 1.6  10–19 J

(iv) 0.5 C 2000 V K = 0.5  2000 = 1000 J

(v) -particle 106 V K = (2e)  106 V = 2 MeV


Electrostatics 67

(5) Electric potential energy of a uniformly charged sphere : Consider a uniformly charged sphere of
radius R having a total charge Q. The electric potential energy of this sphere is equal to the work done in
bringing the charges from infinity to assemble the sphere.
3Q 2
U
20  0 R

(6) Electric potential energy of a uniformly charged thin spherical shell :


Q2
U
8 0 R

(7) Energy density : The energy stored per unit volume around a point in an electric field is given by
U 1 1
Ue    0 E 2 . If in place of vacuum some medium is present then U e   0  r E 2
Volume 2 2

Concepts

 Electric potential energy is not localised but is distributed all over the field

 If a charge moves from one position to another position in an electric field so it’s potential energy change and work done in
this changing is W  U f  U i

 If two similar charge comes closer potential energy of system increases while if two dissimilar charge comes closer potential
energy of system decreases.

Q q

U=0
Examples based on electric potential energy

Example: 74 If the distance of separation between two charges is increased, the electrical potential energy of the

system

(a) May increases or decrease (b) Decreases

(c) Increase (d) Remain the same

Q1 Q 2
Solution: (a)Since we know potential energy U  k .
r

As r increases, U decreases in magnitude. However depending upon the fact whether both charges are

similar or disimilar, U may increase or decrease.


68 Electrostatics

Example: 75 Three particles, each having a charge of 10C are placed at the corners of an equilateral triangle of side
1
10cm. The electrostatic potential energy of the system is (Given  9  10 9 N  m 2 /C 2 )
4 0

(a) Zero (b) Infinite (c) 27 J (d) 100 J

Solution: (c)Potential energy of the system, 10C

 (10  10 6 ) 2 
U  9  10 9   3 = 27 J
 0 .1  10 cm 10 cm

10C 10C
10 cm

Example: 76 Three charges Q, +q and +q are placed at the vertices of a right-angled isosceles triangle as shown. The

net electrostatic energy of the configuration is zero if Q is equal to

Q
q
(a)
1 2

 2q
(b)
1 2
+q +q
(c) – 2 q a

(d) +q

Qq k .q 2 Qq  2q
Solution: (b)Potential energy of the configuration U  k .   k. 0  Q
a a a 2 2 1

Example: 77 A charge 10 e.s.u. is placed at a distance of 2 cm from a charge 40 e.s.u. and 4 cm from another charge

of 20 e.s.u. The potential energy of the charge 10 e.s.u. is (in ergs)

(a) 87.5 (b) 112.5 (c) 150 (d) 250


10 esu
Solution: (d)Potential energy of 10 e.s.u. charge is
2 cm
10  40 10  20 4 cm
U   250 erg.
2 4
40 esu

20 esu
Electrostatics 69

Example: 78 In figure are shown charges q1 = + 2 × 10–8 C and q2 = – 0.4 × 10–8 C. A charge q3 = 0.2 × 10–8 C in

moved along the arc of a circle from C to D. The potential energy of q3


q3
C

80 cm
(a) Will increase approximately by 76%
A B
(b) Will decreases approximately by 76% D
60 cm
q1 q2
(c) Will remain same

(d) Will increases approximately by 12%

q q q q 
Solution: (b)Initial potential energy of q3 U i   1 3  2 3   9  10 9 q3
 0 .8 1 
C
q q q q 
Final potential energy of q3 U f   1 3  2 3   9  10 9
 0 .8 0 .2  80 cm

Change in potential energy = Uf – Ui


A B
U f  Ui
Now percentage change in potential energy   100 D
60 cm
ui q1 q2
80 cm

 1 
q2q3   1   100
 0 .2 
 On putting the values ~–  76 %
q q 
q3  1  2 
 0 . 8 1 

Tricky example: 10

Three charged particles are initially in position 1. They are free to move and they come in position 2

after some time. Let U1 and U2 be the electrostatics potential energies in position 1 and 2. Then

(a) U1 > U2 (b) U2 > U1

(c) U1 = U2 (d) U2  U1

Solution: (a) Particles move in a direction where potential energy of the system is decreased.
70 Electrostatics

Motion of Charged Particle in an Electric Field.

(1) When charged particle initially at rest is placed in the uniform field :

Let a charge particle of mass m and charge Q be initially at rest in an electric field of strength E


 E
E
+Q F = QE

A B

F=QE –Q S

Fig. (A) Fig. (B)

(i) Force and acceleration : The force experienced by the charged particle is F  QE . Positive charge

experiences force in the direction of electric field while negative charge experiences force in the direction

opposite to the field. [Fig. (A)]

F QE
Acceleration produced by this force is a  
m m

Since the field E in constant the acceleration is constant, thus motion of the particle is uniformly

accelerated.

(ii) Velocity: Suppose at point A particle is at rest and in time t, it reaches the point B [Fig. (B)]

V = Potential difference between A and B; S = Separation between A and B

E QEt
(a) By using v  u  at , v0Q t,  v
m m

QE 2QV  V 2 QV
(b) By using v 2  u 2  2 as , v 2  0  2  s v2   E    v 
m m  s m

QEt 2 QV
(iii) Momentum : Momentum p = mv, p  m   QEt or p  m   2mQV
m m
Electrostatics 71

1 1 (QEt ) 2 Q 2 E 2 t 2
(iv) Kinetic energy: Kinetic energy gained by the particle in time t is K  mv 2  m 
2 2 m 2m

1 2 QV
or K m  QV
2 m

(2) When a charged particle enters with an initial velocity at right angle to the uniform field :

When charged particle enters perpendicularly in an electric field, it describe a parabolic path as shown

(i) Equation of trajectory : Throughout the motion particle has uniform velocity along x-axis and horizontal

displacement (x) is given by the equation x = ut

Since the motion of the particle is accelerated along y–axis, we will use equation of motion for uniform
1 2
acceleration to determine displacement y. From S  ut  at
2
Y
E
1
We have u  0 (along y-axis) so y  at 2
2
P(x, y)
i.e., displacement along y-axis will increase rapidly with time (since y  t 2 )
u X
x
From displacement along x-axis t 
u
2
1  QE   x  2
So y      ; this is the equation of parabola which shows y  x
2  m  u 

QEt
(ii) Velocity at any instant : At any instant t, v x  u and v y 
m v

Q 2 E2t2 vy
So v | v |  v x2  v y2  u 2 
m2

vy vx
QEt
If  is the angle made by v with x-axis than tan    .
vx mu

Concepts

 An electric field is completely characterized by two physical quantities Potential and Intensity. Force characteristic of the
field is intensity and work characteristic of the field is potential.

 If a charge particle (say positive) is left free in an electric field, it experiences a force (F  QE ) in the direction of electric
72 Electrostatics

field and moves in the direction of electric field (which is desired by electric field), so its kinetic energy increases, potential
energy decreases, then work is done by the electric field and it is negative.


E
Q
Examples based on motion of charge

Example: 79 An electron (mass = 9 . 1  10 31 kg and charge = 1 . 6  10 19 coul .) is sent in an electric field of intensity

1  10 6 V / m . How long would it take for the electron, starting from rest, to attain one–tenth the velocity of

light

(a) 1 . 7  10 12 sec (b) 1 . 7  10 6 sec (c) 1 . 7  10 8 sec (d) 1 . 7  10 10 sec

QEt 1 (1 . 6  10 19 )  10 6  t
Solution: (b)By using v    3  10 8   t  1 . 7  10 10 sec .
m 10 9 . 1  10  31

Example: 80 Two protons are placed 10 10 m apart. If they are repelled, what will be the kinetic energy of each proton

at very large distance

(a) 23  10 19 J (b) 11 . 5  10 19 J (c) 2 . 56  10 19 J (d) 2 . 56  10 28 J

Solution: (d)Potential energy of the system when protons are separated by a distance of 10 10 m is

9  10 9  (1 . 6  10 19 ) 2
U 10
 23  10 19 J P+ p+
10

According to law of conservation of energy at very larger distance, this energy is equally distributed in

both the protons as their kinetic energy hence K.E. of each proton will be 11 . 5  10 19 J .

Example: 81 A particle A has a charge +q and particle B has charge +4q with each of them having the same mass m.

When allowed to fall from rest through the same electrical potential difference, the ratio of their speeds
vA
will becomes
vB

(a) 2 : 1 (b) 1 : 2 (c) 1 : 4 (d) 4 : 1

1 v QA q 1
Solution: (b)We know that kinetic energy K  mv 2  QV . Since, m and V are same so, v 2  Q  A    .
2 vB QB 4q 2

Example: 82 How much kinetic energy will be gained by an   particle in going from a point at 70 V to another point at

50 V

(a) 40 eV (b) 40 keV (c) 40 MeV (d) 0 eV


Electrostatics 73

Solution: (a)Kinetic energy K  Q V  K  (2 e ) (70  50 ) V  40 eV

Example: 83 A particle of mass 2g and charge 1  C is held at a distance of 1 metre from a fixed charge of 1mC . If the

particle is released it will be repelled. The speed of the particle when it is at a distance of 10 metres from

the fixed charge is

(a) 100 m/s (b) 90 m/s (c) 60 m/s (d) 45 m/s

Solution: (b)According to conservation of energy

1 mC 1 C

A
Moving charge B
Fixed charge 1m

10m

Energy of moving charge at A  Energy of moving charge at B

10 3  10 6 10 3  10 6 1
9  10 9   9  10 9    (2  10  3 ) v 2
1 10 2

 v 2  8100  v  90 m/sec

Tricky example: 11

A mass of 1g carrying charge q falls through a potential difference V. The kinetic energy acquired

by it is E. When a mass of 2g carrying the charge q falls through a potential difference V. What will

be the kinetic energy acquired by it

(a) 0.25 E (b) 0.50 E (c) 0.75 E (d) E

Solution: (d) In electric field kinetic energy gain by the charged particle K = qV. Which depends charge and

potential difference applied but not on the mass of the charged particle.

Force on a Charged Conductor.

To find force on a charged conductor (due to repulsion of like charges) imagine a small part XY to be cut

and just separated from the rest of the conductor MLN. The field in the cavity due to the rest of the conductor is

E2, while field due to small part is E1. Then


74 Electrostatics
E2
E1
X E2
Inside the conductor E  E1  E2  0 or E1  E2 ++
M + Y
+ + +
+
 +
N
+
Outside the conductor E  E1  E2  + E2 + +
+ +
0 + + E1 + + +
+ Inside + + + +
+ +
 L + E=0 + +
Thus E1  E2  + + + + +
2 0
(A) (B)

To find force, imagine charged part XY (having charge  dA placed in the cavity MN having field E2).
2 dF  2
Thus force dF  ( dA )E2 or dF  dA . The force per unit area or electric pressure is 
2 0 dA 2 0

The force is always outwards as ( )2 is positive i.e., whether charged positively or negatively, this force
will try to expand the charged body.

A soap bubble or rubber balloon expands on given charge to it (charge of any kind + or –).

Equilibrium of Charged Soap Bubble.

For a charged soap bubble of radius R and surface tension T and charge density  . The pressure due to
T
surface tension 4 and atmospheric pressure Pout act radially inwards and the electrical pressure (Pel ) acts
R
radially outward.
4T  2
The total pressure inside the soap bubble Pin  Pout  
R 2 0
4T  2
Excess pressure inside the charged soap bubble Pin  Pout  Pexcess   . If air pressure inside and
R 2 0
4T  2
outside are assumed equal then Pin  Pout i.e., Pexcess  0 . So, 
R 2 0
This result give us the following formulae Pout Pout
air + +
8 0 T PT Pelec
(1) Radius of bubble R 
2 air
Pin
PT
2
Pin
 R
(2) Surface tension T  + +
8 0
Uncharged Charged
(3) Total charge on the bubble Q  8R 2 0 TR

8T 32 kT
(4) Electric field intensity at the surface of the bubble E  
0 R R
8 RT
(5) Electric potential at the surface V  3RTk 
0
Electrostatics 75

Electric Dipole.

(1) General information: System of two equal and opposite charges separated by a small fixed distance is

called a dipole.

Equatorial
axis
A B
–q +q Dipole axis
2l

(i) Dipole axis: Line joining negative charge to positive charge of a dipole is called its axis. It may also be

termed as its longitudinal axis.

(ii) Equatorial axis: Perpendicular bisector of the dipole is called its equatorial or transverse axis as it is

perpendicular to length.

(iii) Dipole length: The distance between two charges is known as dipole length (L = 2l)

(iv) Dipole moment : It is a quantity which gives information about the strength of dipole. It is a vector

quantity and is directed from negative charge to positive charge along the axis. It is denoted as p and is

defined as the product of the magnitude of either of the charge and the dipole length.
 
i.e. p  q (2 l )

Its S.I. unit is coulomb-metre or Debye (1 Debye = 3.3 × 10–30 C  m) and its dimensions are M0L1T1A1.

Note :  A region surrounding a stationary electric dipole has electric field only.
 When a dielectric is placed in an electric field, its atoms or molecules are considered as tiny

dipoles.

+ – +

 Water (H2O), Chloroform (CHCl3), Ammonia (NH3), HCl, CO molecules are some example of

permanent electric dipole.


76 Electrostatics

+ H+

O2– –

+ H+

(2) Electric field and potential due to an electric dipole: It is better to understand electric dipole with

magnetic dipole.

S.No. Electric dipole Magnetic dipole

(i) System of two equal and opposite charges System of two equal and opposite magnetic poles

separated by a small fixed distance. (Bar magnet) separated by a small fixed distance.
A B –m +m
–q +q S N
2l
 M
p

   
(ii) Electric dipole moment : p  q (2 l ) , directed from Magnetic dipole moment : M  m (2 l ) , directed

 q to +q. It’s S.I. unit is coulomb × meter or from S to N. It’s S.I. unit is ampere × meter2.

Debye.

(iii) Intensity of electric field Intensity of magnetic field


 
e E
  e B
E Be
e g g
Equatorial line Equatorial line

 +  +
 S N
–q 
+q a Ea a Ba
2l 2l
 Axial line  Axial line
p
M

If a, e and g are three points on axial, equatorial If a, e and g are three points on axial, equatorial

and general position at a distance r from the and general position at a distance r from the centre

centre of dipole of dipole


Electrostatics 77

1 2p 0 2 M
on axial point E a  . (directed from – q to +q) on axial point Ba  . (directed from S to N)
4  0 r 3 4 r 3

1 p 0 M
on equatorial point Ee  . (directed from +q to –q) on equatorial point Be  . (directed from N to S)
40 r3 4 r 3

1 p 0 M
on general point E a  . (3 cos 2   1) on general point B a  . (3 cos 2   1)
4  0 r 3 4 r 3
       
Angle between – E a and p is 0o, Ee and p is Angle between – B a and M is 0o, B e and M is
   1
 1 180o, B and M is ( + ) (where tan   tan  )
180o, E and p is ( + ) (where tan   tan  )
2 2

1 p
Electric Potential – At a Va  . , At e V  0
4  0 r 2

1 p cos 
At g V  .
4  0 r2

(3) Dipole (electric/magnetic) in uniform field (electric/magnetic)


 
(i) Torque : If a dipole is placed in an uniform field such that dipole (i.e. p or M ) makes an angle  with

direction of field then two equal and opposite force acting on dipole constitute a couple whose tendency is to

rotate the dipole hence a torque is developed in it and dipole tries to align it self in the direction of field.

Consider an electric dipole in placed in an uniform A magnetic dipole of magnetic moment M is placed in

electric field such that dipole (i.e. p ) makes an angle uniform magnetic field B by making an angle  as

 with the direction of electric field as shown shown


A M
+q F = qE
N F = mB

–q 
F = qE F = mB S
B

(a) Net force on electric dipole Fnet  0

(b) Produced torque  = pE sin (  P  E ) (a) Net force on magnetic dipole Fnet  0

(b) torque  = MB sin (  M  B )


78 Electrostatics

(ii) Work : From the above discussion it is clear that in an uniform electric/magnetic field dipole tries to

align itself in the direction of electric field (i.e. equilibrium position). To change it’s angular position some work

has to be done.

Suppose an electric/magnetic dipole is kept in an uniform electric/magnetic field by making an angle 1

with the field, if it is again turn so that it makes an angle 2 with the field, work done in this process is given by

the formula

+q +q E B
M

1 2 1 2

–q
–q

W  pE (cos  1  cos  2 ) W  MB(cos  1  cos  2 )

If 1 = 0o and 2 =  i.e. initially dipole is kept along If 1 = 0o and 2 =  then W = MB(1 – cos)

the field then it turn through  so work done

W  pE (1  cos  )

(iii) Potential energy : In case of a dipole (in a uniform field), potential energy of dipole is defined as work done

in rotating a dipole from a direction perpendicular to the field to the given direction i.e. if 1 = 90o and 2 =  then –

E B
P M
P M
 

W  U  pE (cos 90  cos  )  U = – pE cos W  U  MB (cos 90  cos  )  U = – MB cos


Electrostatics 79

(iv) Equilibrium of dipole : We know that, for any equilibrium net torque and net force on a particle (or

system) should be zero.

We already discussed when a dipole is placed in an uniform electric/magnetic field net force on dipole is

always zero. But net torque will be zero only when  = 0o or 180o

When  = 0o i.e. dipole is placed along the electric field it is said to be in stable equilibrium, because after

turning it through a small angle, dipole tries to align itself again in the direction of electric field.

When  = 180o i.e. dipole is placed opposite to electric field, it is said to be in unstable equilibrium.

 
p M
     
E E E B B B
  
p M M

 = 0o  = 90O  = 180o  = 0o  = 90O  = 180o

Stable equilibrium Unstable equilibrium Stable equilibrium Unstable equilibrium

=0 max = pE =0 =0 max = MB =0

W=0 W = pE Wmax = 2pE W=0 W = MB Wmax = 2MB

Umin = – pE U=0 Umax = pE Umin = – MB U=0 Umax = MB

(v) Angular SHM : In a uniform electric/magnetic field (intensity E/B) if a dipole (electric/magnetic) is

slightly displaced from it’s stable equilibrium position it executes angular SHM having period of oscillation. If I =

moment of inertia of dipole about the axis passing through it’s centre and perpendicular to it’s length.

I I
For electric dipole : T  2 and For Magnetic dipole : T  2
pE MB

(vi) Dipole-point charge interaction : If a point charge/isolated magnetic pole is placed in dipole field at a
distance r from the mid point of dipole then force experienced by point charge/pole varies according to the
1
relation F 
r3
80 Electrostatics

(vii) Dipole-dipole interaction : When two dipoles placed closed to each other, they experiences a force

due to each other. If suppose two dipoles (1) and (2) are placed as shown in figure then

Both the dipoles are placed in the field of one another hence potential energy dipole (2) is

1 2p
U 2   p 2 E1 cos 0   p 2 E1   p 2  . 31 1
E2
2
E1
4  0 r O O
dU dU 2 –q +q –q +q
then by using F   , Force on dipole (2) is F2  
dr dr P1 P2
r
d  1 2p p  1 6 p1 p 2
 F2    . 13 2    .
dr  4  0 r  4  0 r4
1 6 p1 p 2 1 6 p1 p 2
Similarly force experienced by dipole (1) F1   . 4
so F1  F2   .
4  0 r 4  0 r4
1 6 p1 p 2 1
Negative sign indicates that force is attractive. | F |  . 4
and F  4
4  0 r r

S. No. Relative position of dipole Force Potential energy

(i) 1 6p p 1 2p p
–q +q –q +q . 14 2 (attractive) . 13 2
  4  0 r 4  0 r
P1 P2

(ii) 1 3p p 1 p p
+q +q . 14 2 (repulsive) . 1 2
4  0 r 4  0 r 3
 
P1 P2

–q –q
r

(iii) 1 3p p 0
+q . 14 2 (perpendicular to r )
4  0 r
 –q +q
P1

P2
–q
r
Electrostatics 81

Note :  Same result can also be obtained for magnetic dipole.

(4) Electric dipole in non-uniform electric field: When an electric dipole is placed in a non-uniform field, the

two charges of dipole experiences unequal forces, therefore the net force on the

dipole is not equal to zero. The magnitude of the force is given by the negative

derivative of the potential energy w.r.t. distance along the axis of the dipole i.e.
qE
+q
dU dE
F   p . .
dr dr qE
–q
Due to two unequal forces, a torque is produced which rotate the dipole so

as to align it in the direction of field. When the dipole gets aligned with the field,
E > E
the torque becomes zero and then the unbalanced force acts on the dipole and

the dipole then moves linearly along the direction of field from weaker portion of the field to the stronger portion

of the field. So in non-uniform electric field

(i) Motion of the dipole is translatory and rotatory

(ii) Torque on it may be zero.

Concepts

 For a short dipole, electric field intensity at a point on the axial line is double than at a point on the equatorial line on electric
dipole i.e. Eaxial = 2Eequatorial

1  1 
 It is intresting to note that dipole field E  decreases much rapidly as compared to the field of a point charge  E  .
r 3
 r2 

–q +q Examples based on electric dipole

Example: 84 If the magnitude of intensity of electric field at a distance x on axial line and at a distance y on equatorial
line on a given dipole are equal, then x : y is
3
(a) 1 : 1 (b) 1 : 2 (c) 1 : 2 (d) 2 :1
1 2p 1 p x
Solution: (d)According to the question .  .   (2)1 / 3 : 1
4  0 x 3 4  0 y 3 y

Example: 85 Three charges of (+2q), (– q) and (– q) are placed at the corners A, B and C of an equilateral triangle of
side a as shown in the adjoining figure. Then the dipole moment of this combination is
82 Electrostatics

+2q
(a) qa A

(b) Zero a a

(c) q a 3 B C
–q a –q
2
(d) qa
3

Solution: (c)The charge +2q can be broken in +q, +q. Now as shown in figure we have two equal dipoles inclined at an
angle of 60o. Therefore resultant dipole moment will be P P

p net  p 2  p 2  2 pp cos 60
60O
 3p

 3 qa

Example: 86 An electric dipole is placed along the x-axis at the origin O. A point P is at a distance of 20 cm from this

origin such that OP makes an angle with the x-axis. If the electric field at P makes an angle  with x-
3
axis, the value of  would be

   3 2  3
(a) (b)  tan 1   (c) (d) tan 1  
3 3  2  3  2 
   
Solution: (b)According to question we can draw following figure. y
 E
As we have discussed earlier in theory   
3 P

1  3
tan   tan    tan 1
2 3 2 /3 
x
 3 O 
So,    tan 1 p
3 2
Example: 87 An electric dipole in a uniform electric field experiences

(a) Force and torque both (b) Force but no torque (c) Torque but no force (d) No force and no
torque

Solution: (c)In uniform electric field Fnet = 0, net  0

Example: 89 Two opposite and equal charges 4 × 10–8 coulomb when placed 2 × 10–2 cm away, form a dipole. If this
dipole is placed in an external electric field 4 × 108 newton/coulomb, the value of maximum torque and
the work done in rotating it through 180o will be

(a) 64 × 10–4 Nm and 64 × 10–4 J (b) 32 × 10–4 Nm and 32 × 10–4 J

(c) 64 × 10–4 Nm and 32 × 10–4 J (d) 32 × 10–4 Nm and 64 × 10–4 J


Electrostatics 83

Solution: (d)max = pE and Wmax = 2pE  p = Q × 2l = 4 × 10–8 × 2 × 10–2 × 10–2 = 8 × 10–12 C-m

So, max = 8 × 10–12 × 4 × 108 = 32 × 10–4 N-m and Wmax = 2 × 32 × 10–4 = 64 × 10–4 J

Example: 90 A point charge placed at any point on the axis of an electric dipole at some large distance experiences a
force F. The force acting on the point charge when it’s distance from the dipole is doubled is

F F F
(a) F (b) (c) (d)
2 4 8
1
Solution: (d)Force acting on a point charge in dipole field varies as F  where r is the distance of point charge from
r3
F
the centre of dipole. Hence if r makes double so new force F'  .
8
Example: 91 A point particle of mass M is attached to one end of a massless rigid non-conducting rod of length L.
Another point particle of the same mass is attached to other end of the rod. The two particles carry
charges +q and – q respectively. This arrangement is held in a region of a uniform electric field E such
that the rod makes a small angle  (say of about 5 degrees) with the field direction (see figure). Will be
minimum time, needed for the rod to become parallel to the field after it is set free

+q

E
–q

mL  mL 3 mL 2mL
(a) t  2 (b) t  (c) t  (d) t  
2 pE 2 2 qE 2 2 pE qE
Solution: (b)In the given situation system oscillate in electric field with maximum angular displacement .
It’s time period of oscillation (similar to dipole)
I
T  2 where I = moment of inertia of the system and p  qL
pE
T  I
Hence the minimum time needed for the rod becomes parallel to the field is t  
4 2 pE
2 2
L L ML 2  ML 2  ML
Here I  M  M    t 
2 2 2 2 2  qL  E 2 2 qE

Tricky example: 12

An electric dipole is placed at the origin O and is directed along the x-axis. At a point P, far away
from the dipole, the electric field is parallel to y-axis. OP makes an angle  with the x-axis then
84 Electrostatics

1
(a) tan   3 (b) tan   2 (c)  = 45o (d) tan  
2

Solution: (b) As we know that in this case electric field makes an angle  + with the direction of dipole

1
Where tan   tan 
2

Here  + = 90o    90  

1 1
Hence tan( 90   )  tan   cot   tan  Y
2 2 E
2
 tan   2  tan   2 P


X
O

P

Electric Flux.

(1) Area vector : In many cases, it is convenient to treat area of a surface as a vector. The length of the

vector represents the magnitude of the area and its direction is along the outward drawn normal to the area.
ds

Area ds

(2) Electric flux : The electric flux linked with any surface in an electric field is basically a measure of total
number of lines of forces passing normally through the surface. or

Electric flux through an elementary area ds is defined as the scalar product of area of field i.e.
d  E  ds  E ds cos 

Hence flux from complete area (S)    E ds cos  = ES cos 


ds 
E
If  = 0o, i.e. surface area is perpendicular to the electric field, so flux
linked with it will be max. 

i.e. max = E ds and if  = 90o, min = 0


ds
(3) Unit and Dimensional Formula
Electrostatics 85

N C
S.I. unit –(volt × m) or
m2
It’s Dimensional formula – (ML3T–3A– 1)

(4) Types : For a closed body outward flux is taken to be positive, while inward flux is to be negative

n E

Body Body

Positive – flux Negative-flux


(A) (B)

Gauss’s Law.

1
(1) Definition : According to this law, total electric flux through a closed surface enclosing a charge is
0
1
times the magnitude of the charge enclosed i.e.   (Q enc . )
0

(2) Gaussian Surface : Gauss’s law is valid for symmetrical charge distribution. Gauss’s law is very

helpful in calculating electric field in those cases where electric field is symmetrical around the source

producing it. Electric field can be calculated very easily by the clever choice of a closed surface that encloses

the source charges. Such a surface is called “Gaussian surface”. This surface should pass through the point

where electric field is to be calculated and must have a shape according to the symmetry of source.

e.g. If suppose a charge Q is placed at the centre of a hemisphere, then to calculate the flux through this
body, to encloses the first charge we will have to imagine a Gaussian surface. This imaginary Gaussian

surface will be a hemisphere as shown.

Q Q
Net flux through this closed body  
0

Q
Hence flux coming out from given hemisphere is   .
2 0

(3) Zero flux : The value of flux is zero in the following circumstances
86 Electrostatics

(i) If a dipole is enclosed by a surface (ii) If the magnitude of positive and negative

charges are equal inside a closed surface


+q

–q +q +4q
  0 ; Q enc  0 –5q

Q enc  0,

so,  = 0

(iii) If a closed body (not enclosing any charge) is placed in an electric field (either uniform or non-

uniform) total flux linked with it will be zero

Sphere

2 2
 in  R E   0  out  R E T  0
T
y 
ds 
E


 ds x
ds
a
a

T  0  in   out  E a 2

(4) Flux emergence : Flux linked with a closed body is independent of the shape and size of the body and

position of charge inside it

Q Q Q
Q

Q Q
Q Q T 
T  T  T  
   0
0 0 0
Electrostatics 87

(i) If a hemispherical body is placed in uniform (ii) If a hemispherical body is placed in non-

electric field then flux linked with the curved uniform electric field as shown below. then flux

surface linked with the curved surface.



E

R
R
 curved  2R 2 E

2

 curved  R E

(v) If charge is kept at the centre of cube (iv) If charge is kept at the centre of a face

1
 total  .(Q)
Q 0

Q
 face 
6 0

First we should enclosed the charge by

Q Q assuming a Gaussian surface (an identical


 corner   edge 
8 0 12 0 imaginary cube)
Q
 total 
0
Q Q
 cube  (i.e. from 5 face only)
2 0

1 Q  Q
 face     .
5  2 0  10  0

Concept
1
 In C.G.S.  0  . Hence if 1C charge is enclosed by a closed surface so flux through the surface will be   4  .
4
88 Electrostatics

Example based on electric flux and Gauss’s law

Example: 91 Electric charge is uniformly distributed along a long straight wire of radius 1 mm. The charge per cm length

of the wire is Q coulomb. Another cylindrical surface of radius 50 cm and length 1 m symmetrically

encloses the wire as shown in the figure. The total electric flux passing through the cylindrical surface is

Q
(a) +
0 +
+
100 Q
(b)
0 1m
10 Q
(c)
( 0 ) +
+ 50 cm
100 Q +
(d)
( 0 )

Solution: (b) Given that charge per cm length of the wire is Q. Since 100 cm length of the wire is enclosed so

Q enc  100 Q

100 Q
 Electric flux emerging through cylindrical surface   .
0

Example: 92 A charge Q is situated at the corner A of a cube, the electric flux through the one face of the cube is

Q Q Q Q
(a) (b) (c) (d)
6 0 8 0 24  0 2 0

Solution: (c)For the charge at the corner, we require eight cube to symmetrically enclose it in a Gaussian surface. The
Q Q
total flux  T  . Therefore the flux through one cube will be  cube  . The cube has six faces and
0 8 0

flux linked with three faces (through A) is zero, so flux linked with remaining three faces will . Now as
8 0
Electrostatics 89

the remaining three are identical so flux linked with each of the three faces will be
1 1  Q  1 Q
     .
3  8   0  24  0

Example: 93 A square of side 20 cm is enclosed by a surface of sphere of 80 cm radius. Square and sphere have the
same centre. Four charges + 2 × 10–6 C, – 5 × 10– 6 C, – 3 × 10– 6 C, +6 × 10– 6 C are located at the four
corners of a square, then out going total flux from spherical surface in N–m2/C will be

(a) Zero (b) (16 ) × 10– 6 (c) (8) × 10–6 (d) 36 × 10–6

Solution: (a)Since charge enclosed by Gaussian surface is

 enc .  (2  10 6  5  10 6  3  10 6  6  10 6 )  0 so   0

Example: 94 In a region of space, the electric field is in the x-direction and proportional to x, i.e., E  E 0 xˆi . Consider
an imaginary cubical volume of edge a, with its edges parallel to the axes of coordinates. The charge
inside this cube is
1 1
(a) Zero (b)  0 E 0 a 3 (c) E0 a 3 (d)  0 E0 a 2
0 6
Solution: (b)The field at the face ABCD = E 0 x 0ˆi .

 Flux over the face ABCD = – (E0x0)a2 Y C G

H
The negative sign arises as the field is directed into the cube. D
a
x0
The field at the face EFGH = E 0 (x 0  a)ˆi . B F
E a
 Flux over the face EFGH = E 0 (x 0  a) a 2 A a
X
The flux over the other four faces is zero as the field is parallel to
Z
the surfaces.
1
 Total flux over the cube  E 0 a 2  q
2
where q is the total charge inside the cube.  q   0 E0 a 3 .

Tricky example: 13

In the electric field due to a point charge + Q a spherical closed surface is drawn as shown by the

dotted circle. The electric flux through the surface drawn is zero by Gauss’s law. A conducting

sphere is inserted intersecting the previously drawn Gaussian surface. The electric flux through the

surface

(a) Still remains zero


90 Electrostatics

(b) Non zero but positive

(c) Non-zero but negative +Q

(d) Becomes infinite

Solution: (b) Due to induction some positive charge will lie within the Gaussian surface drawn and hence flux
becomes something positive.

Application of Gauss’s Law.

Gauss’s law is a powerful tool for calculating electric field in case of symmetrical charge distribution by

choosing a Gaussian surface in such away that E is either parallel or perpendicular to it’s various faces.

e.g. Electric field due to infinitely long line of charge : Let us consider a uniformly
+ n̂
2
charged wire of infinite length having a constant linear charge density is +
+
+ a
 charge 
. Let P be a point distant r from the wire at which the electric field is to
+
    l P
+
 length  +

be calculated. 1

Draw a cylinder (Gaussian surface) of radius r and length l around the line charge which encloses the

charge Q ( Q   . l ). Cylindrical Gaussian surface has three surfaces; two circular and one curved for surfaces

(1) and (2) angle between electric field and normal to the surface is 90o i.e.,   90 o.

So flux linked with these surfaces will be zero. Hence total flux will pass through curved surface and it is

  E ds cos  …. (i)

According to Gauss’s law

Q
 …. (ii)
0

Q
Equating equation (i) and (ii)  E ds   0
Electrostatics 91

Q Q
 
E ds 
0
 Ex 2rl 
0

Q  2k   1 
 E   K  
2 0 rl 2 0 r r  4  0 
92 Electrostatics

Capacitance.

(1) Definition: We know that charge given to a conductor increases it’s potential i.e., Q  V  Q  CV

Where C is a proportionality constant, called capacity or capacitance of conductor. Hence capacitance is

the ability of conductor to hold the charge.

Coulomb
(2) Unit and dimensional formula: S.I. unit is  Farad (F)
Volt

Smaller S.I. units are mF, F, nF and pF ( 1mF  10 3 F , 1 F  10 6 F , 1nF  10 9 F ,

1 pF  1 F  10 12 F )

C.G.S. unit is Stat Farad 1 F  9  10 11 Stat Farad . Dimension: [C ]  [M 1 L2 T 4 A 2 ] .

(3) Capacity of an isolated spherical conductor: When charge Q is given to a spherical conductor of

radius R, then potential at the surface of sphere is +


+ + +
+
Q
+ +
R
Q + +
V  k. + +
R O
+ +
+ +
+ + +

 1 
k  
 4  0 

Q 1
Hence it’s capacity C   4 0 R  C  4 πε 0 R  .R
V 9  10 9

in C.G.S. CR

Note :  If earth is assumed to be spherical having radius R  6400 km . It’s theortical capacitance
1
C  6400  10 3  711 F . But for all practical purpose capacitance of earth is taken
9  10 9
infinity.

(4) Energy of a charged conductor : When a conductor is charged it’s potential increases from 0 to V as

shown in the graph; and work is done against repulsion, between charge stored in the conductor and charge

coming from the source (battery). This work is stored as “electrostatic potential energy”

1
From graph : Work done = Area of graph  QV
2 V

Q
Electrostatics 93

1
Hence potential energy U  QV ; By using Q  CV , we can write
2

1 1 Q2
U QV  CV 2 
2 2 2C

(5) Combination of drops : Suppose we have n identical drops each having – Radius – r, Capacitance – c,

Charge – q, Potential – v and Energy – u.

If these drops are combined to form a big drop of – Radius – R, Capacitance – C, Charge – Q, Potential –

V and Energy – U then –

(i) Charge on big drop : Q  nq


4 4
(ii) Radius of big drop : Volume of big drop = n  volume of a single drop i.e., R 3  n  r 3 , R  n 1 /3 r
3 3

(iii) Capacitance of big drop : C  n 1 /3 c

Q nq
(iv) Potential of big drop : V  1/3 V  n 2 /3 v
C n c

1 1
(v) Energy of big drop : U  CV 2  (n 1 / 3 c) (n 2 / 3 v) 2 U  n 5 /3 u
2 2

(6) Sharing of charge : When two conductors joined together through a conducting wire, charge begins to

flow from one conductor to another till both have the same potential, due to flow of charge, loss of energy also

takes place in the form of heat.

Suppose there are two spherical conductors of radii r1 and r2 , having charge Q1 and Q 2 , potential V1 and

V2 , energies U1 and U 2 and capacitance C 1 and C 2 respectively, as shown in figure. If these two spheres are

connected through a conducting wire, then alteration of charge, potential and energy takes place.

Q1 Q2 Q1 Q2
C1 r1 r2 C2 r1 r2
C1 C2
V1 V2 V V
U1 U2 U1 U2
Q1=C1V1 Q2=C2V2 Q1=C1V1 Q2=C2V2

(A) (B)
94 Electrostatics

(i) New charge : According to the conservation of charge Q1  Q 2  Q1'  Q 2'  Q (say), also
Q1' C1 V 4  0 r1 Q1' r1 Q1' r1 Q1'  Q 2' r1  r2
  ,   1 1  
Q 2' C 2 V 4  0 r2 Q 2' r2 Q 2' r2 Q 2' r2

 r   r 
 Q 2'  Q  2  and similarly Q1'  Q  1 
 r1  r2   r1  r2 
Total charge Q  Q 2 Q1'  Q 2' C V  C 2 V2
(ii) Common potential: Common potential (V )   1   V 1 1
Total capacity C1  C 2 C1  C 2 C1  C 2

(iii) Energy loss: As electrical energy stored in the system before and after connecting the spheres is
2
1 1 1 1  C V  C 2 V2 
Ui  C 1 V12  C 2 V22 and U f  (C 1  C 2 ) . V 2  (C 1  C 2 )  1 1 
2 2 2 2  C1  C 2 
C1 C 2
so energy loss ΔU  U i  U f  (V1  V2 ) 2
2(C 1  C 2 )

Concept

 Capacity of a conductor is a constant term, it does not depend upon the charge Q, and potential (V) and nature of the
material of the conductor.

+ + + +
+ + +
+
+
+ +
+
Examples based on sharing of charge, drops and general concept of capacity
+ + +

Example: 95 Eight drops of mercury of same radius and having same charge coalesce to form a big drop. Capacitance

of big drop relative to that of small drop will be [MP PMT 2002, 1990; MNR 1999, 87; DCE 1998]

(a) 16 times (b) 8 times (c) 4 times (d) 2 times

Solution: (d)By using relation C  n 1 / 3 . c  C  (8 )1 / 3 . c  2c

Example: 96 Two spheres A and B of radius 4 cm and 6 cm are given charges of 80 C and 40 C respectively. If they are
connected by a fine wire, the amount of charge flowing from one to the other is

(a) 20  C from A to B (b) 16  C from A to B (c) 32 C from B to A (d) 32  C from A to B


 r1 
Solution: (d)Total charge Q  80  40  120  C . By using the formula Q 1 '  Q   . New charge on sphere A is
 1  r2
r 
 r   4 
Q 'A  Q  A   120
r  r  4  6   48  C . Initially it was 80  C, i.e., 32  C charge flows from A to B.
 A B   

Example: 97 Two insulated metallic spheres of 3 F and 5 F capacitances are charged to 300 V and 500 V

respectively. The energy loss, when they are connected by a wire, is


Electrostatics 95

(a) 0 .012 J (b) 0 . 0218 J (c) 0 . 0375 J (d) 3 . 75 J

C1 C 2
Solution: (c)By using ΔU  (V1  V2 ) 2 ; ΔU  0 . 375 J
2(C1  C 2 )

Example: 98 64 small drops of mercury, each of radius r and charge q coalesce to form a big drop. The ratio of the

surface density of charge of each small drop with that of the big drop is

(a) 1 : 64 (b) 64 : 1 (c) 4 : 1 (d) 1 : 4

2
 Small q / 4 r 2  q  R 
Solution: (d)  2
     ; since R = n1/3r and Q = nq
 Big Q / 4 R  Q  r 

 Small 1  Small 1
So  1/3  
 Big n  Big 4

Tricky example: 14

Two hollow spheres are charged positively. The smaller one is at 50 V and the bigger one is at 100

V. How should they be arranged so that the charge flows from the smaller to the bigger sphere
when they are connected by a wire

(a) By placing them close to each other

(b) By placing them at very large distance from each other

(c) By placing the smaller sphere inside the bigger one

(d) Information is insufficient

Solution: (c) By placing the smaller sphere inside the bigger one. The potential of the smaller one will now be

150 V. So on connecting it with the bigger one charge will flow from the smaller one to the bigger

one.
96 Electrostatics

Capacitor.

(1) Definition: A capacitor is a device that stores electric energy. It is also named condenser.

or
A capacitor is a pair of two conductors of any shape, which are close to each other and have equal and
opposite charge.

+Q –Q
+ –
+ –
+ –
+ –
+

+

(2) Symbol : The symbol of capacitor are shown below

or variable capacitor

(3) Capacitance: The capacitance of a capacitor is defined as the magnitude of the charge Q on the
Q
positive plate divided by the magnitude of the potential difference V between the plates i.e., C 
V

(4) Charging : A capacitor get’s charged when a battery is connected across the plates. The plate

attached to the positive terminal of the battery get’s positively charged and the one joined to the negative

terminal get’s negatively charged. Once capacitor get’s fully charged, flow of charge carriers stops in the circuit

and in this condition potential difference across the plates of capacitor is same as the potential difference

across the terminals of battery (say V).

+Q –Q
C
+ –


+ – + –
V V
Electrostatics 97

(5) Charge on capacitor: Net charge on a capacitor is always zero, but when we speaks of the charge Q

on a capacitor, we are referring to the magnitude of the charge on each plate. Charge distribution in making of

parallel plate capacitor can easily be understand by reading carefully the following sequence of figures –

+Q
Q Q Q
  Q Q Q
2 2     +Q –Q
2 2 2 2
  

X X X Y Y X Y

(6) Energy stored: When a capacitor is charged by a voltage source (say battery) it stores the electric
energy. If C = Capacitance of capacitor; Q = Charge on capacitor and V = Potential difference across
1 1 Q2
capacitor then energy stored in capacitor U  CV 2  QV 
2 2 2C

Note :  In charging capacitor by battery half the energy supplied is stored in the capacitor and
remaining half energy (1/2 QV) is lost in the form of heat.

(7) Types of capacitors: Capacitors are of mainly three types as described in given table

Parallel Plate Capacitor Spherical Capacitor Cylindrical Capacitor

It consists of two parallel metallic It consists of two concentric It consists of two concentric
plates (may be circular, rectangular, conducting spheres of radii a and b cylinders of radii a and b (a < b),
square) separated by a small (a < b). Inner sphere is given inner cylinder is given charge +Q
distance charge +Q, while outer sphere is while outer cylinder is earthed.
+Q –Q
+ – earthed Common length of the cylinders is l
+ –
then
+ air – – – b
A – a q –q
– –
+ – + –Q

– + +
+Q
+ – b–
– + + l
+ – a
– –
d + +
– + –
– –

–Q– – – +Q


a –
– – b
– –
98 Electrostatics

A = Effective overlaping area of ab Capacitance


Capacitance C  4 πε 0 .
b a
each plate 2 πε 0 l
ab C 
in C.G.S. C  . In the presence b 
d = Separation between the plates ba log e  
a
of dielectric medium (dielectric
Q = Magnitude of charge on the
constant K) between the spheres In the presence of dielectric
inner side of each plate
ab medium (dielectric constant K)
 = Surface density of charge of C '  4  0 K
b a capacitance increases by K times
 Q Special Case :
each plate    and
 A
If outer sphere is given a charge 2 0 Kl
V = Potential difference across the C' 
+Q while inner sphere is earthed b
log e  
plates a
E = Electric field between the
  
plates   
 0 
ε A
Capacitance : C  0
d Induced charge on the inner
A sphere
in C.G.S. : C
4 πd
a b2
If a dielectric medium of dielectric Q'   . Q , C'  4 πε 0 .
b b a
constant K is filled completely
This arrangement is not a
between the plates then
capacitor. But it’s capacitance is
capacitance increases by K times
equivalent to the sum of
C'  KC capacitance of spherical capacitor
and spherical conductor i.e.
b2 ab
4  0 .  4  0  4  0 b
ba ba
Electrostatics 99

Concepts

 It is a very common misconception that a capacitor stores charge but actually a capacitor stores electric energy in the
electrostatic field between the plates.

 Two plates of unequal area can also form a capacitor because effective overlapping area is considered.

 If two plates are placed side by side then three capacitors are formed. One between distant earthed bodies and the first
face of the first plate, the second between the two plates and the third between the second face of the second plate and
distant earthed objects. However the capacitances of the first and third capacitors are negligibly small in comparision to that
between the plates which is the main capacitance.

 Capacitance of a parallel plate capacitor depends upon the effective overlapping area of plates (C  A) , separation

 1
between the plates  C   and dielectric medium filled between the plates. While it is independent of charge given,
 d 
potential raised or nature of metals and thickness of plates.

 The distance between the plates is kept small to avoid fringing or edge effect (non-uniformity of the field) at the bounderies
of the plates.

+ –
+ –
+ –
+ –
+ –
+ –

 Spherical conductor is equivalent to a spherical capacitor with it’s outer sphere of infinite radius.
 A spherical capacitor behaves as a parallel plate capacitor if it’s spherical surfaces have large radii and are close to each
other.
 The intensity of electric field between the plates of a parallel plate capacitor (E = /0) does not depends upon the distance
100 Electrostatics

between them.
 The plates of a parallel plate capacitor are being moved away with some velocity. If the plate separation at any instant of
1
time is ‘d’ then the rate of change of capacitance with time is proportional to .
d2
 Radial and non-uniform electric field exists between the spherical surfaces of spherical capacitor.
 Two large conducting plates X and Y kept close to each other. The plate X is given a charge Q 1 while plate Y is given a

charge Q 2 (Q 1  Q 2 ) , the distribution of charge on the four faces a, b, c, d will be as shown in the following figure.

Q1 X Q2 X
 Q1  Q 2 
Y   Y
 2 
b d 
 Q1  Q 2   Q1  Q 2   Q1  Q 2 
     
a c  2   2   2 

+ –
+ –
Example based on simple concepts of capacitor
+ –

Example: 99 The capacity of pure capacitor is 1 farad. In D.C. circuit, its effective resistance will be

1
(a) Zero (b) Infinite (c) 1 ohm (d) ohm
2

Solution: (b)Capacitor does not work in D.C. for D.C. it’s effective resistance is infinite i.e. it blocks the current to flow in

the circuit.

Example: 100 A light bulb, a capacitor and a battery are connected together as shown here, with switch S initially open.

When the switch S is closed, which one of the following is true

s + –
Electrostatics 101

(a) The bulb will light up for an instant when the capacitor starts charging

(b) The bulb will light up when the capacitor is fully charged

(c) The bulb will not light up at all

(d) The bulb will light up and go off at regular intervals

Solution: (a)Current through the circuit can flow only for the small time of charging, once capacitor get’s charged it blocks

the current through the circuit and bulb will go off.

Example: 101 Capacity of a parallel plate condenser is 10F when the distance between its plates is 8 cm . If the distance

between the plates is reduced to 4cm, its capacity will be

(a) 10F (b) 15F (c) 20F (d) 40F

0 A 1 C1 d d1 8
Solution: (c) C     2 or C2   C1   10  20  F
d d C2 d1 d2 4

Example: 102 What is the area of the plates of a 3 F parallel plate capacitor, if the separation between the plates is
5 mm

(a) 1 .694  10 9 m 2 (b) 4 . 529  10 9 m 2 (c) 9 . 281  10 9 m 2 (d) 12 . 981  10 9 m 2

0 A Cd 3  5  10 3
Solution: (a)By using the relation C   A   1 . 694  10 9 m 2 .
d 0 8 . 85  10 12

Example: 103 If potential difference of a condenser (6  F) is changed from 10 V to 20 V then increase in energy is

(a) 2  10 4 J (b) 4  10 4 J (c) 3  10 4 J (d) 9  10 4 J

1 1
Solution: (d)Initial energy U i  CV12 ; Final energy U f  CV 22
2 2
1 1
 Increase in energy U  U f  U i  C (V22  V12 )   6  10 6 (20 2  10 2 )  9  10  4 J .
2 2

Example: 104 A spherical capacitor consists of two concentric spherical conductors. The inner one of radius R 1
maintained at potential V1 and the outer conductor of radius R 2 at potential V2 . The potential at a point
P at a distance x from the centre (where R 2  x  R1 ) is
V1  V2 V1 R1 (R 2  x )  V2 R 2 (x  R1 )
(a) (x  R1 ) (b)
R 2  R1 (R 2  R 1 ) x
V2 x (V1  V2 )
(c) V1  (d) x
(R 2  R 1 ) (R 1  R 2 )
Solution: (b)Let Q 1 and Q 2 be the charges on the inner and the outer sphere respectively. Now V1 is the total potential
on the sphere of radius R1,
102 Electrostatics

Q1 Q 2
So, V1   …….. (i)
R1 R 2

and V2 is the total potential on the surface of sphere of radius R 2 ,


Q 2 Q1
So, V2   …….. (ii)
R2 R2

If V be the potential at point P which lies at a distance x from the common centre then

Q1 Q 2 Q1 Q 1 1  Q (R  x )
V    V1  1  Q1     V1  1 1  V1 ……..(iii)
x R2 x R1  x R1  xR 1

Substracting (ii) from (i)


Q1 Q 2 (V  V2 )R 1 R 2
V1  V2    (V1  V2 )R1 R 2  R 2 Q1  R1 Q1  Q 1  1
R1 R 2 R 2  R1

Now substituting it in equation (iii), we have


(R1  x )(V1  V2 )R1 R 2 V1 R 1 (R 2  x )  V2 R 2 (x  R 1 )
V  V1  V 
xR 1 (R 2  R1 ) x (R 2  R 1 )

Example: 105 The diameter of each plate of an air capacitor is 4 cm. To make the capacity of this plate capacitor equal
to that of 20 cm diameter sphere, the distance between the plates will be

(a) 4  10 3 m (b) 1  10 3 m (c) 1 cm (d) 1  10 3 cm

0 A A  (2  10 2 ) 2
Solution: (b)According to the question  4 0 R  d   1  10  3 m .
d 4R 4  10  10  2

Example: 106 A spherical condenser has inner and outer spheres of radii a and b respectively. The space between the
two is filled with air. The difference between the capacities of two condensers formed when outer sphere
is earthed and when inner sphere is earthed will be

 b 
(a) Zero (b) 4 0 a (c) 4 0 b (d) 4 0 a  
b a
ab
Solution: (c)Capacitance when outer sphere is earthed C1  4  0 . and capacitance when inner sphere is earthed
b a
b2
C 2  4 0 . . Hence C 2  C 1  4  0 . b
b a
Example: 107 After charging a capacitor of capacitance 4  F upto a potential 400 V, its plates are connected with a
resistance of 1 k  . The heat produced in the resistance will be

(a) 0.16 J (b) 1.28 J (c) 0.64 J (d) 0.32 J

Solution: (d)This is the discharging condition of capacitor and in this condition energy released
1 1
U CV 2   4  10 6  (400 ) 2  0 . 32 J  0 . 32 J .
2 2
Electrostatics 103

Example: 108 The amount of work done in increasing the voltage across the plates of a capacitor from 5 V to 10 V is W.
The work done in increasing it from 10 V to 15 V will be
(a) 0.6 W (b) W (c) 1.25 W (d) 1.67 W
1
Solution: (d)As we know that work done  U final  U initial  C (V22  V12 )
2
When potential difference increases from 5 V to 10 V then
1
W C (10 2  5 2 ) ……..(i)
2
When potential difference increases from 10 V to 15 V then
1
W'  C (15 2  10 2 ) ……..(ii)
2
On solving equation (i) and (ii) we get
W '  1 . 67 W .

Tricky example: 15

In an isolated parallel plate capacitor of capacitance C, the four surface have charges Q1 , Q 2 , Q 3
and Q 4 as shown. The potential difference between the plates is [UP

Q1 Q3

Q2 Q4

Q1  Q 2  Q 3  Q 4 Q2  Q3 Q2  Q3 Q1  Q 4
(a) (b) (c) (d)
2C 2C 2C 2C

Solution: (c) Plane conducting surfaces facing each other must have equal and opposite charge densities. Here

as the plate areas are equal, Q 2  Q 3 .

The charge on a capacitor means the charge on the inner surface of the positive plate (here it is Q 2 )

charge Q 2Q 2
Potential difference between the plates   2 
capacitanc e C 2C

Q 2  (Q 2 ) Q 2  Q 3
  .
2C 2C
104 Electrostatics

Dielectric.

Dielectrics are insulating (non-conducting) materials which transmits electric effect without conducting we

know that in every atom, there is a positively charged nucleus and a negatively charged electron cloud

surrounding it. The two oppositely charged regions have their own centres of charge. The centre of positive

charge is the centre of mass of positively charged protons in the nucleus. The centre of negative charge is the

centre of mass of negatively charged electrons in the atoms/molecules.

(1) Type of Dielectrics: Dielectrics are of two types –

(i) Polar dielectrics : Like water, Alcohol, CO 2 , NH 3 , (ii) Non polar dielectric : Like N 2 , O 2 , Benzene,

HCl etc. are made of polar atoms/molecules. Methane etc. are made of non-polar atoms/molecules.

In polar molecules when no electric field is applied In non-polar molecules, when no electric field is applied

centre of positive charges does not coincide with the the centre of positive charge coincides with the centre of

centre of negative charges. negative charge in the molecule. Each molecule has

O– – zero dipole moment in its normal state.

105o

– + + – P 0
H+
 H
+

A polar molecule has permanent electric dipole


 When electric field is applied, positive charge
moment (p ) in the absence of electric field also. But a

polar dielectric has net dipole moment is zero in the experiences a force in the direction of electric field and

absence of electric field because polar molecules are negative charge experiences a force in the direction

randomly oriented as shown in figure. opposite to the field i.e., molecules becomes induced

electric dipole.
– – 
+
+ + – – E

+
+
– + +

In the presence of electric field polar molecules –

tends to line up in the direction of electric field, and the

substance has finite dipole moment. Note :  In general, any non-conducting, material can
Electrostatics 105

be called as a dielectric but broadly non conducting

– + – + – + material having non polar molecules referred to as


– +
– + – + – + dielectric because induced dipole moment is created in

P the non polar molecule.

(2) Polarization of a dielectric slab : It is the process of inducing


E
equal and opposite charges on the two faces of the dielectric on the + –
+ + – + –
+
application of electric field. + –
+ – + – + –
+ Ei –

Suppose a dielectric slab is inserted between the plates of a + – + – + –


+ –
– + – + –
capacitor. As shown in the figure. +
+ –
– + – + –
+

Induced electric field inside the dielectric is E i , hence this induced

electric field decreases the main field E to E  E i i.e., New electric field between the plates will be E '  E  E i .

(3) Dielectric constant : After placing a dielectric slab in an electric field. The net field is decreased in that
region hence
E
If E  Original electric field and E '  Reduced electric field. Then  K where K is called dielectric
E'
constant

K is also known as relative permittivity ( r ) of the material or SIC (Specific Inductive Capacitance)
The value of K is always greater than one. For vacuum there is no polarization and hence E  E ' and

K 1
(4) Dielectric breakdown and dielectric strength : If a very high electric field is created in a dielectric, the

outer electrons may get detached from their parent atoms. The dielectric then behaves like a conductor. This

phenomenon is known as dielectric breakdown.

The maximum value of electric field (or potential gradient) that a dielectric material can tolerate without it’s

electric breakdown is called it’s dielectric strength.

V kV
S.I. unit of dielectric strength of a material is but practical unit is .
m mm

Variation of Different Variables (Q, C, V, E and U) of Parallel Plate Capacitor.


106 Electrostatics

Suppose we have an air filled charged parallel plate capacitor having variables are as follows :
+Q –Q
Q
Charge – Q, Surface charge density –   , Capacitance – C   0 A + –
A d + –

+ –
Air
Potential difference across the plates – V  E . d A + –

+ –
Electric field between the plates – E    Q + –
0 A 0 d

Energy stored – U  1 CV 2 Q2 1 V
  QV
2 2C 2

(1) When dielectric is completely filled between plates : If a dielectric slab is fills completely the gap
K 0 A
between the plates, capacitance increases by K times i.e., C '   C '  KC
d

The effect of dielectric on other variables such as charge. Potential difference field and energy associated with a

capacitor depends on the fact that whether the charged capacitor is disconnected from the battery or battery is still

connected.

Quantity Battery is Removed Battery Remains connected

A K A K

d d
V

Capacity C = KC C = KC

Charge Q = Q (Charge is conserved) Q = KQ

Potential V = V/K V = V (Since Battery maintains the potential


difference)

Intensity E = E/K E = E

Energy U = U/ K U = U/ K

Note :  If nothing is said it is to be assumed that battery is disconnected.


Electrostatics 107

(2) When dielectric is partially filled between the plates : If a dielectric slab of thickness t (t  d ) is inserted

between the plates as shown below, then E  Main electric field between the plates, E i  Induced electric field

in dielectric. E '  (E  E i )  The reduced value of electric field in the dielectric. Potential difference between the

two plates of capacitor is given by E


+ – + –
+ – + –
E
V '  E (d  t )  E ' t  E (d  t )  .t + – + –
K A Ei
+ – + –

 V '  E  d  t  t     d  t  t   Q  d  t  t  + – + –

K 0  K A 0  K + – + –

d
Now capacitance of the capacitor

Q 0 A
C'   C' 
V' t
d t
K
t1 t2 t2 t1 t2 t3 t4

A K1 K2 K3 A K1 K2 K3 K4

d d

0 A 0 A
C'  C' 
 t t t   t1 t t t 
d  (t1  t 2  t3  ........)   1  2  3  ........    2  3  4 
K
 1 K 2 K 3  K
 1 K 2 K 3 K 4 

(3) When a metallic slab is inserted between the plates:

A K= or A
A K=

d d d
108 Electrostatics

0 A
Capacitance C'  C '   (In this case capacitor is said to be short
(d  t)

circuited)

(4) When separation between the plates is changing: If separation between the plates changes then it’s
1
capacitance also changes according to C  . The effect on other variables depends on the fact that whether
d
the charged capacitor is disconnected from the battery or battery is still connected.

(i) Separation is increasing

Quantity Battery is removed Battery remains connected

A
A

d
d
V

Capacity 1 Decreases i.e., C '  C


Decreases because C  i.e., C '  C
d

Charge Remains constant because a battery is not Decreases because battery is present i.e.,

present Q'  Q

i.e., Q'  Q Remaining charge (Q  Q ' ) goes back to the

battery.

1
Potential Increases because V  Q  V  i.e., V   V (Since Battery maintains the potential
C C
difference difference)
V'  V

Electric Remains constant because E    Q Decrease because E  Q  EQ


0 A 0 A 0
field
i.e., E'  E i.e., E'  E

Q2 1 1
Increases because U   U Decreases because U  CV 2
 UC
2C C 2
Energy
i.e., U'  U i.e., U'  U
Electrostatics 109

(ii) Separation is decreasing

Quantity Battery is removed Battery remains connected

Capacity 1 Increases i.e., C '  C


Increases because C  i.e., C '  C
d

Charge Remains constant because battery is not present Increases because battery is present i.e.,

i.e., Q'  Q Q'  Q

Remaining charge (Q '  Q) supplied from the

battery.

Potential Q 1 V   V (Since Battery maintains the potential


Decreases because V  V  i.e.,
C C
difference difference)
V'  V

Electric Remains constant because E    Q Increases because E  Q  EQ


0 A 0 A 0
field
i.e., E'  E i.e., E'  E

Q2 1 1
Decreases because U   U Increases because U  CV 2
 UC
2C C 2
Energy
i.e., U'  U i.e., U'  U

Force Between the Plates of a Parallel Plate Capacitor.


Field due to charge on one plate on the other is E  , hence the force F  QE
2 0
+ –
   2 + –
F  A      A

 2 0  2 0
A
+
Air
+ –
2 2
 A Q + –
 | F|   + –
2 0 2 0 A d

Energy Density Between the Plates of a Parallel Plate Capacitor.

The energy stored in a capacitor is not localised on the charges or the plates but is distributed in the field.

And as in case of a parallel plate capacitor field is only between the plates i.e. in a volume (A× d), the so called

energy density.
110 Electrostatics

1
CV 2 2
Energy 2 1  A  V 2 1 V  1
Hence Energy density     0   0    0 E2.
Volume Ad 2  d  Ad 2 d 2

Concepts

t
 In the expression of capacitance of parallel plate capacitor filled partially with dielectric term  d  t   is known as
 K
effective air separation between the plates.

 When dielectric is partially filled between the plates of a parallel plate capacitor then it’s capacitance increases but potential
 A
difference decreases. To maintain the capacitance and potential difference of capacitor as before (i.e., c  0 ,
d

V  d ) separation between the plates has to be increased. Suppose separation is increased by d ' so in this case
0
0 A  A t
 0 which gives us K 
 t  d t  d'
 d  d ' t  
 K

Example based on capacitor with dielectric

Example: 109 The mean electric energy density between the plates of a charged capacitor is (here Q = Charge on the
capacitor and A = Area of the capacitor plate)

Q2 Q Q2
(a) (b) (c) (d) None of these
2 0 A 2 2 0 A 2 2 0 A
2
1 1  Q  Q2
Solution: (a)Energy density  0 E2  0    .
2 2  A 0  2 0 A 2

Example: 110 Plate separation of a 15  F capacitor is 2 mm. A dielectric slab (K = 2) of thickness 1 mm is inserted
between the plates. Then new capacitance is given by

(a) 15  F (b) 20  F (c) 30  F (d) 25  F


0 A
Solution: (b)Given C   15  F ……..(i)
d
0 A 0 A 2
Then by using C '      0 A  10 3 ; From equation (i) C '  20  F.
t 103 3
dt 2  10  3  10  3 
K 2

Example: 111 There is an air filled 1 pF parallel plate capacitor. When the plate separation is doubled and the space is

filled with wax, the capacitance increases to 2 pF . The dielectric constant of wax is

(a) 2 (b) 4 (c) 6 (d) 8


Electrostatics 111

Solution: (b) Given that capacitance C  1 pF

C
After doubling the separation between the plates C ' 
2
KC
and when dielectric medium of dielectric constant k filled between the plates then C ' 
2

KC
According to the question, C '  2  K  4.
2

Example: 112 If a slab of insulating material 4  10 5 m thick is introduced between the plate of a parallel plate capacitor,

the distance between the plates has to be increased by 3 .5  10 5 m to restore the capacity to original

value. Then the dielectric constant of the material of slab is

(a) 10 (b) 12 (c) 6 (d) 8

t 4  10 5
Solution: (d)By using K  ; here t  4  10 5 m ; d '  3 . 5  10 5 m  K =8
t  d' 4  10 5  3 . 5  10 5

Example: 113 The force between the plates of a parallel plate capacitor of capacitance C and distance of separation of

the plates d with a potential difference V between the plates, is

CV 2 C 2V 2 C 2V 2 V 2d
(a) (b) (c) (d)
2d 2d 2 d2 C

Q2 C 2 V 2 CV 2
Solution: (a)Since F   F  .
2 0 A 2 0 A 2d

Example: 114 A capacitor when filled with a dielectric K  3 has charge Q 0 , voltage V0 and field E 0 . If the dielectric is

replaced with another one having K = 9, the new values of charge, voltage and field will be respectively

V0 V0 E 0
(a) 3 Q 0 , 3 V0 , 3 E 0 (b) Q 0 , 3 V0 , 3 E 0 (c) Q 0 , , 3 E0 (d) Q 0 , ,
3 3 3

Solution: (d)Suppose, charge, potential difference and electric field for capacitor without dielectric medium are Q, V and E

respectively

With dielectric medium of K  3 With dielectric medium of K  9

Charge Q 0  Q Charge Q '  Q  Q 0

V V V
Potential difference V0  Potential difference V '   0
3 9 3

E E E0
Electric field E 0  Electric field E '   .
3 9 3
112 Electrostatics

Example: 115 A slab of copper of thickness b is inserted in between the plates of parallel plate capacitor as shown in
d
the figure. The separation between the plates is d. If b  then the ratio of capacities of the capacitor
2
after and before inserting the slab will be

A = Area

d Cu b

(a) 2 :1 (b) 2 : 1 (c) 1 : 1 (d) 1 : 2

0 A  A
Solution: (b)Capacitance before inserting the slab C  and capacitance after inserting the slab C '  0 .
d dt

d 2 0 A C' 2
Where t  b  so C '  hence,  .
2 d C 1

Example: 116 The capacity of a parallel plate condenser is C 0 . If a dielectric of relative permitivity  r and thickness

equal to one fourth the plate separation is placed between the plates, then its capacity becomes C. The
C
value of will be
C0

5 r 4 r 3 r 2 r
(a) (b) (c) (d)
4 r  1 3 r  1 2 r  1 r  1

0 A 0 A
Solution: (b)Initially capacitance C 0  ……..(i) Finally capacitance C  ……..(ii)
d d d /4
d 
4 r

C 4 r
By dividing equation (ii) by equation (i) 
C0 3 r  1

Tricky example: 16

An air capacitor of capacity C  10  F is connected to a constant voltage battery of 12 V. Now the

space between the plates is filled with a liquid of dielectric constant 5. The charge that flows now

from battery to the capacitor is

(a) 120  C (b) 600  C (c) 480  C (d) 24  C

Solution: (c) Initially charge on the capacitor Q i  10  12  120  C


Electrostatics 113

When dielectric medium is filled, so capacitance becomes K times, i.e. new capacitance

C '  5  10  50  C

Final charge on the capacitor Q f  50  12  600  C

Hence additional charge supplied by the battery  Q f  Q i  480  C.

Grouping of Capacitors.

Series grouping Parallel grouping

(1) Charge on each capacitor remains same and equals (1) Potential difference across
+Q1 –Q1
+ –
to the main charge supplied
C
by the battery
C2 C3 each capacitor remains + –
1 + –
+Q –Q +Q –Q +Q –Q + –
+ – + – + – same and equal to the Q1
+ – + – + – +Q2 –Q2
+ – + – + –
Q + – + – + – applied potential difference +
+


Q2 + –
V1 V2 V3 + –

V = V1 + V2 + V3 + – Q Q3 +Q3
+ –
–Q3
+ –
V Q = Q1 + Q2 + Q3 + –

+

V
(2) Equivalent capacitance
1 1 1 1
   or C eq  (C 11  C 21  C 31 ) 1
Ceq C1 C2 C3
(3) In series combination potential difference and energy (2) Ceq = C1 + C2 + C3
distribution in the reverse ratio of capacitance i.e.,
1 1
V and U  . (3) In parallel combination charge and energy distributes
C C
(4) If two capacitors having capacitances C1 and C2 are in the ratio of capacitance i.e. Q  C and U  C

connected in series then


(4) If two capacitors having capacitance C1 and C2
C1 C 2 Multiplication
C eq   respectively are connected in parallel then
C1  C 2 Addition
 C1    C eq  C 1  C 2
V1    . V and V 2   C 2
 C  C  . V
 1  C2
C   1 2   C1   C2 
Q 1    . Q and Q 2    . Q
(5) If n identical capacitors each having capacitances C  C1  C 2   C1  C 2 
are connected in series with supply voltage V then (5) If n identical capacitors are connected in parallel
C
Equivalent capacitance C eq  and Potential Equivalent capacitance C eq  nC and Charge on
n
V Q
difference across each capacitor V '  . each capacitor Q ' 
n n
114 Electrostatics

Redistribution of Charge Between Two Capacitors.


When a charged capacitor is connected across an uncharged capacitor, then redistribution of charge
occur to equalize the potential difference across each capacitor. Some
energy is also wasted in the form of heat.
– – – – – –
C1 C2
Suppose we have two charged capacitors C1 and C2 after ++ +
Q1 + + +
Q2
disconnecting these two from their respective batteries. These two capacitors V1 V2

are connected to each other as shown below (positive plate of one capacitor
is connected to positive plate of other while negative plate of one is connected to negative plate of other)
 C1 
Charge on capacitors redistributed and new charge on them will be Q1'  Q   ,
 C1  C 2 
 C2 
Q 2'  Q  
 C1  C 2 
Q1  Q 2 C 1 V1  C 2 V2 C1 C 2
The common potential V   and loss of energy U  (V1  V2 ) 2
C1  C 2 C1  C 2 2 (C 1  C 2 )
Note :  Two capacitors of capacitances C1 and C2 are charged to potential of V1 and V2 respectively. After
disconnecting from batteries they are again connected to each other with reverse polarity i.e., positive
plateof a capacitor connected to negative plate of other. So common potential
Q  Q 2 C 1 V1  C 2 V2
V 1  .
C1  C 2 C1  C 2
Electrostatics 115

Circuit With Resistors and Capacitors.

(1) A resistor may be connected either in series or in parallel with the capacitor as shown below

Series RC Circuit Parallel RC Circuit

C R R

S V0 S V0

In this combination capacitor takes longer time to Resistor has no effect on the charging of capacitor.

charge.

The charging current is maximum in the beginning; it Resistor provides an alternative path for the electric

decreases with time and becomes zero after a long current.

time.

(2) Three states of RC circuits

(i) Initial state : i.e., just after closing the switch or just after opening the switch.

(ii) Transient state : or instantaneous state i.e., any time after closing or opening the switch.

(iii) Steady state : i.e., a long time after closing or opening the switch. In the steady state condition, the

capacitor is charged or discharged.

(3) Charging and discharging of capacitor in series RC circuit : As shown in the following figure (i) when

switch S is closed, capacitor start charging. In this transient state potential difference appears across capacitor

as well as resistor. When capacitor gets fully charged the entire potential difference appeared across the

capacitor and nothing is left for the resistor. [shown in figure (ii)]

C R (ii) C R Steady state


(i) + – + –
V V V0 Q0
Transient state i V0  (Max charge)
C

+ – + –
S V0 S V0
116 Electrostatics

 t

(i) Charging : In transient state of charging charge on the capacitor at any instant Q  Q 0  1  e RC  and
 
 
 t

potential difference across the capacitor at any instant V  V0  1  e RC 
 
 

Q0 V0

Q Q = Q0(1 – e–t/RC) VC VC = V0(1 – e–t/RC)

O t O t

(ii) Discharging : After the completion of charging, if battery is removed capacitor starts discharging. In

transient state charge on the capacitor at any instant Q  Q 0 e  t / RC and potential difference cross the capacitor

at any instant V  V0 e  t / CR .

Q0 V0

Q Q= Q0 e–t/RC VC VC = V0 e–t/RC

O t O t

(iii) Time constant () : The quantity RC is called the time constant i.e.,   RC .

In charging : It is defined as the time during which charge on the capacitor rises to 0.63 times (63%) the

maximum value. That is when t    RC , Q  Q 0 (1  e 1 )  0 . 639 Q 0 or

In discharging : It is defined as the time during which charge on a capacitor falls to 0.37 times (37%) of

the initial charge on the capacitor that is when t    RC , Q  Q 0 (e 1 )  0 . 37 Q 0


Electrostatics 117

(iv) Mixed RC circuit : In a mixed RC circuit as shown below, when switch S is closed current flows

through the branch containing resistor as well as through the branch contains capacitor and resistor (because

capacitor is in the process of charging)

R1 R1
i

C R2 C R2
No
i S i S
R R current
V0 V0
Transient state Steady state

When capacitor gets fully charged (steady state), no current flows through the line in which capacitor is
V0
connected. Therefore the current through resistor R1 is , hence potential difference across resistance
R 1  r 
V0
will be equal to R . The same potential difference will appear across the capacitor, hence charge on
R 1  r  1
CV 0 R 1
capacitor in steady state Q 
R 1  r 
2 4 6

1 3 5 7

Concepts

 In series combination equivalent capacitance is always lesser than that of either of the individual capacitors e.g. If two
capacitors having capacitances 3F and 6F respectively are connected in series then equivalent capacitance
36
C eq   2 F Which is lesser than the smallest capacitance (3F) of the network.
36
 In parallel combination, equivalent capacitance is always greater than the maximum capacitance of either capacitor in
network.
 If n identical plates arranged as shown below, constitutes (n – 1) capacitors in series. If each capacitors having capacitance
0 A  A
then C eq  0
d (n  1)d

In this situation except two extreme plates each plate is common to adjacent capacitors.

 If n identical plates are arranged such that even no. of plates are connected together and odd number of plates are
connected together, then (n – 1) capacitors will be formed and they will be in parallel grouping.
118 Electrostatics

+ – + – + – + –
+ –
+ – + – + – + –
+ – + – + – + –
+ – + – + – + –

0 A
Equivalent capacitance C '  (n  1) C Where C = capacitance of a capacitor 
d

 If n identical capacitors are connected in parallel which are charged to a potential V. If these are separated and connected in
series then potential difference of combination will be nV.

Network Solving.

To solve capacitive network for equivalent capacitance following guidelines should be followed.

Guideline 1. Identify the two points across which the equivalent capacitance is to be calculated.

Guideline 2. Connect (Imagine) a battery between these points.

Guideline 3. Solve the network from the point (reference point) which is farthest from the points
between which we have to calculate the equivalent capacitance. (The point is likely to be not a node)

(1) Simple circuits : Suppose equivalent capacitance is to be determined in the following networks between
points A and B
Electrostatics 119

Suppose equivalent capacitance is to be determined in the following networks between points A and B

(i)

36 C  2  3  5 F
 2 F 2F AB
36
6F 3F 6F
3F
 
A 3F B
A B A 3F B
3F

(ii) 3 + 3 = 6F 6
6F 6F 3F  3 F C  3  2  5 μF
AB
2
  
3F 6F
3F 6F 6F
6F 6F 6F
3F 6F
6
 3 F
2 B
A B A 2F A 2F B A 2F B
2F

(iii)
9F Series
9F 9F 9 9F 9 9 9F
A A 9 A
F F  3 F F
3 +
6F 6F 9F + 6 6F 6 6F 3
 –
9 –
F F F F
B B B
9F 9F 9F 9F 9F Parallel
9F 9F 9F
 6 + 3 = 9F
9 9F
A
F
 + 6 9F
By similar process CAB = 3F –
F
B
9F 9F
120 Electrostatics

(2) Circuits with extra wire: If there is no capacitor in any branch of a network then every point of this

branch will be at same potential. Suppose equivalent capacitance is to be determine in following cases

(i)
A A C
C C C  C B C C
B  C
A B
A B A A A B C
B B
+ –

CAB = 3C

(ii) A A B
No p.d.
C  C C
 across
C C
A B A B A B vertical
C C C
A A B branch so it
is removed
CAB = 2C
 
(iii) C C C
A B
C
C C A B
C
A B A B
C A C B + –

CAB = 3C

(iv) C C C C C
A B
  C C

C C C
A B A B A B
Parallel
A B C + C = 2C
C A C + –

Parallel Series
2C/3 2C 5C C
C   2C  C 2C
3 3 
2C 3
2C  C
C C
A B A B

+ – + –
Electrostatics 121

Hence equivalent capacitance


5C
between A and B is
3

(v) Since there is no capacitor in the path APB, the points A, P and B are electrically same i.e., the input

and output points are directly connected (short circuited).


C C

Thus, entire charge will prefer to flow along path APB. It means
C 
that the capacitors connected in the circuit will not receive any
A B

charge for storing. Thus equivalent capacitance of this circuit is


P
zero.

(3) Wheatstone bride based circuit: If in a network five capacitors are arranged as shown in following
C1 C 3
figure, the network is called wheatstone bridge type circuit. If it is balanced then  hence C5 is
C2 C4

removed and equivalent capacitance between A and B

(i) (ii) (iii) C1


D
D
C1 C2 C5 C2
C1 C2 E
A C3 D B
C5 C5
A C5 A B
B
C3 C4
C3 C4 C4
E E

C1 C 2 C3C4
C AB  
C1  C 2 C 3  C 4

(4) Extended wheatstone bridge : The given figure consists of two wheatstone bridge connected together.

One bridge is connected between points AEGHFA and the other is connected between points EGBHFE.

This problem is known as extended wheatstone bridge problem, it has two branches EF and GH to the

left and right of which symmetry in the ratio of capacities can be seen.
122 Electrostatics

It can be seen that ratio of capacitances in branches AE and EG is same as that between the

capacitances of the branches AF and FH. Thus, in the bridge AEGHFA; the branch EF can be removed.

Similarly in the bridge EGBHFE branch GH can be removed

C C C C C C
E G E G

C C 
A B A B

F H C F H C
C C C C
2C
C AB 
3

(5) Infinite chain of capacitors : In the following figure equivalent capacitance between A and B

(i)
A
C1 C1 C1 Suppose the effective capacitance between A and B is

C2 C2 C2 CR. Since the network is infinite, even if we remove one

B pair of capacitors from the chain, remaining network

would still have infinite pair of capacitors, i.e., effective

capacitance between X and Y would also be CR

C1 X C1
A A Series
Parallel C1 (C 2  C R )
CR C2 (C2 + CR)  CR (C2 + CR) C1  C 2  C R
CR

B B
Y

Hence equivalent capacitance between A and B

C 1 (C 2  C R ) C2   C  
C AB   CR  C AB   1  4 1   1 
C1  C 2  C R 2   C2  

(ii) For what value of C0 in the circuit shown below will the net effective capacitance between A and B be
Electrostatics 123

independent of the number of sections in the chain

Suppose there are n sections between A and B

and the network is terminated by C0 with


A C1 C1 C1 C1 C
equivalent capacitance CR. Now if we add one

C2 C2 C2 C2 C0 more sections to the network between D and C

(as shown in the following figure), the equivalent


B C1  D
A C capacitance of the network CR will be
Parallel
C2 + C0 independent of number of sections if the
CR C0 C2 C0
capacitance between D and C still remains C0

B D
i.e.,

C 1  (C 2  C 0 ) 2
Hence C 0   C 0  C 2 C 0  C1 C 2  0
C1  C 2  C 0

C2   C  
On simplification C 0   1  4 1   1 
2   C2  

(6) Network with more than one cell :

(i) E1 E1 – E2

Potential difference across C1 is



C1 C2 C1 C2  C2 
  (E1  E 2 ) and potential difference across
 C1  C 2 
E2  C1 
C2 is   (E1  E 2 )
 C1  C 2 
(ii) Potential difference between the ends of this arrangement is given V. (E – V)
C1 E C2
+ –
C1 C2 A B
E 

A B
+ –
V C1 C2

(7) Advance case of compound dielectrics: If several dielectric medium filled between the plates of a
parallel plate capacitor in different ways as shown.
124 Electrostatics

(i) The system can be assumed to be made up of two capacitors C1 and C2 which
1 2
may be said to connected in series

A K1 K2 K1 0 A K  A 1 1 1  2K1 K 2  0 A
C1  , C 2  2 0 and    C eq    .
d d C eq C1 C 2  K1  K 2  d
2 2
d/ d/ 2 K1 K 2
2 2
Also K eq 
K1  K 2

(ii) In this case these two capacitors are in parallel and


1
K1 0 A K  A
A/2 C1  , C2  2 0
K1 2d 2d
K2  K  K2  0 A
A/2 Hence, C eq  C 1  C 2  C eq   1 .
2  2  d
d K  K2
Also K eq  1
2
(iii) In this case C1 and C2 are in series while this combination is in parallel with C3
C3 A A A
K1 0 K1 0 K 3 0
A/2
K3 C1 
K  A
2  1 0 , C  K  A
2  2 0 and C  2  K 3 0 A
2 3
d d d d d 2d
K1 K2 2 2 2
A/2
C1 C22 k 1 0 A k 2  0 A
C1 C 2 
d/ d/ d d k  A  k k k   A
Hence, C eq   C3   3 0  C eq   1 2  3  . 0
2 2 C1  C 2 k 1 0 A k 2  0 A 2d  k1  k 2 2  d

d d
k k k 
Also k eq   3  1 2 
 2 k1  k 2 
Electrostatics 125

Example based on series and parallel grouping of capacitors

Example: 117 Three capacitors of 2 f, 3 f and 6 f are joined in series and the combination is charged by means of a
24 volt battery. The potential difference between the plates of the 6 f capacitor is

(a) 4 volts (b) 6 volts (c) 8 volts (d) 10 volts

1 1 1 1
Solution: (a)Equivalent capacitance of the network is   
C eq 2 3 6
2F 3F 6F
C eq  1 F
Q
Charge supplied by battery Q = Ceq.V  1 × 24 = 24 C
+ –
24 24V
Hence potential difference across 6 F capacitor   4 volt .
6

Example: 118 Two capacitors each of 1  f capacitance are connected in parallel and are then charged by 200 V D.C.
supply. The total energy of their charges in joules is

(a) 0.01 (b) 0.02 (c) 0.04 (d) 0.06

1
Solution: (c)By using formula U  C eq V 2 1F
2
Here C eq  2 F
1F
1
 U   2  10 6  (200 ) 2
2
+ –
200V
 0 . 04 J
Example: 119 Five capacitors are connected as shown in the figure. The equivalent capacitance between the point A
and B is

2f
A

1f 2f
1f

B
2f

(a) 1  f (b) 2  f (c) 3  f (d) 4  f

Solution: (b)
A Parallel
1 + 1 = 2F
 1f 1f
1f

B
2f
126 Electrostatics

2f
Series
A
2
+  1 F
– 1f 2f 2
1f

B
2f 

A Parallel A Series
1 + 1 = 2F  2
1f 2f  1 F
1f 1f 2

B B
2f

Hence equivalent capacitance between A and B is 2F.

Example: 120 In the following network potential difference across capacitance of 4.5 F is

3F
4.5F

6F

+ –
12 V

(a) 8 V (b) 4 V (c) 2 V (d) 6 V


Parallel
9  4 .5
Solution: (a)Equivalent capacitance C eq   3 F 3 + 6 = 9F
9  4 .5 3F
4.5F
Charge supplied by battery Q = Ceq  V = 3 × 12 = 36 C

36 6F
Hence potential difference across 4.5 F  = 8V.
4 .5
+ –
12 V

Example: 121 A parallel plate capacitor of area A, plate separation d and capacitance C is filled with three different
dielectric materials having dielectric constants K1, K2 and K3 as shown in fig. If a single dielectric material
is to be used to have the same capacitance C in this capacitor, then its dielectric constant K is given by
Electrostatics 127

A/2 A/2

K1 K2 d/2
d
K3

A
A = Area of plates

1 1 1 1 1 1 1
(a)    (b)  
K K1 K 2 2 K 3 K K1  K 2 2 K 3

K1 K 2
(c) K   2K 3 (d) K  K 1  K 2  2K 3
K1  K 2

1 d  1 1 
Solution: (b)The effective capacitance is given by    
C eq 0 A  2 K 3 (K 1  K 2 ) 
K 0 A
The capacitance of capacitor with single dielectric of dielectric constant K is C 
d
0 A K 0 A
According to question C eq  C i.e., 
 1 1  d
d  
2
 3 K K 1  K 2 

1 1 1
   .
K 2 K 3 K1  K 2

Example: 122 Two capacitors C1 = 2F and C2 = 6F in series, are connected in parallel to a third capacitor C3 =
4F. This arrangement is then connected to a battery of e.m.f. = 2 V, as shown in the fig. How much
energy is lost by the battery in charging the capacitors ?

C1 C2

C3

+ –
2V

 32   16 
(a) 22  10 6 J (b) 11  10 6 J (c)    10 6 J (d)    10 6 J
 3   3 
C1 C 2 26
Solution: (b)Equivalent capacitance C eq   C3   4  5 . 5 F
C1  C 2 8
128 Electrostatics

1 1
 U C eq .V 2   5 . 5  (2) 2  11  10 6 J
2 2
Example: 123 In the circuit shown in the figure, each capacitor has a capacity of 3F. The equivalent capacity between

A and B is
A B

3
(a) F (b) 3 F (c) 6 F (d) 5 F
4
3F
A B 3F
Solution: (d)  3F 
A B A
A B B
3F A B
Parallel 3F 6F
B
3F
3 + 3 = 6F

36
Hence equivalent capacitance C eq   3  5 F.
36

Example: 124 Given a number of capacitors labelled as 8F, 250 V. Find the minimum number of capacitors needed to
get an arrangement equivalent to 16 F, 1000 V

(a) 4 (b) 16 (c) 32 (d) 64

Solution: (c)Let C = 8 F, C = 16 F and V = 250 volt, V = 1000 V

Suppose m rows of given capacitors are connected in parallel which each row contains n capacitor then
V' mC
Potential difference across each capacitors V  and equivalent capacitance of network C '  .
n n
On putting the values, we get n = 4 and m = 8. Hence total capacitors = m × n = 8 × 4 = 32.
2 2
C'  V '  16  1000 
Short Trick : For such type of problem number of capacitors n     . Here n     32
C V 8  250 
Example: 125 Ten capacitors are joined in parallel and charged with a battery up to a potential V. They are then

disconnected from battery and joined again in series then the potential of this combination will be[RPET 2000]

(a) V (b) 10 V (c) 5 V (d) 2 V

Solution: (b)By using the formula V '  nV  V '  10 V .

Example: 126 For the circuit shown, which of the following statements is true

S1 V1 = 30V V2 = 20V S2
+ – + –
S3
C1 = 2pF C2 = 3pF
Electrostatics 129

(a) With S1 closed, V1 = 15 V, V2 = 20 V (b) With S3 closed, V1 = V2 = 25 V

(c) With S1 and S2 closed V1 = V2 = 0 (d) With S1 and S3 closed V1 = 30 V, V2 = 20 V

Solution: (d)When S3 is closed, due to attraction with opposite charge, no flow of charge takes place through S3.
Therefore, potential difference across capacitor plates remains unchanged or V1 = 30 V and V2 = 20 V.

Alternate Solution

Charges on the capacitors are – q 1  (30 ) (2)  60 pC , q 2  (20 ) (3)  60 pC or q 1  q 2  q (say )

q q
+ – + –
2PF 3PF q = 60pC q = 60pC
 + – + –
The situation is similar as the two capacitors in series are
V1 = 30V V2 = 20V
first charged with a battery of emf 50 V and then
+ –
disconnected. 50V

When S3 is closed, V1 = 30 V and V2 = 20 V.

Example: 127 A finite ladder is constructed by connecting several sections of 2  F,4  F capacitor combinations as

shown in the figure. It is terminated by a capacitor of capacitance C. What value should be chosen for C,

such that the equivalent capacitance of the ladder between the points A and B becomes independent of

the number of sections in between

4F 4F 4F


A

2F 2F 2F C

(a) 4  F (b) 2  F (c) 18  F (d) 6  F

C2  C   C  4 F
Solution: (a)By using formula C  1  4 1
C   1 ; 1 We get C  4  F.
2   2   C 2  2 F
 
Example: 128 Figure shows two capacitors connected in series and joined to a battery. The graph shows the variation in

potential as one moves from left to right on the branch containing the capacitors.
130 Electrostatics

+ –

(a) C 1  C 2
V

(b) C 1  C 2

C1 C2 X
(c) C 1  C 2

(d) The information is insufficient to decide the relation between C 1 and C 2

Solution: (c)According to graph we can say that potential difference across the capacitor C 1 is more than that across C 2 .
C1 V
Since charge Q is same i.e., Q  C 1 V1  C 2 V2   2  C1  C 2
C2 V1
(V1  V2 ).
Example: 129 Two condensers of capacity C and 2C are connected in parallel and these are charged upto V volt. If the
battery is removed and dielectric medium of constant K is put between the plates of first condenser, then
the potential at each condenser is
V K 2V 3V
(a) (b) 2  (c) (d)
K2 3V K2 K2
Solution: (d)Initially C

2C Equivalent capacitance of the system C eq  3 C

Q Total charge Q  (3 C )V
+ –
V

Finally Equivalent capacitance of the system


KC
C eq  KC  2 C

2C

Q 3 CV 3V
Hence common potential V    .
(KC  2 C ) (K  2)C K  2

Example: 130 Condenser A has a capacity of 15 F when it is filled with a medium of dielectric constant 15. Another
condenser B has a capacity 1 F with air between the plates. Both are charged separately by a battery of
100V. after charging, both are connected in parallel without the battery and the dielectric material being
removed. The common potential now is

(a) 400V (b) 800V (c) 1200V (d) 1600V

Solution: (b)Charge on capacitor A is given by Q 1  15  10 6  100  15  10 4 C

Charge on capacitor B is given by Q 2  1  10 6  100  10 4 C


Electrostatics 131

15  10 6
Capacity of capacitor A after removing dielectric   1 F
15
Now when both capacitors are connected in parallel their equivalent capacitance will be Ceq
 1  1  2 F

(15  10 4 )  (1  10 4 )
So common potential   800 V .
2  10 6
Example: 131 A capacitor of 20 F is charged upto 500V is connected in parallel with another capacitor of 10 F which
is charged upto 200V. The common potential is

(a) 500V (b) 400V (c) 300V (d) 200V


C 1 V1  C 2 V2
Solution: (b)By using V  ; C1 = 20 F, V1 = 500 V, C2 = 10 F and V2 = 200 V
C 1 V2
20  500  10  200
V   400 V .
20  10
Example: 132 In the circuit shown

12 V
+ –

C1 = 4F C2 = 8F

+ –

6V

(a) The charge on C2 is greater than that of C1 (b) The charge on C2 is smaller than that of C1

(c) The potential drop across C1 is smaller than C2 (d) The potential drop across C1 is greater than C2

Solution: (d)Given circuit can be redrawn as follows 12 – 6 = 6 V


+ –
4 8 8
C eq   F
12 3 V1 V2
8
So Q   6  16 C C1 = 4F C2 = 8F
3
16 16
Hence potential difference V1   4 volt and V2   2 volt i.e. V1 > V2
4 8

Example: 133 As shown in the figure two identical capacitors are connected to a battery of V volts in parallel. When

capacitors are fully charged, their stored energy is U 1 . If the key K is opened and a material of dielectric
U1
constant K  3 is inserted in each capacitor, their stored energy is now U 2 . will be
U2
132 Electrostatics

+
V C C
A B

1
(a) 3 (b) 5 (c) 3 (d)
5 3 3

Solution: (a)Initially potential difference across both the capacitor is same hence energy of the system is
1 2 1 2 2 ……..(i)
U1  CV  CV  CV
2 2

In the second case when key K is opened and dielectric medium is filled between the plates, capacitance
of both the capacitors becomes 3C, while potential difference across A is V and potential difference
V
across B is hence energy of the system now is
3
2
1 1 V  10
U2  (3 C )V 2
 (3 C )    CV 2 …….(ii)
2 2 3 6

So, U 1  3
U2 5

Example: 134 In the following figure the resultant capacitance between A and B is 1 F . The capacitance C is

C 1F
A

8F 6F 4F

12F

2F 2F

32 11 23 32
(a) F (b) F (c) F (d) F
11 32 32 23

Solution: (d)Given network can be simplified as follows


1F C 1F
C
A A
4F 8F 4F 4F
8F 6F 
Parallel
Series
2 + 2 = 4F
12F 6  12 Parallel
 4 F 4F 4  4  8 F
2F 2F 6  12
Series
B B
84 8
 F
84 3
Electrostatics 133


C C 1F
Parallel Series
A 8 8 32 A 18 8
  F  F
3 9 9  8 18 9
8 8
F F F 8F
3 9 3

B B


A B
C 32
F
9

Given that equivalent capacitance between A and B i.e., CAB  1 F


32 32
C C
But C AB  9 hence 9  1  C  32 F.
32 32 23
C C
9 9
Example: 135 A 1 F capacitor and a 2 F capacitor are connected in parallel across a 1200 volts line. The charged
capacitors are then disconnected from the line and from each other. These two capacitors are now
connected to each other in parallel with terminals of unlike signs together. The charges on the capacitors
will now be

(a) 1800 C each (b) 400 C and 800 C (c) 800 C and 400 C (d) 800 C and 800 C

Solution: (b)Initially charge on capacitors can be calculated as follows

Q1 + – 1F Q1 = 1 × 1200 = 1200 C and Q2 = 2 × 1200 = 2400 C


C1

Q2
Finally when battery is disconnected and unlike plates are
+ – 2F
C2 Q 2  Q1
connected together then common potential V' 
+ – 1F C1  C 2
+ – C
+ – 2400  1200 + – 1
1200 V   400 V
12 – + 2F
– + C
– + 2

V
134 Electrostatics

Hence, New charge on C 1 is 1  400  400 C

And New charge on C 2 is 2  400  800 C.

Example: 136 The two condensers of capacitances 2 F and 3 F are in series. The outer plate of the first condenser is

at 1000 volts and the outer plate of the second condenser is earthed. The potential of the inner plate of

each condenser is

(a) 300 volts (b) 500 volts (c) 600 volts (d) 400 volts

Solution: (d)Here, potential difference across the combination is V A  V B  1000 V


23 6 2F 3F
Equivalent capacitance C eq   F 0V
23 5 + 1000 V B
A C
6
Hence, charge on each capacitor will be Q  C eq  (V A  V B )   1000  1200 C
5
1200
So potential difference between A and C, V A  VC   600 V  1000  VC  600  Vc  400 V
2
Example: 137 Four identical capacitors are connected in series with a 10V battery as shown in the figure. The point N is
earthed. The potentials of points A and B are

+ –
10V

N
A B
C CA C C B

(a) 10 V ,0 V (b) 7 . 5 V  2 . 5 V (c) 5 V  5 V (d) 7 . 5 V , 2 . 5 V


10
Solution: (b)Potential difference across each capacitor will be  2 .5 V
4
Hence potential difference between A & N i.e., V A  V N  2 . 5  2 . 5  2 . 5  7 . 5 V

 V A  0  V A  7 . 5 V While V N  V B  2 . 5  0  VB  2 .5  V B  2 . 5 V

Example: 138 In the figure below, what is the potential difference between the points A and B and between B and C
respectively in steady state
Electrostatics 135
3F 1F
B

3F 1F

1F

C
0
A
100 V 2
(a) 100 volts both 0 (b) V AB  
75 volts, V BC  25 volts

(c) V AB  25 volts, V BC  75 volts (d) V AB  50 volts V BC  50 volts



Solution: (c)In steady state No current flows in the given circuit hence resistances can be eliminated

Parallel Parallel
3+3= 6F 1+1= 2F
3F 1F 6F 2F C
B B B A B
A C Line (1)
3F 1F V1=VAB V2=VBC

A C  Line (2)
1F 1F

A C A C
+ –
100 V 100 V

By using the formula to find potential difference in series combination of two capacitor
  C2  C1 
 V1   .V and V2  V
 C C C 2  C 2 
  1 2 
 2   6 
V1  V AB     100  25 V ; V2  V BC     100  75 V .
26 26
Example: 139 A capacitor of capacitance 5 F is connected as shown in the figure. The internal resistance of the cell is
0.5. The amount of charge on the capacitor plate is

1 1

5F 2

2.5 V
+ –

(a) 0 C (b) 5 C (c) 10 C (d) 25 C


136 Electrostatics

2 .5
Solution: (c)In steady state current drawn from the battery i   1A
(1  1  0 . 5 )
1 1
Line (1)
In steady state capacitor is fully charged hence No current will 5F 2
Line (2)
flow through line (2)
i 0.5 
Hence potential difference across line (1) is V  1  2  2volt ,
the same potential difference appears across the capacitor, so
2

charge on capacitor Q  5  2  10 C
.
5
Example: 140 When the key K is pressed at time t  0 . Which of the following statements about the current i in the
resistor AB of the adjoining circuit is true
V
2V
A 1000  B
K
1000 
C=1F

(a) i  2mA at all t (b) i oscillates between 1mA and 2mA

(c) i = 1mA at all t (d) At t = 0, i = 2mA and with time it goes to 1mA

Solution: (d)At t  0 whole current passes through capacitance; so effective resistance of circuit is 1000  and current
2
i  2  10  3 A  2mA . After sufficient time, steady state is reached; then there is no current in
1000
capacitor branch; so effective resistance of circuit is 1000  1000  2000  and current
2
i  1  10  3 A  1mA i.e., current is 2mA at t  0 and with time it goes to 1mA .
2000
Example: 141 The plates of a capacitor are charged to a potential difference of 320 volts and are then connected across
a resistor. The potential difference across the capacitor decays exponentially with time. After 1 second
the potential difference between the plates of the capacitor is 240 volts, then after 2 and 3 seconds the
potential difference between the plates will be

(a) 200 and 180 volts (b) 180 and 135 volts (c) 160 and 80 volts (d) 140 and 20 volts

Solution: (b)During discharging potential difference across the capacitor falls exponentially as V  V0 e   t ( = 1/RC)

Where V = Instantaneous P.D. and V0  max. P.D. across capacitor


3
After 1 second V1 = 320 (e–)  240 = 320 (e–)  e  
4
Electrostatics 137

2
3
After 2 seconds V2 = 320 (e–)2  320     180 volt
4
3
3
After 3 seconds V3 = 320 (e–)3 = 320     135 volt
4

Example: 142 Five similar condenser plates, each of area A. are placed at equal distance d apart and are connected to
a source of e.m.f E as shown in the following diagram. The charge on the plates 1 and 4 will be


1 2 3 4 5 V
+

 0 A 2 0 A  0 AV 2 0 AV  0 AV 3 0 AV  0 AV 4  0 AV


(a) , (b) , (c) , (d) ,
d d d d d d d d

Solution: (b)Here five plates are given, even number of plates are connected together while odd number of plates are

connected together so, four capacitors are formed and they are in parallel combination, hence redrawing

the figure as shown below.

Capacitance of each
1 + – 2
+ –
0 A + –
Capacitor is C 
d 3 + – 2
+ –
+ –
Potential difference across each capacitor is V
3 + – 4
+ –
 A + –
So charge on each capacitor Q  0 V
d + –
+ –
5 + – 4
 0 AV
Charge on plate (1) is  + –
d V

 0 AV 2 AV
While charge on plate 4 is  2   0 .
d d

Example: 143 Four plates are arranged as shown in the diagram. If area of each plate is A and the distance between

two neighbouring parallel plates is d, then the capacitance of this system between A and B will be

A B

d
138 Electrostatics

4 0 A 3 0 A 2 0 A 0 A
(a) (b) (c) (d)
d d d d

Solution: (c)To solve such type of problem following guidelines should be follows
1

2
A B
3
Guideline 1. Mark the number (1,2,3……..) on the plates 4

1 2
A B

3 2

Guideline 2. Rearrange the diagram as shown below


4 3

Guideline 3. Since middle capacitor having plates 2, 3 is short circuited so it should be eliminated from
the circuit
1 2
A B

4 3
 A
Hence equivalent capacitance between A and B C AB 2 0
d

Tricky example: 17

A capacitor of capacitance C1 = 1F can withstand maximum voltage V1 = 6 KV (kilo-volt) and


another capacitor of capacitance C2 = 3F can withstand maximum voltage V2 = 4KV. When the
two capacitors are connected in series, the combined system can withstand a maximum voltage of

(a) 4 KV (b) 6 KV (c) 8 KV (d) 10 KV

Solution: (c) We know Q = CV

Hence (Q1)max = 6 mC while (Q2)max = 12 mC

However in series charge is same so maximum charge on C2 will also be 6 mC (and not 12 mC) and
6 mC
hence potential difference across C2 will be V2   2 KV and as in series V = V1 + V2
3 F

So Vmax  6 KV  2 KV  8 KV
Electrostatics 139

+

+

– +


+

Assignment

Charge, Coulombs law, Electric field

Basic Level


1. An electron is moving round the nucleus of a hydrogen atom in a circular orbit of radius r. The coulomb force F between the two
1
is (Where K  )
4  0

e2 e2  e  e2
(a)  K rˆ (b) K r (c) K r (d) K rˆ
r3 r3 r3 r2

2. Two point charges +2C and +6C repel each other with a force of 12 Newtons. If a charge of – 4C is given to each of these
charges the force now is

(a) Zero (b) 4 N (attractive) (c) 12 N (attractive) (d) 8 N (repulsive)

volt
3. Electric field intensity at a point at a distance 60 cm from charge is 2 then charge will be
metre

(a) 8  10 11 C (b) 8  10 11 C (c) 4  10 11 C (d) 4  10 11 C

4. In a uniformly charged spherical shell of radius r the electric field is

(a) Zero (b) Non-zero constant (c) Varies with r (d) Inversely varies with r

5. If  is the charge per unit area on the surface of a conductor, then the electric field intensity at a point on the surface is

    
(a)   normal to surface
 (b)   normal to surface

 0   2 0 
    
(c)   tangential to surface
 (d)   tangential to surface

 0   2 0 

6. Electric field intensity at a point in between two parallel sheets with like charges of same surface charge densities () is

  2
(a) (b) (c) Zero (d)
2 0 0 0
140 Electrostatics

7. The electric field due to cylindrical charge distribution of infinite length at a distance equal to its radius from its surface will be –
(  linear charge density, R = radius of the cylinder)
2 K K K 3 K
(a) (b) (c) (d)
R R 2R 2R

8. There is a solid dielectric sphere of radius ‘ R’ having uniformly distributed charge. What is the relation between electric field ‘ E’
inside the sphere and radius of sphere ‘R’ is

1
(a) E  R 2 (b) E  R 1 (c) E (d) E  R2
R3

9. Electric field strength due to a point charge of 5 C at a distance of 80 cm from the charge is

(a) 8  104 N/C (b) 7  104 N/C (c) 5  104 N/C (d) 4  104 N/C

10. One metallic sphere A is given positive charge where as another identical metallic sphere B of exactly same mass as of A is
given equal amount of negative charge. Then

(a) Mass of A and mass of B still remain equal (b) Mass of A increases

(c) Mass of B decreases (d) Mass of B increases

11. When 1019 electrons are removed from a neutral metal plate, the electric charge on it is

(a) – 1.6 C (b) + 1.6 C (c) 10+19 C (d) 10–19 C

12. When air is replaced by a dielectric medium of constant k, the maximum force of attraction between two charges separated by a
distance

(a) Decreases k times (b) Remains unchanged (c) Increases k times (d) Increases k–1 times

13. Two infinite plane parallel sheets separated by a distance d have equal and opposite uniform charge densities . Electric field at
a point between the sheets is

(a) Zero (b)
0


(c) (d) Depend on the nature of the materials of the spheres
2 0

14. A hollow insulated conduction sphere is given a positive charge of 10 C. What will be the electric field at the centre of the
sphere if its radius is 2 metres

(a) Zero (b) 5C m–2 (c) 20 C m–2 (d) 8 C m–2

15. A body can be negatively charged by

(a) Giving excess of electrons to it (b) Removing some electrons from it

(c) Giving some protons to it (d) Removing some neutrons from it

16. Three equal charges are placed on the three corners of a square. If the force between Q 1 and Q 2 is F12 and that between Q 1
F12
and Q 3 is F13 , then the ratio of magnitudes
F13

(a) 1/2 (b) 2 (c) 1 2 (d) 2


Electrostatics 141

17. The magnitude of electric field E in the annular region of a charged cylindrical capacitor

(a) Is same throughout (b) Is higher near the outer cylinder than near the inner
cylinder

(c) Varies as 1/r, where r is the distance from the axis (d) Varies as 1/r2, where r is the distance from the axis

18. A glass rod rubbed with silk is used to charge a gold leaf electroscope then charged electroscope is exposed to X-rays for a
short period. Then

(a) The divergence of leaves will not be affected (b) The leaves will diverge further

(c) The leaves will collapse (d) The leaves will melt

19. A cube of side b has a charge q at each of its vertices. The electric field due to this charge distribution at the center of this cube
will be

(a) q/b2 (b) q/2b2 (c) 32q/b2 (d) Zero

20. The intensity of electric field, due to a uniformly charged infinite cylinder of radius R, at a distance r(> R) from its axis is
proportional to
1 1
(a) r2 (b) r3 (c) (d)
r r2

21. Two parallel plates have equal and opposite charge. When the space between them is evacuated, the electric field between the
plates is 2  105 V/m. When the space is filled with dielectric, the electric field becomes 1  105 V/m. The dielectric constant of
the dielectric material

(a) 1/2 (b) 1 (c) 2 (d) 3

Charge, Coulombs law, Electric field

Advance Level

22. Six charges, three positive and ;three negative of equal magnitude are to be placed at the vertices of a regular hexagon such

that the electric field at O is double the electric field when only one positive charge of same magnitude is placed at R. Which of

the following arrangements of charges is possible for P, Q, R, S, T and U respectively ?

P Q

U R
O

T S

(a) +, –, +, –, –, + (b) +, –, +, –, +, – (c) +, +, –, +, –, – (d) –, +, +, –, +, –


142 Electrostatics

23. Three charges – q1, + q2 and – q3 are placed as shown in the figure. The X-component of the force on – q1 is proportional to

– q3 Y


a
b
– q1 X
+q2

(a) q2 / b2 – (q3 / a2) sin (b) q2 / b2 – (q3 / a2) cos (c) q2 / b2 + (q3 / a2) sin (d) q2 / b2 + (q3 / a2) cos

24. Two particle of equal mass m and charge q are placed at a distance of 16 cm. They do not experience any force. The value of
q
is
m

 0 G
(a) 40G (b) (c) (d) 4 0 G
G 4 0

25. A solid conducting sphere of radius a has a net positive charge 2 Q . A conducting spherical shell of inner radius b and outer
radius c is concentric with the solid sphere and has a net charge Q . The surface charge density on the inner and outer
surfaces of the spherical shell will be

2Q Q Q Q Q
(a)  , (b)  , (c) 0, (d) None of the above
4 b 2 4 c 2 4 b 2 4 c 2 4 c 2

26. Two conducting solid spheres of radii R and 2R are given equal charges (+Q) each. When they are connected by a thin
conducting wire, the charges get redistributed. The ratio of charge Q1 on smaller sphere to charge Q2 on larger sphere becomes
Q1 Q1 Q1 1
(a) 1 (b) 2 (c)  (d) None of these
Q2 Q2 Q2 2

27. Electric charges of 1 C, – 1 C and 2 C are placed in air at the corners A, B and C respectively of an equilateral triangle ABC
 
having length of each side 10 cm. The resultant force on the charge at C is  0  10 7 Hm 1 
 4  
(a) 0.9 N (b) 1.8 N (c) 2.7 N (d) 3.6 N

28. A solid metallic sphere has a charge + 3Q. Concentric with this sphere is a conducting spherical shell having charge – Q. The
radius of the sphere is a and that of the spherical shell is b(b > a). What is the electric field at a distance R(a  R  b ) from the
centre
Electrostatics 143

Q 3Q 3Q 4Q
(a) (b) (c) (d)
2 0 R 2 0 R 4  0 R 2 4  0 R 2

29. Two copper balls, each weighing 10 g are kept in air 10 cm apart. If one electron from every 10 6 atoms is transferred from one
ball to the other, the coulomb force between them is (atomic weight of copper is 63.5)

(a) 2.0  1010 N (b) 2.0  104 N (c) 2.0  107 N (d) 2.0  106 N

30. A non-conducting solid sphere of radius R is uniformly charged. The magnitude of the electric field due to the sphere at a
distance r from its centre

(a) Increases as r increases for r < R (b) Decreases as r increases for 0 < r < 
(c) Decreases as r increases for R < r <  (d) In discontinuous at r = R

31. Two infinitely long parallel wires having linear charge densities 1 and 2 respectively are placed at a distance of R metres. The
 1 
force per unit length on either wire will be  k  

 4  0 
2 12 2 12 12 12
(a) k (b) k (c) k (d) k
R2 R R2 R

32. A point charge of 40 stat coulomb is placed 2 cm in front of an earthed metallic plane plate of large size. Then the force of
attraction on the point charge is

(a) 100 dynes (b) 160 dynes (c) 1600 dynes (d) 400 dynes

33. Two point charges are kept separated by 4 cm of air and 6 cm of a dielectric of relative permittivity 4. The equivalent dielectric
separation between them so far their coulombian interaction is conserved is

(a) 10 cm (b) 8 cm (c) 5 cm (d) 16 cm

34. A regular polygon has n sides each of length l. Each corner of the polygon is at a distance r from the centre. Identical charges
each equal to q are placed at (n – 1) corners of the polygon. What is the electric field at the centre of the polygon
n q n q 1 q 1 q
(a) (b) (c) (d)
4  0 r 2 4  0 l 2 4  0 r 2 4  0 l 2

35. Two spheres A and B of gold (each of mass 1 kg.) are hung from two pans of a sensitive physical balance. If A is given 1
Faraday of positive charge and B is given 1 F of negative charge, then to balance the balance we have to put a weight of (1 F =
96500 C)

(a) 0.6  g on the pan of A (b) 0.6  g on the pan of B

(c) 1.01 milligram on the pan of A (d) 1.2 milligram on the pan of B

36. A long thin rod lies along the x-axis with one end at the origin. It has a uniform charge density  C/m. Assuming it to infinite in
length the electric field point x = – a on the x-axis will
   2
(a) (b) (c) (d)
 0 a 2 0 a 4 0 a  0 a

37. The charge on 500 cc of water due to protons will be


(a) 6.0  1027 C (b) 2.67  107 C (c) 6  1023 C (d) 1.67  1023 C
144 Electrostatics

38. In the figure shown, if the linear charge density is , then the net electric field at O will be


+ + + + + + + + + + +
+
R
+
O
+
+ + + + + + + + + + +

k 2k 2 k
(a) Zero (b) (c) (d)
R R R

39. A positively charged ball is supported on a rigid insulating stand. We wish to measure the electric field E at a point in the some

horizontal level as that of the hanging charge. To do so we put a positive test charge q0 and measure F/q0 than E at that point

(a) > F/q0 (b) = F/q0 (c) < F/q0 (d) Cannot be estimated

40. Two point charges placed at a distances of 20 cm in air repel each other with a certain force. When a dielectric slab of thic kness

8 cm and dielectric constant K is introduced between these point charges, force of interaction becomes half of it’s previous

value. Then K is approximately

(a) 2 (b) 4 (c) 2 (d) 1

41. A conducting sphere of radius R, and carrying a charge q is joined to a conducting sphere of radius 2 R, and carrying a charge –

2q. The charge flowing between them will be

q 2q 4q
(a) (b) (c) q (d)
3 3 3

Potential, E = – dV/dr, Electric Lines and Work done

Basic Level

42. A hollow conducting sphere is placed in an electric field produced by a point charge placed at P as shown in figure. Let

VA , VB , VC be the potentials at points A, B and C respectively. Then

A
C P

B
Electrostatics 145

(a) VC  VB (b) VB  VC (c) VA  VB (d) VA  VC

43. A thin spherical conducting shell of radius R has a charge q. Another charge Q is placed at the centre of the shell. The
R
electrostatic potential at a point P a distance from the centre of the shell is
2

(q  Q) 2 2Q 2Q 2q 2Q q
(a) (b) (c)  (d) 
4  0 R 4 0 R 4  0 R 4  0 R 4  0 R 4  0 R

44. A charged oil drop is to be held stationary between two plates separated by a distance of 25 mm . If the mass of the drop is

5  10 15 kg and the charge on it is 10 18 C , the potential to be applied between the two plates is ( g  10 ms 2 )

(a) 125 V (b) 1250 V (c) 2500 V (d) 450 V

45. A hollow conducting sphere of radius R has a charge (+Q) on its surface. What is the electric potential within the sphere at a
R
distance r  from its centre
3
1 Q 1 Q 1 Q
(a) Zero (b) (c) (d)
4  0 r 4  0 R 4  0 r 2

46. A cube of a metal is given a positive charge Q. For the above system, which of the following statements is true

(a) Electric potential at the surface of the cube is zero (b) Electric potential within the cube is zero

(c) Electric field is normal to the surface of the cube (d) Electric field varies within the cube

47. Two spheres A and B of radius ‘a’ and ‘b’ respectively are at same electric potential. The ratio of the surface charge densities of

A and B is

a b a2 b2
(a) (b) (c) (d)
b a b2 a2

48. Electric potential at equatorial point of a small dipole with dipole moment p (At r, distance from the dipole) is

p p 2p
(a) Zero (b) (c) (d)
4  0 r 2 4  0 r 3 4  0 r 3

49. The radius of a soap bubble whose potential is 16 V is doubled. The new potential of the bubble will be

(a) 2 V (b) 4 V (c) 8 V (d) 16 V

50. A unit charge is taken from one point to another over an equipotential surface. Work done in this process will be

(a) Zero (b) Positive (c) Negative (d) Optimum

51. The displacement o a charge Q in the electric field E  e 1 i  e 2 j  e 3 k is r  ai  bj. The work done is
146 Electrostatics

(a) Q(ae 1  be 2 ) (b) Q (ae 1 ) 2  (be 2 ) 2 (c) Q(e 1  e 2 ) a 2  b 2 (d) Q( e 12  e 22 ) (a  b )

52. Two electric charges 12 C and – 6 C are placed 20 cm apart in air. There will be a point P on the line joining these charges

and outside the region between them, at which the electric potential is zero. The distance of P from – 6 C charge is

(a) 0.10 m (b) 0.15 m (c) 0.20 m (d) 0.25 m

53. Two charges of 4 C each are placed at the corners A and B of an equilateral triangle of side length 0.2 m in air. The electric

 1 N m2 
potential at C is   9  10 9 
 4  C2 
 0 

(a) 9  104 V (b) 18  104 V (c) 36  104 V (d) 36  104 V

54. The figure given below shows two parallel equipotential surfaces A and B kept at a small distance r from each other A point

charge of – q coul is taken from the surface A to B. The amount of net work W done will be given by

A r B

1 q 1  q  1  q 
(a) W     (b) W     (c) W    (d) Zero
4  0  r  4  0  r 2  4  0  r 2 

55. Two metal spheres of radii R1 and R2 are charged to the same potential. The ratio of charges on the spheres is

(a) R1 : R 2 (b) R1 : R2 (c) R 12 : R 22 (d) R 13 : R 23

56. Electric charges of + 10C, +5C, – 3C and + 8C are placed at the corners of a square of side 2 m . The potential at the
centre of the square is

(a) 1.8 V (b) 1.8  106 V (c) 1.8  105 V (d) 1.8  104 V

57. An electron enters between two horizontal plates separated by 2 mm and having a p.d. of 1000 V. The force on electron is

(a) 8  10– 12 N (b) 8  10–14 N (c) 8  109 N (d) 8  1014 N

58. Two unlike charges of magnitude q are separated by a distance 2d. The potential at a point midway between them is
Electrostatics 147

1 1 q 1 2q
(a) Zero (b) (c) . (d) .
4  0 4  0 d 4  0 d 2

59. A hollow metal sphere of radius 5 cm is charged such that the potential on its surface is 10 V. The potential at a distance of 2 cm

from the centre of the sphere

(a) Zero (b) 10 V (c) 4 V (d) 10/3 V

60. Below figures (1) and (2) represent lines of force. Which is correct statement

(1) (2)

(a) Figure (1) represents magnetic lines of force (b) Figure (2) represents magnetic lines of force

(c) Figure (1) represents electric line of force (d) Both (1) and (2) represent magnetic line of force

61. At a certain distance from a point charge the electric field is 500 V/m and the potential is 3000 V. What is this distance

(a) 6 m (b) 12 m (c) 36 m (d) 144 m

62. Two plates are 2 cm apart, a potential difference of 10 volt is applied between them, the electric field between the plates is

(a) 20 N/C (b) 500 N/C (c) 5 N/C (d) 250 N/C

10
63. Charges   10 9 are placed at each of the four corners of a square of side 8 cm. The potential at the intersection of the
3
diagonals is

(a) 150 2 volt (b) 1500 2 volt (c) 900 2 volt (d) 900 volt

64. Three charges 2q, – q, – q are located at the vertices of an equilateral triangle. At the centre of the triangle

(a) The field is zero but potential is non-zero (b) The field is non-zero but potential is zero

(c) Both field and potential are zero (d) Both field and potential are non-zero

65. The potential due to a infinite line charge XX at point A is 20 V and at point B is 50 V. Point A and C are situated on

equipotential surface then the work done in carrying an electron from

X A
+
+
+ B
+
+
+
C
X
148 Electrostatics

(a) A to B is 30 eV (b) B to C is 30 eV (c) A to C is – 30 eV (d) A to B and from B to C is 30

eV

Potential, E = – dV/dr, Electric Lines and Work done

Advance Level

66. A metallic shell has a point charge ‘q’ kept inside its cavity. Which one of the following diagrams correctly represents the electric

lines of forces

(a) (b) (c) (d)

67. Electric potential at any point is V  5 x  3 y  15 z , then the magnitude of the electric field is

(a) 3 2 (b) 4 2 (c) 5 2 (d) 7

68. Two concentric spheres of radii R and r have similar charges with equal surface densities ( ) . What is the electric potential at
their common centre ?

 R  
(a) (b) (c) ( R  r) (d) ( R  r)
0 r 0 0 0

69. A uniform electric field pointing in positive x-direction exists in a region. Let A be the origin, B be the point on the x-axis at x = +1
cm and C be the point on the y-axis at y = +1 cm. Then the potentials at the points A, B and C satisfy

(a) VA < VB (b) VA > VB (c) VA < VC (d) VA > VC

70. An ellipsoidal cavity is carved within a perfect conductor. A positive charge q is placed at the centre of the cavity. The points A

and B are on the cavity surface as shown in the figure. Then


A 
B
q
Electrostatics 149

(a) Electric field near A in the cavity = Electric field near B in the cavity (b) Charge density at A = Charge density at B

(c) Potential at A = Potential at B (d) Total electric field flux through the surface of the cavity

is q/0

71. The radius of a hollow metallic sphere is r. If the p.d. between its surface and a point at distance 3r from its centre is V, then the intensity

of electrical field at a distance of 3r from its centre will be

(a) V/6r (b) V/4r (c) V/3r (d) V/2r

72. In Millikan’s oil drop experiment an oil drop carrying a charge Q is held stationary by a potential difference 2400 V between the

plates. To keep a drop of half the radius stationary the potential difference had to be made 600 V. What is the charge on the

second drop

Q Q 3Q
(a) (b) (c) Q (d)
4 2 2

73. A non-conducting ring of radius 0.5 m carries a total charge of 1.11  10–10 C distributed non-uniformly on its circumference
l0
producing an electric field E everywhere in space. The value of the line integral
  E.dl (l = 0 being centre of the ring) in volt is
l

(a) + 2 (b) – 1 (c) – 2 (d) Zero

74. A sphere of radius r is placed concentrically inside a hollow sphere of radius R. The bigger and smaller spheres are given

charges Q and q respectively and are insulated. The potential difference between the two spheres depends on

(a) Only charge q (b) Only charge Q (c) Both q and Q (d) Not on q and Q

75. Four equal charges q are held fixed at (0, R), (0, – R), (R, R) and (R, – R) respectively of a (x, y) co-ordinate system. The work

done in moving a charge Q from point A (R, 0) to origin (0, 0) is

qQ 2 1 2 qQ 2 qQ 2 1
(a) Zero (b) (c) (d)
4  0 2R  0 R 4  0 2R

76. Consider a parallelogram ABCD, with angle at B is 120o. A charge + Q placed at corner A produces field E and potential V at

corner D. If we now added charges – 2Q and + Q at corners B and C respectively, the magnitude of field and potential at D will

become, respectively – 2Q

B
+Q A C +Q

D
150 Electrostatics

V E V
(a) E and 0 (b) 0 and V (c) E 2 and (d) and
2 2 2
100
77. The intensity of electric field in a region of space is represented by E  V / m . The potential difference between the points
x2
x = 10 m and x = 20 m will be

(a) 15 m (b) 10 V (c) 5 V (d) 1 V

78. Two points A and B lying on Y-axis at distances 12.3 cm and 12.5 cm from the origin. The potentials at these points are 56 V and
54.8 V respectively, then the component of force on a charge of 4 C placed at A along Y-axis will be

(a) 0.12 N (b) 48  10–3 N (c) 24  10–4 N (d) 96  10–2 N

79. When two uncharged metal balls of radius 0.09 mm each collide, one electron is transferred between them. The potential
difference between them would be

(a) 16 V (b) 16 pV (c) 32 V (d) 32 pV

80. An electric field of 100 Vm–1 exists along x-axis. The potential difference between a point A (– 1m, 0) and B(+3m, 0) is

(a) 200 V (b) – 200 V (c) 400 V (d) – 400 V

81. The potential in an electric field has the form V = a(x2 + y2 + z2). The modulus of the electric field at a point ( z, y, z) is

2a
(a) 2 a( x 2  y 2  z 2 ) 3 / 2 (b) 2 a x 2  y 2  z 2 (c) a x2  y2  z2 (d)
x  y2  z2
2

82. Electric potential is given by V  6 x  8 xy 2  8 y  6 yz  4 x 2 . Then electric force acting on 2 coulomb point charge placed on

origin will be

(a) 2N (b) 6N (c) 8N (d) 20N

83. An electric line of force in the xy plane is given by equation x2 + y2 = 1. A particle with unit positive charge, initially at rest at the

point x = 1, y = 0 in the xy plane

(a) Not move at all (b) Will move along straight line

(c) Will move along the circular line of force (d) Information is insufficient to draw any conclusion

84. An electric field E  50ˆi  75 ˆj N / C exists in a certain region of space. Presuming the potential at the origin to be zero, the

potential at point P (1 m, 2m) will be

(a) 100 V (b) – 100 V (c) 200 V (d) – 200 V

K 
85. Electric potential in an electric filed is given as V  , (K being constant), if position vector r  2ˆi  3 ˆj  6 kˆ , then electric field
r
will be

(a) 2ˆi  3ˆj  6kˆ  243


K
(b) 2ˆi  3ˆj  6kˆ  343
K
(c)
K
243

3ˆi  2 ˆj  6 kˆ  (d)
K
343

6ˆi  2 ˆj  3 kˆ 
Electrostatics 151

86. Two points are at distances a and b (a < b) from a long string of charge per unit length . The potential difference between the

points is proportional to

(a) b/a (b) b2/a2 (c) b/a (d) ln (b/a)

87. Some spherical equipotential surfaces are shown in the figure. The values of the potentials are 100 V, 80 V, 40 V on surfaces of

radii 10 cm, 12.5 cm and 25 cm. The electric field at a distance r from the common centre is

12.5 cm
40 V

10 cm 80 V

25 cm
100 cm

20 10 20 10
(a) (b) (c) (d)
r2 r3 r3 r2

88. An arc of radius r carries charge. The linear density of charge is  and the are subtends a angle at the centre. What is electric
3
potential at the centre
   
(a) (b) (c) (d)
4 0 8 0 12 0 16 0

89. A wire is bent in the form of a regular hexagon of side a and a total charge Q is distributed uniformly over it. One side of the
hexagon is removed. The electric field due to the remaining sides at the centre of the hexagon is
Q Q Q Q
(a) (b) (c) (d)
12 3 0 a 2
16 3 0 a 2
8 2 0 a 2
8 2 0 a 2

90. Some equipotential plane parallel surfaces are shown in the figure. The planes are inclined to x-axis by 45o and the distance

from one plane to another plane along X-axis is 20 cm. The electric field is

45o
X
20 cm 40 cm 60 cm 80 cm

(a) 177 Vm–1 at angle 135o with X-axis (b) 125 Vm–1 at angle 45o to the X-axis
(c) 177 Vm–1 at angle 45o to the X-axis (b) 125 Vm–1 at angle 135o to the X-axis

91. An electric field of strength 50 V m–1 exists along the negative direction of Y–axis. If 1 C of positive charge is shifted from a
point A (1 m, –1 m) to B (1 m, 3 m), the work done by agent is
152 Electrostatics

B(1, 3)

X
O
A (1, –1)

(a) 0 (b) – 0.2 mJ (c) + 0.2 mJ (d) + 0.8 mJ

92. A radioactive source in the form of a metal sphere of radius 10 –2 m, emits beta particles at the rate of 5  1010 particles per sec.
The source is electrically insulated. How long will it take for it’s potential to be raised by 2 volts, assuming 40% of the emitted
beta particles escape the source

(a) 700 sec (b) 700 milli sec (c) 700  sec (d) 700 n sec

Equilibrium, Motion of Charge, Neutral Point and E.P.E.

Basic Level

93. A bullet of mass 2g is having a charge of 2C. Through what potential difference must it be accelerated, starting from rest, to
acquire a speed of 10 m/s ?

(a) 50 kV (b) 5 V (c) 50 V (d) 5 kV

94. Three point charges are placed at the corners of an equilateral triangle. Assuming only electrostatic forces are acting

(a) The system can never be in equilibrium

(b) The system will be in equilibrium if the charge rotate about the centre of the triangle

(c) The system will be in equilibrium if the charges have different magnitudes and different signs

(d) The system will be in equilibrium if the charges have the same magnitudes but different signs

95. The distance between charges 5  10–11 C and – 2.7  10–11 C is 0.2 m. The distance at which a third charge should be placed
from – 2.7  10–11 C charge in order that it will not experience any force along the line joining the two charges is

(a) 0.44 m (b) 0.65 m (c) 0.556 m (d) 0.350

96. If 3 charges are placed at the vertices of equilateral triangle of charge ‘ q’ each. What is the net potential energy, if the side of
equilateral  is l cm

1 q2 1 2q 2 1 3q 2 1 4q 2
(a) (b) (c) (d)
4  0 l 4  0 l 4  0 l 4  0 l
97. A charged particle of mass 0.003 gm is held stationary in space by placing it in a downward direction of electric field of 6 
104 N/C. Then the magnitude of the charge is
(a) 5  10–4 C (b) 5  10–10 C (c) – 18  10–6 C (d) – 5  10–9 C

98. Cathode rays travelling from east to west enter into region of electric field directed towards north to south in the plane of paper.
The deflection of cathode rays is towards
Electrostatics 153

(a) East (b) South (c) West (d) North

99. An electron and a proton are kept in a uniform electric field. The ratio of their acceleration will be
mp me
(a) Unity (b) Zero (c) (d)
me mp

100. A drop of 10–6 kg water carries 10–6 C charge. What electric field should be applied to balance its weight (assume g = 10 m/s2)

(a) 10 V/m upward (b) 10 V/m downward (c) 0.1 V/m downward (d) 0.1 V/m upward

101. If identical charges (– q) are placed at each corner of a cube of side b, then electric potential energy of charge (+q) which is placed at
centre of the cube will be

8 2q 2  8 2q 2  4 2q 2  4q 2
(a) (b) (c) (d)
4  0 b  0 b  0 b 3 0 b

102. Two point charges + 9e and + e are at 16 cm away from each other. Where should another charge q be placed between them so
that the system remains in equilibrium

(a) 24 cm from + 9e (b) 12 cm from + 9e (c) 24 cm from + e (d) 12 cm from + e

103. Three charges Q, +q and +q are placed at the vertices of an equilateral triangle of side l as shown in the figure. If the net electric
energy of the system is zero, then Q is equal to

+q +q
l

q
(a)  (b) – q (c) +q (d) Zero
2

104. The acceleration of an electron in an electric field of magnitude 50 V/cm, if e/m value of the electron is 1.76  1011 C/kg is

(a) 8.8  1014 m/sec2 (b) 6.2  1013 m/sec2 (c) 5.4  1012 m/sec2 (d) Zero

105. A simple pendulum has a metal bob, which is negatively charged. If it is allowed to oscillate above a positively charged metallic
plate, then its time period will

(a) Increases (b) Decreases (c) Become zero (d) Remain the same

106. A particle of mass ‘m’ and charge ‘q’ is accelerated through a potential difference of V volt, its energy will be

q q
(a) qV (b) mqV (c)  V (d)
m  mV
154 Electrostatics

107. Consider two point charges of equal magnitude and opposite sign separated by certain distance. The neutral point due to them

(a) Does not exist

(b) Will be in mid way between them

(c) Lies on the perpendicular bisector of the line joining the two

(d) Will be closer to the negative charge

108. The dimension of (1/2).0E2 (0 : permittivity of free space; E : electric field) is

(a) MLT–1 (b) ML2T–2 (c) ML–1T–2 (d) ML2T–1

109. In a uniform electric field a charge of 3 C experiences a force of 3000 N. The p.d. between two points 1 cm apart along the
electric lines of force will be

(a) 10 V (b) 30 V (c) 100 V (d) 300 V

110. An electron enters an electric field with its velocity in the direction of the electric lines of force. Then

(a) The path of the electron will be a circle (b) The path of the electron will be a parabola

(c) The velocity of the electron will decrease (d) The velocity of the electron will increase

111. An electron of mass m and charge e is accelerated from rest through a p.d. V in vacuum. The final speed of the electron will be

(a) V e / m (b) eV / m (c) 2 eV / m (d) 2eV/m

112. In the figure distance of the point from A, where the electric field is zero is

A B

10 C 20 C
80 cm

(a) 20 cm (b) 10 cm (c) 33 cm (d) None of these

113. Two metal pieces having a potential difference of 800 V are 0.02 m apart horizontally. A particle of mass 1.96  10–15 kg is
suspended in equilibrium between the plates. If e is the elementary charge, then charge on the particle is

(a) e (b) 3e (c) 6e (d) 8e

114. A sphere of radius 1 cm has potential of 8000 V, then energy density near its surface will be

(a) 64  105 J/m3 (b) 8  103 J/m3 (c) 32 J/m3 (d) 2.83 J/m3
Electrostatics 155

115. A particle of mass m and charge q is placed at rest in a uniform electric field E and then released. The kinetic energy attained by
the particle after moving a distance y is

(a) qEy2 (b) qE2y (c) qEy (d) q2Ey

116. Two equal charges q are placed at a distance of 2 a and a third charge – 2q is placed at the midpoint. The potential energy of the
system is

q2 6q 2 7q 2 9q 2
(a) (b) (c)  (d)
8 0 a 8 0 a 8 0 a 8 0 a

117. In the diagram shown electric field intensity will be zero at a distance

–q 2q

(a) Between – q and +2q charges (b) Towards +2q on the line drawn

(c) Away from the line towards 2q (d) Away from the line towards – q

118. If an electron has an initial velocity in a direction different from that of an electric field, the path of the electron is

(a) A straight line (b) A circle (c) An ellipse (d) A parabola

119. Three charges 4q, Q and q are in a straight line in the position of 0, l/2 and l respectively. The resultant force on q will be zero. If
Q=

q
(a) – q (b) – 2 q (c)  (d) 4 q
2

120. A pendulum bob of mass 80 mg and carrying a charge of 2  10–8 C is at rest in a horizontal uniform electric field of 20,000 V/m.
The tension in the thread of the pendulum is

(a) 2.2  10–4 N (b) 4.4  10–4 N (c) 8.8  10–4 N (d) 17.6  10–4 N

Equilibrium, Motion of Charge, Neutral Point and E.P.E.

Advance Level
156 Electrostatics

121. An electron moving with the speed 5  10 6 per sec is shooted parallel to the electric field of intensity 1  10 3 N/C . Field is
responsible for the retardation of motion of electron. Now evaluate the distance travelled by the electron before coming to rest for

an instant (mass of e  9  10 31 Kg . charge  1 . 6  10 19 C )

(a) 7 m (b) 0.7 mm (c) 7 cm (d) 0.7 cm

122. A small sphere carrying a charge ‘q’ is hanging in between two parallel plates by a string of length L. Time period of pendulum is
T0 . When parallel plates are charged, the time period changes to T. The ratio T / T0 is equal to

+ + + + + + + + +
L
m

– – – – – – – – –

1/2 3/2 1/2


 qE     
g  
g
  g 
(a)  m  (b)   (c)   (d) None of these
 g   qE   qE 
  g  g 
   m   m 

123. Two equal point charges are fixed at x = – a and x = +a on the x-axis. Another point charge Q is placed at the origin. The change in the
electrical potential energy of Q, when it is displaced by a small distance x along the x-axis, is approximately proportional to

(a) x (b) x2 (c) x3 (d) 1/x

124. An elementary particle of mass m and charge +e is projected with velocity v towards a much more massive particle of charge Ze,
where Z > 0. What is the closest possible approach of the incident particle

(a) Ze2/20mv2 (b) Ze2/40mrn (c) Ze2/80rn (d) – Ze2/80rn

125. A ball of mass 1 g and charge 10–8 C moves from a point A. Where potential is 600 volt to the point B where potential is zero.
Velocity of the ball at the point B is 20 cm/s. The velocity of the ball at the point A will be

(a) 22.8 cm/s (b) 228 cm/s (c) 16.8 m/s (d) 168 m/s

126. An electron of mass me initially at rest moves through a certain distance in a uniform electric field in time t1. A proton of mass mp
also initially at rest takes time t2 to move through an equal distance in this uniform electric field. Neglecting the effect of gravity,

the ratio of t2/t1 is nearly equal to

(a) 1 (b) (mp/me)1/2 (c) (me/mp)1/2 (d) 1836

127. Point charges + 4q, – q and + 4q are kept on the x-axis at points x = 0, x = a and x = 2a respectively, then
Electrostatics 157

(a) Only – q is in stable equilibrium (b) None of the charges are in equilibrium

(c) All the charges are in unstable equilibrium (d) All the charges are in stable equilibrium

128. A mass m = 20 g has a charge q = 3.0 mC. It moves with a velocity of 20 m/s and enters a region of electric field of 80 N/C in the
same direction as the velocity of the mass. The velocity of the mass after 3 seconds in this region is

(a) 80 m/s (b) 56 m/s (c) 44 m/s (d) 40 m/s

129. An electron moves round a circular path of radius 0.1 m about an infinite linear charge of density +1 C/m. The speed of the
electron will be

(a) 5.6  103 m/s (b) 2.8  105 m/s (c) 5.6  107 m/s (d) 2.8  107 m/s

130. An electron falls through a distance of 8 cm in a uniform electric field of 105 N/C. The time taken by the electron in falling will be

(a) 3  10–6 s (b) 3  10–7 s (c) 3  10–8 s (d) 3  10–9 s

131. Two particles, each of mass 10 g and having charge of 1C are in equilibrium on a horizontal table at a distance of 50 cm. The
coefficient of friction between the particles and the table is

(a) 0.18 (b) 0.54 (c) 0.36 (d) 0.72

132. A small ball of mass (36 ) gm has a charge of 10–8 C on it. It is suspended by a thread from a vertical charged metal plate.
In equilibrium the thread makes an angle of 45 o with the plate. If g = 10 m/s2, then the charge density on the plate is

(a) 10–9 C/m2 (b) 10–8 C/m–2 (c) 10–7 C/m2 (d) 10–6 C/m2

133. A charged plate has charge density of 2  10–6 C/m2. The initial distance of an electron which is moving towards plate, cannot
strike the plate, if it is having energy of 200 eV

(a) 1.77 mm (b) 3.51 mm (c) 1/77 cm (d) 3.51 cm

134. A piece of cloud having area 25  106 m2 and electric potential of 105 volts. If the height of cloud is 0.75 km, then energy of
electric field between earth and cloud will be

(a) 250 J (b) 750 J (c) 1225 J (d) 1475 J

135. 10  C charge is uniformly distributed over a thin ring of radius 1 m. A particle (mass = 0.9 gram , charge 1  c) is placed on the
axis of ring. It is displaced towards centre of ring, then time period of oscillations of particle

(a) 0.6 sec (b) 0.2 sec (c) 0.3 sec (d) 0.4 sec

136. Two identical particles of same mass are having same magnitude of charge Q. One particle is initially at rest on a frictionless
horizontal plane and the other particle is projected directly towards the first particle from a very large distance with a velocity v.

The distance of closest approach of the particle will be


158 Electrostatics

1 4Q 2 1 2Q 2 1 Q2 1 4Q 2
(a) (b) (c) (d)
4  0 mv 2 4  0 mv 2 4  0 m 2 v 2 4  0 m 2 v 2

137. A very small sphere of mass 80 gm having a charge Q is held at a height 9m velocity above the centre of a fixed conducting
sphere of radius 1m, carrying an equal charge Q, when released is falls until it is repelled just before it comes in contact with the

sphere. What will be the charge Q (g = 9.8 m/s2)

(a) 28 mC (b) 28 C (c) 28 C (d) None of these

138. A thin conducting ring of radius r has an electric charge + Q, if a point charge q is placed at the centre of the ring, then tension of
the wire of ring will be

Qq Qq Qq Qq
(a) (b) (c) (d)
8 0 r 2 4  0 r 2 8 2  0 r 2 4 2  0 r 2

139. A particle of specific charge (q/m) enters into uniform electric field E along the centre line, with velocity v. After how much time it
will collide with one of the plate (figure) +Q

E
d

–Q
l

d d md 2md
(a) (b) (c) (d)
V 2V qE qE

140. A dust particle of radius 5  10–7 m lies in an electric field of 6.28  105 V/m. The surrounding medium is air whose coefficient of
viscosity is 1.6  10–5 N-s/m2. If the particle moves with a horizontal uniform velocity of 0.02 m/s, the number of electrons on it is

(a) 10 (b) 20 (c) 30 (d) 40

141. A drop of water of mass 18  10–3 gm falls from a hole of the bottom of a charged conducting sphere of radius 20 cm, carrying
with it a charge of 10–9 C and leaving a uniformly distributed charge of 2.5  10–6 C on the sphere. The speed of drop after it had
fallen by 30 cm, will be

(a) 3.67 m/s (b) 4.62 m/s (c) 1.61 m/s (d) 3.06 m/s

142. How should three charge q, 2q and 8q be arranged on a 9 cm long line such that the potential energy of the system is minimum
?

(a) q at a distance of 3 cm from 2q (b)q at a distance of 5 cm from 2q

(c) 2q at a distance of 7 cm from q (d)2q at a distance of 9 cm from q

143. A proton and an -particle are situated at r distance apart. At very large distance apart when released, the kinetic energy of
proton will be

2ke 2 8 ke 2 ke 2 8 ke 2
(a) (b) (c) (d)
r 5 r r r
Electrostatics 159

Electric dipole, Flux and Gauss’s Law

Basic Level

144. An electric dipole has the magnitude of its charge as q and its dipole moment is p. It is placed in a uniform electric field E. If its
dipole moment is along the direction of the field, the force on it and its potential energy are respectively

(a) q.E and p.E (b) Zero and minimum (c) q.E and maximum (d) 2q.E and minimum

145. Shown below is a distribution of charges. The flux of electric field due to these charges through the surface S is

+q +q

+q

(a) 3 q /  0 (b) 2 q /  0 (c) q /0 (d) Zero

146. A charge q is located at the centre of a cube. The electric flux through any face is
4 q q q 2q
(a) (b) (c) (d)
6(4  0 ) 6(4  0 ) 6(4  0 ) 6(4  0 )

147. If the electric flux entering and leaving an enclosed surface respectively is  1 and  2 , the electric charge inside the surface will
be

(a) (1   2 ) 0 (b) ( 2  1 ) 0 (c) (1   2 ) /  0 (d) ( 2  1 ) /  0

148. q1 , q 2 , q 3 and q 4 are point charges located at points as shown in the figure and S is a spherical Gaussian surface of radius R.
Which of the following is true according to the Gauss’s law

q1 R
q4
q2 q3

    q q q     (q  q  q )
(a)
 (E
s
1  E 2  E 3 ).d A  1 2

2 0
3
(b)
 (E
s
1  E 2  E 3 ).d A  1 2

0
3
160 Electrostatics

    (q  q  q  q )
(c)
 (E
s
1  E 2  E 3 ).d A  1 2

0
3 4
(d) None of the above

149. The distance between H+ and Cl– ions in HCl molecule is 1.28Å. What will be the potential due to this dipole at a distance of 12Å
on the axis of dipole

(a) 0.13 V (b) 1.3 V (c) 13 V (d) 130 V

150. The potential at a point due to an electric dipole will be maximum and minimum when the angles between the axis of the dipole
and the line joining the point to the dipole are respectively

(a) 90o and 180o (b) 0o and 90o (c) 90o and 0o (d) 0o and 180o
   
151. When an electric dipole p is placed in a uniform electric field E then at what angle between P and E the value of torque will
be maximum

(a) 90o (b) 0o (c) 180o (d) 45o

152. According to Gauss’s theorem, electric field of an infinitely long straight wire is proportional to

1 1 1
(a) r (b) (c) (d)
2 3 r
r r

153. The electric field at a distance ‘r’ from an electric dipole is proportional to

(a) 1/r (b) 1/r2 (c) 1/r3 (d) r2

154. Water is an excellent solvent because its molecules are

(a) Neutral (b) Highly polar (c) Non-polar (d) Anodes

155. An electric dipole is placed in an electric field generated by a point charge

(a) The net electric force on the dipole must be zero (b) The net electric force on the dipole may be zero

(c) The torque on the dipole due to the field must be zero (d) The torque on the dipole due to the field may be zero

156. Eight dipoles of charges of magnitude e are placed inside a cube. The total electric flux coming out of the cube will be

8e 16 e e
(a) (b) (c) (d) Zero
0 0 0

157. A cube of side l is placed in a uniform field E, where E  Eˆi . The net electric flux through the cube is

(a) Zero (b) l 2 E (c) 4l2E (d) 6l2E

158. The distance between a proton and electron both having a charge 1.6  10–19 coulomb, of a hydrogen atom is 10 10 metre . The
value of intensity of electric field produced on electrons due to proton will be

(a) 2.304  10–10 N/C (b) 14.4 V/m (c) 16 V/m (d) 1.44  1011 N/C
Electrostatics 161

159. The electric field at a point on equatorial line of a dipole and direction of the dipole

(a) Will be parallel (b) Will be in opposite direction (c) Will be perpendicular (d) Are not related

160. For a given surface the Gauss’s law is stated as  E.ds  0 . From this we can conclude that
(a) E is necessarily zero on the surface (b) E is perpendicular to the surface at every point

(c) The total flux through the surface is zero (d) The flux is only going out of the surface

161. An electric dipole when place in a uniform electric field E will have minimum potential energy, if the positive direction of dipole
moment makes the following angle with E

(a)  (b) /2 (c) Zero (d) 3/2

Electric dipole, Flux and Gauss’s Law

Advance Level

162. Consider the charge configuration and a spherical Gaussian surface as shown in the figure. When calculating the flux of the
electric field over the spherical surface, the electric field will be due to

+q1 q2

–q1

(a) q2 (b) Only the positive charges (c) All the charges (d) + q1 and – q1

163. Two electric dipoles of moment P and 64 P are placed in opposite direction on a line at a distance of 25 cm. The electric field will
be zero at point between the dipoles whose distance from the dipole of moment P is

25 4
(a) 5 cm (b) cm (c) 10 cm (d) cm
9 13

164. Two point charges +q and – q are held fixed at (– d, 0) and (d, 0) respectively of a (X, Y) co-ordinate system. Then

(a) E at all points on the Y-axis is along î

(b) The electric field E at all points on the X-axis has the same direction

(c) Dipole moment is 2qd directed along î

(d) Work has to be done in bringing a test charges from infinity to the origin
162 Electrostatics

165. The electric dipole is situated in an electric field as shown in adjacent figure. The dipole and the electric field are both in the
plane of the paper. The dipole is rotated about an axis perpendicular the plane of the paper about its axis at a point A in anti-

clockwise direction. If the angle of rotation is measured with respect to the direction of the electric field, then the torque for

different values of the angle of rotation ‘’ will be represented in fig. given below by the

1 2 3 4
C

A +q 2
–q 
O 

(a) Curve (1) (b) Curve (2) (c) Curve (3) (d) Curve (4)

166. As shown in the figure q1 = 1c, q2 = 2c and q3 = – 3 c and S1, S2, S3 and S4 are four closed surfaces. The values of electric
flux coming out of the surfaces S1 and S2 will respectively be

S4
q3
q1 S1 q2
S2

(a) Zero, 1.113  10–5 V/m (b) 1.13  105 V-m and zero

(c) 1.13  105 V-m and – 1.13  105 V-m (d) – 1.13  105 V-m and 1.13  105 V-m

167. What will be the charge present inside a cube which produces electric field Ex = 600 x1/2, Ey = 0, Ez = 0

y
0.1 m

z 0.1 m
Electrostatics 163

(a) 600 C (b) 60 C (c) 7 C (d) 6 C

168. There exists a non-uniform electric field along x-axis as shown in figure. The field increases at a uniform rate along positive x-
axis. A dipole is placed inside the field as shown. For the dipole which one of the following statement is correct

–q
(a) Dipole moves along positive x-axis and undergoes a clockwise rotation X-axis
+q

(b) Dipole moves along negative x-axis after undergoing a clockwise rotation

(c) Dipole moves along positive x-axis after under going an anticlockwise rotation

(d) Dipole moves along negative x-axis and undergoes an anticlockwise rotation

169. A point charge +q is at a distance d/2 from a square surface of side d and is directly above the centre of the square as shown in
fig. The electric flux through the square is

d/2

(a) q/0 (b) q/60 (c) q/0 (d) Zero

Capacitance and Capacitor

Basic Level

170. A parallel plate capacitor carries a charge q. The distance between the plates is doubled by application of a force. The work
done by the force is

q2 q2 q2
(a) Zero (b) (c) (d)
C 2C 4C

171. A parallel plate capacitor of capacity C 0 is charged to a potential V0

(i) The energy stored in the capacitor when the battery is disconnected and the separation is doubled E1

(ii) The energy stored in the capacitor when the charging battery is kept connected and the separation between the capacitor
plates is doubled is E 2 . Then E1 / E 2 value is

(a) 4 (b) 3/2 (c) 2 (d) ½


164 Electrostatics

172. Capacitance of a parallel plate capacitor becomes 4/3 times its original value if a dielectric slab of thickness t = d/2 is inserted
between the plates (d is the separation between the plates). The dielectric constant of the slab is

(a) 8 (b) 4 (c) 6 (d) 2

173. As shown in the figure, a very thin sheet of aluminium in placed in between the plates of the condenser. Then the capacity

(a) Will increase (b) Will decrease (c) Remains unchanged (d) May increase or decrease

174. The work done in placing a charge of 8  10 18 coulomb on a condenser of capacity 100 micro-farad is

(a) 32  10 32 joule (b) 16  10 32 joule (c) 3 . 1  10 26 joule (d) 4  10 10 joule

175. What fraction of the energy drawn from the charging battery is stored in a capacitor ?

(a) 100% (b) 75% (c) 50% (d) 25%

176. Capacitance (in F) of a spherical conductor with radius 1 m is

(a) 1 . 1  10 10 (b) 10 6 (c) 9  10 9 (d) 10 5

177. Work done by an external agent in separating the parallel plate capacitor is

1 2 1
(a) CV (b) C V (c) CV 2 (d) None of these
2 2

178. A parallel plate capacitor has an electric field of 10 5 V / m between the plates. If the charge on the capacitor plate is 1 C , the
force on each capacitor plate is

(a) 0.5 N (b) 0.05 N (c) 0.005 N (d) None of these

179. A conducting sphere of radius 10 cm is charged 10 C . Another uncharged sphere of radius 20 cm is allowed to touch it for
some time. After that if the spheres are separated, then surface density of charges on the spheres will be in the ratio of

(a) 1 : 4 (b) 1 : 3 (c) 2 : 1 (d) 1 : 1

180. If the distance between parallel plates of a capacitor is halved and dielectric constant is doubled then the capacitance

(a) Decreases two times (b) Increases two times (c) Increases four times (d) Remain the same

181. N identical spherical drops are charged to the same potential V . They are combine to form a bigger drop. The potential of the
big drop will be

(a) VN1/3 (b) VN2/3 (c) V (d) VN


Electrostatics 165

182. A capacitor is used to store 24 watt hour of energy at 1200 volt. What should be the capacitance of the capacitor

(a) 120 mF (b) 120 F (c) 12 F (d) 24 mF

183. Change Q on a capacitor varies with voltage V as shown in the figure, where Q is taken along the X-axis and V along the Y-axis.
The area of triangle OAB represents

A
V

X
O B
Q

(a) Capacitance (b) Capacitive reactance

(c) Magnetic field between the plates (d) Energy stored in the capacitor

184. A solid conducting sphere of radius R 1 is surrounded by another concentric hollow conducting sphere or radius R 2 . The
capacitance of this assembly is proportional to

R 2  R1 R 2  R1 R1 R 2 R1 R 2
(a) (b) (c) (d)
R1 R 2 R1 R 2 R 2  R1 R 2  R1

185. A condenser having a capacity 2.0 microfarad is charged to 200 volts and then the plates of the capacitor are connected to a
resistance wire. The heat produced in joules will be

(a) 4  10 4 Joule (b) 4  10 10 Joule (c) 4  10 2 Joule (d) 2  10 2 Joule

186. A metallic sheet is inserted between plates parallel to the plates of a parallel plate capacitor. The capacitance of the capacitor

(a) Increases

(b) Is independent of the position of the sheet

(c) Is maximum when the metal sheet is in the middle

(d) Is maximum when the metal sheet touches one of the capacitor plates

187. The capacity of parallel plate condenser depends on

(a) The type of metal used (b) The thickness of plates

(c) The potential applied across the plates (d) The separation between the plates

188. A variable condenser is permanently connected to a 100 V battery. If the capacity is changed from 2 F to 10 F , then change
in energy is equal to

(a) 2  10 2 J (b) 2 .5  10 2 J (c) 3 .5  10 2 J (d) 4  10 2 J


166 Electrostatics

189. The capacity of a parallel plate capacitor with no dielectric substance but with a separation of 0.4 cm is 2 F . The separation is
reduced to half and it is filled with a dielectric substance of value 2.8. The final capacity of the capacitor is

(a) 11 . 2 F (b) 15 . 6 F (c) 19 . 2 F (d) 22 . 4 F

190. The capacity o a condenser is 4  10–6 farad and its potential is 100 volt. The energy released on discharging it fully will be
(a) 0.02 joule (b) 0.04 joule (c) 0.025 joule (d) 0.05 joule

191. When we touch the terminals of a high voltage capacitor, even after a high voltage has been cut off, then the capacitor has a
tendency to

(a) Restore energy (b) Discharge energy (c) Affect dangerously (d) Both (b) and (c)

192. A parallel plate air capacitor is charged to a potential difference of V. After disconnecting the battery, distance between the plates
of the capacitor is increased using an insulating handle. As a result, the potential difference between the plates

(a) Decreases (b) Increases (c) Becomes zero (d) Does not change

193. A 10 pF capacitor is connected to a 50 V battery. How much electrostatic energy is stored in the capacitor ?

(a) 1 . 25  10 8 J (b) 2 . 5  10 7 J (c) 3 .5  10 5 J (d) 4 . 5  10 2 J

194. When a dielectric material is introduced between the plates of a charged condenser, the electric field between the plates
(a) Decreases (b) Increases (c) Remain constant (d) First ‘b’ then ‘a’

195. Two protons A and B are placed in space between plates of a parallel plate capacitor charged upto V volts (see fig.) Forces on
protons are FA and FB , then
+ –

+ –
+ –
A B
+

+

(a) FA  FB (b) FA  FB (c) FA  FB (d) Nothing can be said

196. A condenser is charged and then battery is removed. A dielectric plate is put between the plates of condenser, then correct
statement is

(a) Q constant, V and U decrease (b) Q constant, V increases, U decreases

(c) Q increases, V decreases, U increases (d) None of these

197. 1000 small water drops each of radius r and charge q coalesce together to form one spherical drop. The potential of the big drop
is larger than that of the smaller drop by a factor of

(a) 1000 (b) 100 (c) 10 (d) 1


Electrostatics 167

198. Two metal spheres of radii 1cm and 2 cm are given charges of 102 C and 5  10 2 C respectively. If both spheres are joined by a
metal wire, then the final charge on the smaller spheres will be

(a) 2  10 2 C (b) 4  10 2 C (c) 3  10 2 C (d) 4  10 2 C

199. A condenser is charged and then battery is removed. A dielectric plate is put between the plates of condenser, then correct
statement is

(a) Q constant, V and U decrease (b) Q constant, V increases, U decreases

(c) Q increases, V decreases, U increases (d) None of these

200. 1000 small water drops each of radius r and charge q coalesce together to form one spherical drop. The potential of the big drop
is larger than that of the smaller drop by a factor of

(a) 1000 (b) 100 (c) 10 (d) 1

201. Two metal spheres of radii 1cm and 2 cm are given charges of 102 C and 5  10 2 C respectively. If both spheres are joined by a
metal wire, then the final charge on the smaller spheres will be

(a) 2  10 2 C (b) 4  10 2 C (c) 3  10 2 C (d) 4  10 2 C

202. A capacitor of capacity C has charge Q and stored energy is W. If the charge is increased to 2Q, the stored energy will be

(a) 2W (b) W/2 (c) 4W (d) W/4

203. 64 drops each having the capacity C and potential V are combined to form a big drop. If the charge on the small drop is q, then
the charge on the big drop will be

(a) 2q (b) 4q (c) 16q (d) 64q

204. The capacity of a parallel plate condenser is 5 F. When a glass plate is placed between the plates of the conductor, its potential
becomes 1/8th of the original value. The value of dielectric constant will be

(a) 1.6 (b) 5 (c) 8 (d) 40

205. Which one statement is correct ? A parallel plate air condenser is connected with a battery. Its charge, potential, electric field and
energy are Q0, V0, E0 and U0 respectively. In order to fill the complete space between the plates a dielectric slab is inserted, the

battery is still connected. Now the corresponding values Q, V, E and U are in relation with the initially stated as

(a) Q > Q0 (b) V > V0 (c) E > E0 (d) U > U0

206. The capacity of a parallel plate air capacitor is 10 F and it is given a charge 40 C. the electrical energy stored in the capacitor
in ergs is

(a) 80  106 (b) 800 (c) 8000 (d) 20000


168 Electrostatics

207. There is an air filled 1 pF parallel plate capacitor. When the plate separation is doubled and the space is filled with wax, the
capacitance increases to 2 pF. The dielectric constant of wax is

(a) 2 (b) 4 (c) 6 (d) 8

208. A parallel plate capacitor is charged and the charging battery is then disconnected. If the plates of the capacitor are moved
further apart by means of insulating handles, then

(a) The charge on the capacitor increases

(b) The voltage across the plates decreases

(c) The capacitance increases

(d) The electrostatics energy stored in the capacitor increases

209. An air capacitor is connected to a battery. The effect of filling the space between the plates with a dielectric is to increase

(a) The charge and the potential difference (b) The potential difference and the electric field

(c) The electric field and the capacitance (d) The charge and the capacitance

210. Between the plates of a parallel plate condenser there is 1 mm thick paper of dielectric constant 4. It is charged at 100 volt . The
electric field in volt/metre between the plates of the capacitor is

(a) 100 (b) 100000 (c) 25000 (d) 400000

211. A capacitor is kept connected to the battery and a dielectric slab is inserted between the plates. During this process

(a) No work is done

(b) Work is done at the cost of the energy already stored in the capacitor before the slab is inserted

(c) Work is done at the cost of the battery

(d) Work is done at the cost of both the capacitor and the battery

212. A capacitor with air as the dielectric is charged to a potential of 100 volts. If the space between the plates is now filled with a
dielectric of dielectric constant 10, the potential difference between the plates will be

(a) 1000 volts (b) 100 volts (c) 10 volts (d) Zero

213. The distance between the circular plates of a parallel plate condenser 40 mm in diameter, in order to have same capacity as a
sphere of radius 1 metre is

(a) 0.01 mm (b) 0.1 mm (c) 1.0 mm (d) 10 mm

214. Force acting upon a charged particle kept between the plates of a charged condenser is F. If one plate of the condenser is
removed, then the force acting on the same particle will become
Electrostatics 169

(a) 0 (b) F/2 (c) F (d) 2F

215. Two metallic charged spheres whose radii are 20 cm and 10 cm respectively, have each 150 micro-coulomb positive charge.
The common potential after they are connected by a conducting wire is

(a) 9  106 volts (b) 4.5  106 volts (c) 1.8  107 volts (d) 13.5  106 volt

216. A capacitor of capacity C is connected with a battery of potential V in parallel. The distance between its plates is reduced to half
at once, assuming that the charge remains the same. Then to charge to capacitance upto the potential V again, the energy given

by the battery will be

(a) CV2/4 (b) CV2/2 (c) 3CV2/4 (d) CV2

Capacitance and Capacitor

Advance Level

217. If on charging a capacitor current is kept constant then the variation of potential V of the capacitor with time t is shown as

(a) (b) (c) (d)

V V V V

t t t t

218. Two capacitors of capacitance 2 and 3F are joined in series. Outer plate first capacitor is at 1000 volt and outer plate of
second capacitor is earthed (grounded). Now the potential on inner plate of each capacitor will be

(a) 700 Volt (b) 200 Volt (c) 600 Volt (d) 400 Volt

219. In the given figure each plate of capacitance C has partial value of charge

E R

R2

C R1
170 Electrostatics

CER 1 CER 2 CER 1


(a) CE (b) (c) (d)
R2  R R2  R R1  R

220. A parallel plate capacitor has plate area A and separation d. It is charged to a potential difference V0 . The charging battery is
disconnected and the plates are pulled apart to three times the initial separation. The work required to separate the plates is

3 0 AV 02  0 AV 02  0 AV 02  0 AV 02
(a) (b) (c) (d)
d 2d 3d d
Charge on the capacitor after 1 second
221. A charged 100F capacitor is discharged through a 10 k resistor. The ratio is
Original charge on the capacitor

(a) (1 – 1/e) (b) ln 2 (c) (1 – ln 2) (d) 1/e

222. The area of the plates of a parallel plate capacitor is A and the distance between the plates is 10 mm. There are two dielectric
sheets in it, one of dielectric constant 10 and thickness 6 mm and the other of dielectric constant 5 and thickness 4 mm. The
capacity of the condenser is
12 2 5000
(a) 0 A (b) 0 A (c) 0 A (d) 1500 0A
35 3 7

223. A 500 F capacitor is charged at a steady rate of 100 C/sec. The potential difference across the capacitor will be 10 V after an
interval of

(a) 5 sec (b) 20 sec (c) 25 sec (d) 50 sec

224. The space between the plates of a parallel plate capacitor is filled completely with a dielectric substance having dielectric
constant 4 and thickness 3 mm. The distance between the plates in now increased by inserting a second sheet of thickness 5
mm and dielectric constant K. If the capacitance of the capacitor so formed is one-half of the original capacitance, the value of K
is

(a) 10/3 (b) 20/3 (c) 5/3 (d) 15/3

225. A capacitor of capacitance 160 F is charged to a potential difference of 200 V and then connected across a discharge tube,
which conducts until the potential difference across it has fallen to 100 V. The energy dissipated in the tube is

(a) 6.4 J (b) 4.8 J (c) 3.2 J (d) 2.4 J

226. A 0.1 F capacitor filled completely with a dielectric and it is charged until the p.d. between the plates becomes 25 V. Then the
charge is shared with a similar capacitor which has air as dielectric. The potential difference falls to 15 V. The dielectric constant

of the first capacitor is

(a) 2.5 (b) 1.5 (c) 7.5 (d) 5.5

227. A parallel plate capacitor of plate area A and plate separation d is charged to potential V and then the battery is disconnected. A
slab of dielectric constant K is then inserted between the plates of the capacitors so as to fill the space between the plates. If Q,

E and W denote respectively, the magnitude of charge on each plate, the electric field between the plates (after the slab is
inserted) and work done on the system in the process of inserting the slab, then state incorrect relation from the following
Electrostatics 171

 0 AV 2 2
(d) W   0 AV  1  1 
 0 AV V
(a) Q  (b) W  (c) E 
d 2 Kd Kd 2d K 

228. A dielectric slab of thickness d is inserted in a parallel plate capacitor whose negative plate is at x = 0 and positive plate is at x =
3d. The slab is equidistant from the plates. The capacitor is given some charge. As x goes from 0 to 3d

(a) The magnitude of the electric field remains the same

(b) The direction of the electric field remains the same

(c) The electric potential increases continuously

(d) The electric potential increases at first, then decreases and again increases

229. A capacitor of capacitance C0 is charged to a potential V0 and then isolated. A small capacitor C is then charged from C0,
discharged and charged again; the process being repeated n times. Due to this, potential of the larger capacitor is decreased to

V. Value of C is
n
V 
1/n
 V 
1/n
  V    V 
n

(a) C 0  0  (b) C  0   1 (c) C    1
 (d) C    1

V   V    V0    V0
  

Grouping of Capacitors

Basic Level

230. Three capacitors each of capacity 4F are to be connected in such a way that the effective capacitance is 6 F. This can be
done by

(a) Connecting them in parallel (b)Connecting two in series and one in parallel

(c) Connecting two in parallel and one in series (d)Connecting all of them in series

231. A 0.2 F capacitor is charged to 600 V. After removing it from battery it is connected to another capacitor of 1F in parallel. The
voltage on the capacitor will become

(a) 300 V (b) 600 V (c) 100 V (d) 120 V

232. Two identical capacitors, have the same capacitance C. One of them is charged to potential V1 and the other to V2. The negative
ends of the capacitors are connected together. When the positive ends are also connected, the decrease in energy of the
combined system is

1 1 1 1
(a) C(V1 2  V2 2 ) (b) C(V1 2  V2 2 ) (c) C(V1  V2 ) 2 (d) C(V1  V2 ) 2
4 4 4 4

233. If there are n -capacitors in parallel connected to V volt source, then the energy stored is equal to

1 1
(a) CV (b) nCV 2 (c) CV 2 (d) CV 2
2 2n
172 Electrostatics

234. Two capacitors C1 and C 2  2C1 are connected in a circuit with a switch between them as shown in the figure. Initially the
switch is open and C1 holds charge Q . The switch is closed. At steady state, the charge on each capacitor will be

Q C1

R
S

C2 = 2C1

(a) Q, 2 Q (b) Q / 3, 2 Q / 3 (c) 3 Q / 2, 3 Q (d) 2 Q / 3, 4 Q / 3

235. Two capacitors of 1 F and 2 F are connected in series, the resultant capacitance will be
2 3
(a) 4 F (b) F (c) F (d) 3 F
3 2
236. A capacitor of 10 F charged up to 250 volts is connected in parallel with another capacitor of 5 F charged up to 100 volts.
The common potential is

(a) 500 V (b) 400 V (c) 300 V (d) 200 V

237. A body of capacity 4 F is charged to 80 V and another body of capacity 6 F is charged to 30 V . When they are connected
the energy lost by 4 F capacitor is

(a) 1. 8 mJ (b) 4 . 6 mJ (c) 3 . 2 mJ (d) 2 . 5 mJ

238. A 20 F capacitor is charged to 5 V and isolated. It is then connected in parallel with an uncharged 30 F capacitor. The
decrease in the energy of the system will be

(a) 25 J (b) 200 J (c) 125 J (d) 150 J

239. Capacitance of an air filled parallel plate capacitor is 10 F. If two dielectric medium are filled as shown in figure then equivalent
capacitance will be

K1 = 2 K2 = 4

(a) 30 F (b) 15 F (c) 5 F (d) 10 F

240. Three capacitors of capacitance 3F, 10F and 15 F are connected in series to a voltage source of 100 V. The charge on 15
F is

(a) 25 C (b) 100 C (c) 200 C (d) 280 C


Electrostatics 173

241. A parallel plate capacitor has capacitance C. If it is equally filled with parallel layers of materials of dielectric constant K1 and K2
its capacity becomes C1. The ratio of C1 to C is

K1 K 2 K1  K 2 2 K1 K 2
(a) K1  K 2 (b) (c) (d)
K1  K 2 K1 K 2 K1  K 2

242. A capacitor of capacity C1, is charged by connecting it across a battery of e.m.f. V0. The battery is then removed and the
capacitor is connected in parallel with an uncharged capacitor of capacity C2. The potential difference across this combination is

C2 C1 C1  C 2 C1  C 2
(a) .V0 (b) .V0 (c) .V0 (d) .V0
C1  C 2 C1  C 2 C2 C1

243. Two capacitors with capacitances C1 and C 2 are charged to potentials V1 and V2 respectively. When they are connected in
parallel, the ratio of their respective charges is

V12 V1 C12 C1
(a) (b) (c) (d)
V22 V2 C 22 C2

244. Two condensers of capacity 0.3 F and 0.6 F respectively are connected in series. The combination is connected across a
potential of 6 volts. The ratio of energies stored by the condensers will be

1 1
(a) (b) 2 (c) (d) 4
2 4

245. Three capacitors of capacitances 3 F are connected once in series and another time in parallel. The ratio of equivalent
 Cs 
capacitance in the two cases   will be
 Cp 
 

(a) 1 : 9 (b) 9 : 1 (c) 1 : 1 (d) 1 : 3

10
246. Seven capacitors each of capacity 2F are to be so connected to have a total capacity F. Which will be the necessary
11
figure as shown ?

(a) (b)

(c) (d)
174 Electrostatics

247. Three capacitors are connected to d.c. source of 100 volts as shown in the adjoining figure. If the charge accumulated on plates
of C1, C2 and C3 are qa, qb, qc, qd, qe and qf respectively, then

2F 3F 4F

a b c d e f

100 Volts

100
(a) q b  q d  q f  coulombs (b) q b  q d  q f  0
9

(c) qa + qc + qe = 50 coulombs (d) qb = qd = qf

248. Four capacitors of each of capacity 3F are connected as shown in the adjoining figure. The ratio of equivalent capacitance
between A and B and between A and C will be

A B

(a) 4 : 3 (b) 3 : 4 (c) 2 : 3 (d) 3 : 2

249. The capacities and connected of five capacitors are shown in the adjoining figure. The potential difference between the points A
and B is 60 volts. Then the equivalent capacity between A and B and the charge on 5 F capacitance will be respectively

12F 10F 5F 9F 8F


A B

(a) 44 F; 300 C (b) 16 F; 150 C (c) 15 F; 200 C (d) 4 F; 50C

250. Three identical capacitors are combined differently. For the same voltage to each combination, the one that stores the greatest
energy is

(a) Two in parallel and the third in series with it (b) Three in series
Electrostatics 175

(c) Three in parallel (d) Two in series and third in parallel with it

251. The charge on a capacitor of capacitance 10 F connected as shown in the figure is

2

2 10F

2V

(a) 20 C (b) 15 C (c) 10 C (d) Zero

252. The equivalent capacitance between A and B in the figure is 1 F. Then the value of capacitance C is

C
A

2.5F 1F

(a) 1.4 F (b) 2.5 F (c) 3.5 F (d) 1.2 F

253. A condenser of capacity C1 is charged to a potential V0. The electrostatic energy stored in it is U0. It is connected to another
uncharged condenser of capacity C2 in parallel. The energy dissipated in the process is

C2 C1  C1  C 2  C1 C 2
(a) U0 (b) U0 (c)   U 0 (d) U0
C1  C 2 C1  C 2  C1  C 2  2(C 1  C 2 )

254. Minimum number of capacitors of 2 F capacitance each required to obtain a capacitor of 5 F will be

(a) Three (b) Four (c) Five (d) Six

255. 2 F capacitance has p.d. across its two terminals 200 volts. It is disconnected with battery and then another uncharged
capacitance is connected in parallel to it, then p.d. becomes 20 volts. Then the capacity of another capacitance will be

(a) 2 F (b) 4 F (c) 18 F (d) 16 F

256. Two capacitors each of capacity 2 F are connected in parallel. This system is connected in series with a third capacitance of 12
F capacity. The equivalent capacity of the system will be

(a) 16 F (b) 13 F (c) 4 F (d) 3 F


176 Electrostatics

257. A 4 F condenser is connected in parallel to another condenser of 8 F. Both the condensers are then connected in series with a
12 F condenser and charged to 20 volts. The charge on the plate of 4 F condenser is

(a) 3.3 C (b) 40 C (c) 80 C (d) 240 C

258. In the diagram below are shown three capacitors C 1 , C 2 and C 3 joined to battery. With symbols having their usual meanings,
the correct conditions will be
V2
C2
Q2
V1
C1
Q1 V3
Q C3
Q3

(a) Q 1  Q 2  Q 3 and V1  V2  V3  V (b) Q 1  Q 2  Q 3 and V  V1  V2  V3

(c) Q 1  Q 2  Q 3 and V  V1  V2 (d) Q 3  Q 2 and V2  V3

Grouping of Capacitors

Advance Level

259. If four plates each of area A are arranged according to the given diagram with distance d between neighboring plates then the
capacitance of the system between A and B will be

A
B

4 0 A 3 0 A 2 0 A 0 A
(a) (b) (c) (d)
d d d d

260. Four metallic plates, each with a surface area of one side A are placed at a distance d from each other. The plates are
connected as shown in the figure. Then the capacitance of the system between A and B is

B
Electrostatics 177

3 0 A 2 0 A 2 0 A 3 0 A
(a) (b) (c) . (d) .
d d 3 d 2 d

261. The equivalent capacity between A and B in the adjoining figure will be

1
A
2
B
3

0 A 3 0 A 2 0 A 2 0 A
(a) (b) (c) (d)
d 2 d d 3d

262. Consider the situation shown in the figure. The capacitor A has a charge q on it whereas B is uncharged. The charged
appearing on the capacitor B a long time after the switch is closed is

q
+ –
+ –
+ – s
+ –
+ –
A B

(a) Zero (b) q / 2 (c) q (d) 2q

263. Two capacitors of capacitances 3 F and 6 F are charged to a potential of 12 V each. They are now connected to each other,
with the positive plate of each joined to the negative plate of the other. The potential difference across each will be

(a) 6 V (b) 4 V (c) 3V (d) Zero

264. A capacitor of capacity C1 is charged to the potential of V0. ON disconnecting with the battery, it is connected with a capacitor of
capacity C2 as shown in the adjoining figure. The ratio of energies before and after the connection of switch S will be

C1V0 C2

(a) (C1 + C2)/C1 (b) C1/(C1 + C2) (c) C1C2 (d) C1/C2

265. The two metallic plate of radius r are placed at a distance d apart and its capacity is C. If a plate of radius r/2 and thickness d of
dielectric constant 6 is placed between the plates of the condenser, then its capacity will be

(a) 7C/2 (b) 3C/7 (c) 7C/3 (d) 9C/4


178 Electrostatics

266. The capacity of the capacitors are shown in the adjoining fig. The equivalent capacitance between the points A and B and the
charge on the 6 F will be
A
90 volt

B
9F 6F 12F

(a) 27 F; 540 C (b) 15 F; 270 C (c) 6 F; 180 C (d) 15 F; 90 C

267. A parallel plate capacitor of capacitance C is connected to a battery and is charged to a potential difference V. Another capacitor
of capacitance 2C is connected to another battery and is charged to potential difference 2 V. The charging batteries are now
disconnected and the capacitors are connected in parallel to each other in such a way that the positive terminal of one is
connected to the negative terminal of the other. The final energy of the configuration is

25 CV 2 3 CV 2 9 CV 2
(a) Zero (b) (c) (d)
6 2 2

268. Two identical parallel plate capacitors are connected in series to a battery of 100 V. A dielectric slab of dielectric constant 4.0 is
inserted between the plates of second capacitor. The potential difference across the capacitor will now be respectively

(a) 50 V, 50 V (b) 80 V, 20 V (c) 20 V, 80 V (d) 75 V, 25 V

269. In the circuit shown here C1 = 6F, C2 = 3 F and battery B = 20 V. The switch S1 is first closed. It is then opened and afterwards
S2 is closed. What is the charge finally on C2

C2 3F

S2
C1 6F

S1

B = 20V

(a) 120 C (b) 80 C (c) 40 C (d) 20 C

270. In the give circuit if point C is connected to the earth and a potential of + 2000 V is given to the point A, the potential at B is

10F 10F

5F
A B C

10F
Electrostatics 179

(a) 1500 V (b) 1000 V (c) 500 V (d) 400 V

271. A 10 F capacitor and a 20 F capacitor are connected in series across a 200 V supply line. The charged capacitors are then
disconnected from the line and reconnected with their positive plates together and negative plates together and no external

voltage is applied. What is the potential difference across each capacitor

800 800
(a) V (b) V (c) 400 V (d) 200 V
9 3

272. An uncharged capacitor with a solid dielectric is connected to a similar air capacitor charged to a potential of V0. If the common
potential after sharing of charges becomes V, then the dielectric constant of the dielectric must be

V0 V (V0  V ) (V0  V )
(a) (b) (c) (d)
V V0 V V0

273. The capacitance of a parallel plate condenser is C1 (fig. a). A dielectric of dielectric constant K is inserted as shown in figure (b)
and (c). If C2 and C3 are the capacitances in figure (b) and (c), then

d/2
d K
K

(a) (b) (c)

(a) Both C2 and C3 > C1 (b) C3 > C1 but C2 < C1

(c) Both C2 and C3 < C1 (d) C1 = C2 = C3

274. The potential difference between the points Q and S of the given circuit is

Q
C1 C2

P R

C3 C4
S
V B
A

(C 2  C 1 ) (C 4  C 3 ) (C 2 C 3  C1 C 4 )V (C 4 C 1  C 2 C 3 )V
(a) V (b) V (c) (d)
C1 C3 (C1  C 2  C 3  C 4 ) (C 1  C 2 )(C 3  C 4 )

275. Five capacitors are connected as shown in the diagram. If the p.d. between A and B is 22 V, the emf of the cell is
180 Electrostatics

10F 2.9F
B
D
3F

12F C
7F

A
E

(a) 26 V (b) 42 V (c) 38 V (d) 46 V

276. In the circuit shown in the figure the amount of charge that will flow through any section of the connecting wires to the battery
when the key K is closed is 2F
4F

4F
K

300 V

(a) 800C (b) 1800C (c) 1200C (d) 1600C

277. In the following figure, the charge on each condenser in the steady state will be

3F 4 3F

4

3F 3F
4

1
10V

(a) 3C (b) 6 C (c) 9C (d) 12C

278. The figure shows a circuit with E as the earthing of the common plate. The potentials at P and Q are

P + – + – Q
10F E 15F

+ –
1000 V
Electrostatics 181

(a) 0V, – 1000 V (b) 1000 V, 0 V (c) + 600 V, – 400 V (d) + 400 V, – 600 V

279. In the given circuit, with steady current, the potential drop across the capacitor must be

V R

V C

2V 2R

V V 2V
(a) V (b) (c) (d)
2 3 3

280. A parallel plate capacitor is charged to a potential difference of 50 V . It is discharged through a resistance. After 1 second, the
potential difference between plates becomes 40 V . Then
(a) Fraction of stored energy after 1 second is 16/25 (b) P.d. between the plates after 2 seconds will be 32 V
4
(c) P.d. between the plates after 2 seconds will be 20 V (d) Fraction of stored energy after 1 second is
5
281. The equivalent capacitance of three capacitors of capacitance C1, C2 and C3 connected in parallel is 12 units and the product
C1C2C3 = 48. When the capacitors C1 and C2 are connected in parallel the equivalent capacitance is 6 units. Then the
capacitance are
(a) 1.5, 2.5, 8 (b) 2,3,7 (c) 2,4,6 (d) 1,5,6

282. Two condensers C1 and C2 in a circuit are joined as shown in figure. The potential of point A is V1 and that of B is V2. The
potential of point D will be
A D B

V1 C1 C2 V2

1 C 2 V1  C 1 V2 C 1 V1  C 2 V2 C 2 V1  C 1 V2
(a) (V1  V2 ) (b) (c) (d)
2 C1  C 2 C1  C 2 C1  C 2

283. Three plates A, B, C each of area 50 cm2 have separation 3 mm between A and B and 3 mm between B and C. The energy
stored when the plates are fully charged is

A
B
C 12V
182 Electrostatics

(a) 1.6  10–9 J (b) 2.1  10–9 J (c) 5  10–9 J (d) 7  10–9 N

284. Five identical plates are connected across a battery as follows. If the charge on plate 1 be +q, then the charges on the plates 2,
3, 4 and 5 are 1

2
3

4
5

(a) – q, + q, – q, + q (b) – 2q, + 2q, – 2q, +q (c) – q, + 2q, – 2q, + q (d) None of the above

285. One plate of a parallel plate capacitor is suspended from a beam of a physical balance as shown in the figure. The area of each
plate is 625 cm2 and the distance between these plates is 5 mm. If an additional mass 0.04 gm is placed in the other pan of the
balance, then the potential difference required between the plates to keep it in equilibrium will be

mg

(a) 150 V (b) 188 V (c) 225 V (d) 310 V

286. The V versus x plot for six identical metal plates of cross-sectional area A is as shown. What will be the equivalent capacitance
between 2 and 5 (The plates are placed with a separation d)

1 2 3 4 5 6

v 10 V
5V

I. O x
d 2d 3d 4d 5d

2 0 A 0 A 3 0 A 4 0 A
(a) (b) (c) (d)
d d d d

287. Two parallel metal plates are inserted at equal distances into a parallel plate capacitor as shown in the figure. Plates 1 and 4 are
connected to a battery of emf . With reference to the positive plate of the battery at zero potential, the potential of other plates

are respectively
1 2 3 4

V
Electrostatics 183

V V V 2V
(a) 0, V, V, V (b) 0, , ,V (c) 0, , ,V (d) 0, 0, 0, 0
2 3 3 3

288. Four plates, each of area A and each side are placed parallel to each other at a distance d. A battery is connected between the
combinations 1 and 3 and 2 and 4. The modulus of charge on each plate is

1 2 3 4

2 0 A 3 0 A 2 0 A 0 A
(a) E (b) E (c) E (d) E
d d 3d d

289. Three capacitors are connected as shown in figure. Then the charge on C1 is

C1 C2

2F 2F

C3 4F

+ – – +
6V 6V

(a) 6 C (b) 12 C (c) 18 C (d) 24 C


184 Electrostatics

Practice Networks (Find equivalent capacitance between X and Y)

Basic Level

1.

4F (a) 3  F
X

(b) 2  F
4F 2F 2F
(c) 4F

(d) 8  F
4F Y

2.

C 5
C (a) C
8

X Y 3
(b) C
C 5
C 5
(c) C
3

(d) C

3.
C
Y (a) 2C

C (b) C/5
C C
(c) 5C
X C (d) C/2

4.

4F 4F
(a) 8F

(b) 6F
4F

Y (c) 268 F
X
4F 4F
10
(d) F
38
Electrostatics 185

5.
15F X (a) 15F

(b) 20F
15F 15F
(c) 25F

15F Y
(d) 30F

6.
C1 = 6F
(a) 24 F
C3 = 6F C2 = 6F
(b) 18 F
X C5 = 6F Y
(c) 12 F
C4 = 6F
(d) 6 F

7.
12F
X (a) 47 F

(b) 3 F
2F 3F

(c) 60 F
Y
20F (d) 10 F

8.
X
(a) 2 . 2 F
5F 10F

(b) 3 . 2 F
4F
(c) 1 . 2 F
Y

(d) 4 . 7 F

9.
2F 12F
(a) 28/9F

(b) 4F
2F
X Y

2F
186 Electrostatics

(c) 5F

(d) 18F

10.
2F
1
2F 5F 2F (a) F
1F 2
X Y (b) 1 F

2F 1F (c) 2 F

(d) 1.33 F

11.

10F
(a) 40 F
10F
(b) 20 F
10F
X Y
(c) 30 F
10F 10F
(d) 10 F

12.
X
(a) 3C
2C 2C
(b) 2C
2C
C
(c) C
C C
Y C
(d)
3

13.
3C 3C 3C 3C
(a) C
X

2C 2C 2C 3C (b) 9 C

Y (c) 0.86 C
3C 3C 3C 3C
(d) 5.0 C
Electrostatics 187

14.

C C C C (a) Zero
X
 (b) Infinite
C C C C
(c) 0.62 C
Y
(d) 1.62 C

15.
2F
X (a) 10 F
8F 1F 1F (b) 20 F

4F 4F (c) 5 F


Y
(d) 2.5 F

16.
2F
Each 3F
(a) 3F
Each 6F
(b) 4F
X Y

(c) 5F
Each 4F
(d) 6F

17.
2F
Each 3F
(a) 3 F
Each 6F

X
(b) 6 F
Y

(c) 1.5 F
Each 4F
(d) 9 F

Practice Networks (Find equivalent capacitance between X and Y)

Advance Level
188 Electrostatics

18.
6F 3F
(a) 3.6 F
X 6F 9F
4F (b) 6 F
9F
5F (c) 1.5 F
Y (d) 9 F

19.
C
X (a) C/2
C C
(b) C

(c) 2C
C C C
Y (d) 6C

20.
C C
(a) C
C C
(b) C/2
C C
X Y (c) 2C
C C
(d) 3C
Electrostatics 189

21.
Y
5
X (a) C
C 3
2
C C (b) C
3
C
(c) C

(d) 2C

22.

(a) C
X C 2C 4C 8C
Y (b) C/2

(c) 2C

(d) 

23.
2F
X
4F
(a) 17/6F
2F
4F (b) 12/17F

3F 3F (c) 6/5 F


Y
(d) 3 F

24.
C C
(a) C
C
C (b) 2C
C
X Y
C (c) 3C
C C (d) 4C

25.
C
(a) C/10
C C
C C C
C C
X C Y
190 Electrostatics

(b) 10C/3

(c) 3C/10

(d) 9C

26.
X – +
Y
(a) C
C
C (b) 5C
C
(c) 2C
C
C (d) 3C

27.
C
C (a) C
C
C (b) 2C

C (c) 3C
X C Y (d) 4C

28.
C X – + Y
(a) 6C

(b) 5C
C C C
(c) 3C
C C
(d) 2C

29.
C Y
(a) C
C
(b) 4C
C C C C C
C (c) 6C

X C
Electrostatics 191

(d) 0

30.

C (a) C
C C C (b) 2C

(c) 3C
+ –
X Y
(d) 2C/3

31.

(a) 32 F
8F 8F 8F 8F
(b) 2 F
X
(c) 8 F

Y (d) 16 F

32.
X 10  0 A
a (a)
5a
a 9 0 A
(b)
7a
a
Y 11 0 A
a a (c)
a 8a
12  0 A
(d)
18 a

33.
1F
19
X (a) F
56
64
2F 3F 2F (b) F
11
1F
56
(c) F
Y 11
192 Electrostatics

9
(d) F
37

34.

C 5
(a) C
C C 6
C
X C C
C (b)
C 6
C C
C Y C
(c)
C 5
C
6C
(d)
5

35.

1F X (a) 1 F
1F
16 Capacitor

1F 1F
8 Capacitor

  (b) 2 F
1F
1F 1
1F
(c) F
2
Y
(d) 

36.

2l 3C
X (a)
l 7
l 7C
(b)
l Y 3
2l (c) 5C

(d) 2C

37.
C

C C 4C
C (a)
C C 5
X C Y 5C
C (b)
C C C 4
C (c) 12 C
C
(d)
12

***
Electrostatics 193

+

+

– +


+

Answer Sheet
Assignment (Basic & Advance Level)

1 2 3 4 5 6 7 8 9 10 11 12 13 14 15 16 17 18 19 20

c b a a a c b c b d b a b a a b c c d c

21 22 23 24 25 26 27 28 29 30 31 32 33 34 35 36 37 38 39 40

c d c d a c b c c a, c b d b c c c b a a b

41 42 43 44 45 46 47 48 49 50 51 52 53 54 55 56 57 58 59 60

d d d b c c b a c a a c c d b c b a b a

61 62 63 64 65 66 67 68 69 70 71 72 73 74 75 76 77 78 79 80

a b b b a c d c b c, d a b a a c a c c c c

81 82 83 84 85 86 87 88 89 90 91 92 93 94 95 96 97 98 99 100

b d c d b d d c a c c c a a c c b d c a

101 102 103 104 105 106 107 108 109 110 111 112 113 114 115 116 117 118 119 120

d b a a b a a c a c c c b d c c d d a c

121 122 123 124 125 126 127 128 129 130 131 132 133 134 135 136 137 138 139 140

c c c a a b c b c d c a a d a a b c c c

141 142 143 144 145 146 147 148 149 150 151 152 153 154 155 156 157 158 159 160

a a b b b a b b a b a d c b d d a d b c

161 162 163 164 165 166 167 168 169 170 171 172 173 174 175 176 177 178 179 180

c c a a b c c c b c a d c b c a c b c c

181 182 183 184 185 186 187 188 189 190 191 192 193 194 195 196 197 198 199 200

b a d d c a, b d d a a d b a a c a b a a b

201 202 203 204 205 206 207 208 209 210 211 212 213 214 215 216 217 218 219 220

a c d c d b d d d b d c b d a d a d c d

221 222 223 224 225 226 227 228 229 230 231 232 233 234 235 236 237 238 239 240
194 Electrostatics

d c d b d b b b, c b b c c b b b d a d a c

241 242 243 244 245 246 247 248 249 250 251 252 253 254 255 256 257 258 259 260

d b d b a a d a d c a a a b c d b c b d

261 262 263 264 265 266 267 268 269 270 271 272 273 274 275 276 277 278 279 280

d a b a d c c b c c a c a d d b d c c a

281 282 283 284 285 286 287 288 289

c c b b b b c d a

Assignment (Practice Networks)

1 2 3 4 5 6 7 8 9 10 11 12 13 14 15 16 17 18 19 20

c c a a b d b b c b d a a c c c a a c c

21 22 23 24 25 26 27 28 29 30 31 32 33 34 35 36 37

a b d b c d c d a d a c d d b a b

Das könnte Ihnen auch gefallen